Sunteți pe pagina 1din 112

Matemática II

Farith Briceño - 2016


Material en revisión
Indice

1 Antiderivada. 3

2 Método de integración: Manipulación algebraica. 19

3 Método de integración: u-sustitución. 37

4 Notación sigma. 61

5 Integral definida. 73

6 Teorema Fundamental del Cálculo. 95

Última actualización: Julio 2016 Farith J. Briceño N. farith.math@gmail.com


Matemática II - Guía 1
Antiderivada.

Objetivos a cubrir Código : MAT-CI.1


• Definición de antiderivadas.
• Integral indefinida. Propiedades de la integral indefinida. Ejercicios resueltos

1
Ejemplo 1 : Demuestre que si f (x) = arcsen x, entonces f ′ (x) = √ , con −1 < x < 1.
1 − x2
Demostración : Es conocido que la función inversa de g (x) = sen x, es f (x) = arcsen x, definida en
−1 ≤ x ≤ 1, es decir, g −1 (x) = f (x), además si una función g tiene inversa y es diferenciable, entonces g −1
es diferenciable y su derivada viene dada por
′ 1
g −1 (x) = .
g ′ (g −1 (x))
Como g ′ (x) = cos x, se tiene que
′ ′ 1
g −1 (x) = (arcsen x) = .
cos (arcsen x)
De la identidad trigonométrica básica,
sen2 (·) + cos2 (·) = 1,
despejamos cos (·) y obtenemos p
cos (·) = ± 1 − sen2 (·).
h π πi
Al componer la expresión del cos (·) con la función f (x) = arcsen x, como Rgo f = − , y el coseno
2 2
es positivo en ese intervalo,

y 1 y
π y = arcsen x y = cos x
2

− π2 π
2
3π −π 0
−1 1 x − 2 π 3π x
2

π
−2
−1

h π πi h π πi
Rgo f = − , Se tiene cos x ≥ 0 si x ∈ − ,
2 2 2 2
por lo tanto, se toma la expresión positiva del coseno y se tiene,
p q p
2
cos (arcsen x) = 1 − sen2 (arcsen x) = 1 − (sen (arcsen x)) = 1 − x2 .

Luego,
′ 1
(arcsen x) = √ ,
1 − x2
definida para −1 < x < 1. ⋆

Última actualización: Julio 2016 Farith J. Briceño N. farith.math@gmail.com


Matemática II - Guía 1. Antiderivada. 4

Ejemplo 2 : Hallar una función f , tal que se cumpla la siguiente igualdad


Z
f (x) dx = arcsen x + C.

Solución : Por la definición de primitiva se tiene que cumplir



(arcsen x + C) = f (x) ,
así,
′ ′ ′ 1 1
(arcsen x + C) = (arcsen x) + (C) = √ +0= √ .
| {z } | {z } |{z} 1−x2 1 − x2
↑ ↑ ↑
Derivada de una Derivada: Derivada de
suma de funciones Ver ejemplo 1 una constante

Luego,
1
f (x) = √ .
1 − x2

Ejemplo 3 : Hallar una función f , tal que se cumpla la siguiente igualdad
Z
√ 
f (x) dx = arctan x + C.

Solución : Por la definición de primitiva se tiene que cumplir


√  ′
arctan x + C = f (x)
así,
√  ′ √ ′ ′ 1 √ ′ 1 1
arctan x + C = arctan x + (C) = √ 2 ( x) + 0 = √ .
| {z } | {z } |{z} 1 + ( x) 1 + x 2 x
↑ ↑ ↑
Derivada de una Derivada: Regla Derivada de
suma de funciones de la cadena una constante

Luego,
1
f (x) = √ .
2 x (1 + x)

Z
Ejemplo 4 : Integre −2−5 dx.

Solución : Se tiene
xn+1
Z
xn dx = +C con n=0
n+1


Z zZ }| {
x0+1
−2−5 dx = −2−5 1 dx = −2−5 + C = −2−5 x + C.
| {z } 0+1

Linealidad de la integral
Sale de la integral por ser constante
respecto a la variable de integración

Luego, Z
−2−5 dx = −2−5 x + C.

Última actualización: Julio 2016 Farith J. Briceño N. farith.math@gmail.com


Matemática II - Guía 1. Antiderivada. 5

Z

Ejemplo 5 : Integre x dx.

Solución : Es conocido que √


x = x1/2 ,
entonces Z Z 1
√ x 2 +1 x3/2 2x3/2
x dx = x1/2 dx = +C = +C = + C.
1 3 3
| {z } +1
2 2

xn+1 1
Z
xn dx = +C con n=
n+1 2

Luego,
Z
√ 2x3/2
x dx = + C.
3

Z
Ejemplo 6 : Integre 2 csc2 x dx.

Solución : Se tiene
(cot x)′ = − csc2 x


Z zZ }| {
2 2
2 csc x dx = 2 csc x dx = −2 cot x + C.

Linealidad de la integral
Sale de la integral por ser constante
respecto a la variable de integración

Luego, Z
2 csc2 x dx = −2 cot x + C.


Z
Ejemplo 7 : Integre (πx + sec x tan x) dx.

Solución : Se tiene

Linealidad de la integral
Z Z Z
(f (x) + g (x)) dx = f (x) dx + g (x) dx

Z ↓ Z Z Z Z
(πx + sec x tan x) dx = πx dx + sec x tan x dx = π x dx + sec x tan x dx,

Linealidad de la integral
Sale de la integral por ser constante
respecto a la variable de integración

donde Z Z
x1+1 x2
x dx = + C1 = + C1 y sec x tan x dx = sec x + C2 ,
1+1 2
| {z } | {z }
↑ ↑
xn+1
Z
xn dx = +C con n=1 (sec x)′ = sec x tan x
n+1

Última actualización: Julio 2016 Farith J. Briceño N. farith.math@gmail.com


Matemática II - Guía 1. Antiderivada. 6

entonces
Z  2 
x πx2 πx2
(πx + sec x tan x) dx = π + C1 + sec x + C2 = + C1 π + sec x + C2 = + sec x + (C1 π + C2 )
2 2 2 | {z }

Constante C

Luego, Z
πx2
(πx + sec x tan x) dx = + sec x + C.
2

Z  
1
Ejemplo 8 : Integre x2 + 1 + 2 dx.
x

Solución : Se tiene

Linealidad de la integral
Z Z Z
(f (x) + g (x)) dx = f (x) dx + g (x) dx

Z 
1
 ↓ Z Z Z
1
Z Z Z
2 2
x + 1 + 2 dx = x dx + 1 dx + dx = x dx + 1 dx + x−2 dx,
2
x x2

donde
Z Z
x2+1 x3 x0+1
x2 dx = + C1 = + C1 , mientras que 1 dx = + C2 = x + C2 ,
2+1 3 0+1
| {z } | {z }
↑ ↑
xn+1 xn+1
Z Z
xn dx = +C con n=2 xn dx = +C con n=0
n+1 n+1

por último Z
x−2+1 x−1 1
x−2 dx = + C3 = + C3 = − + C3 .
−2 + 1 −1 x
| {z }

xn+1
Z
xn dx = +C con n = −2
n+1

Entonces
Z    
2 1 x3 1 x3 1
x + 1 + 2 dx = + C1 + x + C2 + − + C3 = + x − + (C1 + C2 + C3 )
x 3 x 3 x | {z }

Constante C

Luego, Z  
1 x3 1
x2 + 1 + 2 dx = + x − + C.
x 3 x

Z √ 
5
2
2
Ejemplo 9 : Integre x − √
5
dx.
x3
Solución : Se tiene, por linealidad de la integral indefinida que

Última actualización: Julio 2016 Farith J. Briceño N. farith.math@gmail.com


Matemática II - Guía 1. Antiderivada. 7

Linealidad de la integral
Sale de la integral por ser constante
respecto a la variable de integración

Z √  Z √ Z ↓ Z √ Z
5
2
2 5
2
2 5
2 dx − 2
1
x − √
5
dx = x dx − √
5
dx = x √
5
dx,
x3 ↑ x 3 x3
Linealidad de la integral
Z Z Z
(f (x) − g (x)) dx = f (x) dx − g (x) dx

donde Z √ Z 2
5 x 5 +1 x7/5 5x7/5
x dx = x2/5 dx =
2 + C1 = + C1 = + C1 ,
2 7 7
| {z } +1
5 5

xn+1 2
Z
xn dx = +C con n=
n+1 5

mientras que
Z Z Z 3
1 1 −3/5 x− 5 +1 x2/5 5x2/5
√ dx = dx = x dx = + C2 = + C2 = + C2 ,
5
x3 x3/5 3 2 2
| {z } − +1
5 5

xn+1 3
Z
xn dx = +C con n=−
n+1 5

entonces
Z √   2/5 
5
2
2 5x7/5 5x 5x7/5 5x2/5
x − √ dx = + C1 − 2 + C2 = + C1 − 2 − 2C2
5
x3 7 2 7 2

5x7/5
= − 5x2/5 + (C1 − 2C2 )
7 | {z }

Constante C

Luego,
Z √ 
5 2 5x7/5
x2 − √ dx = − 5x2/5 + C.
5
x3 7

Z
√ 
Ejemplo 10 : Integre x − 5x dx.

Solución : Se tiene

Linealidad de la integral
Z Z Z
(f (x) − g (x)) dx = f (x) dx − g (x) dx

Z
√  ↓ Z √ Z Z

Z
x − 5x dx = x dx − 5x dx = x dx − 5 x dx,

Linealidad de la integral
Sale de la integral por ser constante
respecto a la variable de integración

Última actualización: Julio 2016 Farith J. Briceño N. farith.math@gmail.com


Matemática II - Guía 1. Antiderivada. 8

donde Z Z 1
√ 1/2 x 2 +1 x3/2 2x3/2
x dx = x dx = + C1 = + C1 = + C1 ,
1 3 3
| {z } +1
2 2

xn+1 1
Z
xn dx = +C con n=
n+1 2

mientras que Z
x1+1 x2
x dx = + C2 = + C2 ,
1+1 2
| {z }

xn+1
Z
n
x dx = +C con n=1
n+1

entonces
Z  2 
√  2x3/2 x 2x3/2 5x2 2x3/2 5x2
x − 5x dx = + C1 − 5 + C2 = + C1 − − 5C2 = − + (C1 − 5C2 )
3 2 3 2 3 2 | {z }

Constante C

Luego, Z
√  2x3/2 5x2
x − 5x dx = − + C.
3 2

Z  
5 3
Ejemplo 11 : Integre sen x + −√ dx.
1 + x2 1 − x2
Solución : Se tiene
Linealidad de la integral Linealidad de la integral
Sale de la integral por ser constante
Z Z Z
(f (x) + g (x)) dx = f (x) dx + g (x) dx
respecto a la variable de integración

Z   ↓ Z Z ↓ Z ↓
5 3 5 3
sen x + 2
−√ dx = sen x dx + dx − √ dx
1+x 1 − x2 1 + x2 1 − x2
Z Z Z
1 1
= sen x dx + 5 dx − 3 √ dx,
1 + x2 1 − x2
donde
Z Z Z
dx dx
sen x dx = − cos x + C1 , = arctan x + C2 y √ = arcsen x + C3
1 + x2 1 − x2
| {z } | {z } | {z }
↑ ↑ ↑
1 1
(cos x)′ = − sen x (arctan x)′ = (arcsen x)′ = √
1 + x2 1 − x2

entonces
Z  
5 3
sen x + 2
−√ dx = − cos x + C1 + 5 (arctan x + C2 ) − 3 (arcsen x + C3 )
1+x 1 − x2
= − cos x + C1 + 5 arctan x + 5C2 − 3 arcsen x − 3C3

= − cos x + 5 arctan x − 3 arcsen x + (C1 + 5C2 − 3C3 )


| {z }

Constante C

Última actualización: Julio 2016 Farith J. Briceño N. farith.math@gmail.com


Matemática II - Guía 1. Antiderivada. 9

Luego, Z  
5 3
sen x + −√ dx = − cos x + 5 arctan x − 3 arcsen x + C.
1 + x2 1 − x2

Ejemplo 12 : Encuentre,
√ en el plano xy, la curva y = f (x) que pasa por el punto (9, 4) y cuya pendiente
en cada punto es 3 x.

Solución : Es conocido que la pendiente de la recta tangente en un punto cualquiera x es mtan = f ′ (x),
por lo tanto, √
f ′ (x) = 3 x.

Para obtener f integramos respecto a x,


Z Z

f ′ (x) dx = 3 x dx =⇒ f (x) = 2x3/2 + C,

puesto que la función f pasa por el punto (9, 4) se tiene que


3/2 3/2
4 = f (9) = 2 (9) +C =⇒ 4 = 2 32 +C =⇒ 2 = 27 + C =⇒ −25 = C.

Luego,
f (x) = 2x3/2 − 25.

d2 y
Ejemplo 13 : En cualquier punto (x, y) de una curva se tiene = 4 − x2 y una ecuación de la recta
dx2
tangente a la curva en el punto (1, −1) es 2x − 3y = 3. Encontrar una ecuación de la curva.

Solución : Tenemos que


Z 2 Z
d y  dy x3
dx = 4 − x2 dx =⇒ = 4x − + C1 ,
dx2 dx 3
del hecho que la ecuación de la recta tangente a la curva en el punto (1, −1) es 2x − 3y = 3, se tiene que
2
f ′ (1) = , por lo tanto,
3
3
2 (1) 2 1
= f ′ (1) = 4 (1) − + C1 =⇒ = 4 − + C1 =⇒ C1 = −3
3 3 3 3
entonces,
dy x3
= 4x − − 3,
dx 3
integramos nuevamente
Z Z  
dy x3 x4
dx = 4x − − 3 dx =⇒ y = 2x2 − − 3x + C2 ,
dx 3 12
la curva pasa por el punto (1, −1) así,
4
(1) 1 1
−1 = 2 (1) − − 3 (1) + C2 =⇒ −1 = 2 − − 3 + C2 =⇒ C2 = .
12 12 12

Luego
x4 1
y = 2x2 − − 3x + .
12 12

Última actualización: Julio 2016 Farith J. Briceño N. farith.math@gmail.com


Matemática II - Guía 1. Antiderivada. 10

Ejemplo 14 : Se lanza una piedra hacia arriba verticalmente desde el suelo con una velocidad inicial de 128
p/seg. Si la única fuerza considerada es la que se le atribuye a la aceleración de la gravedad, encontrar que tan
alto llegará la piedra y la velocidad con la que llegará al suelo. Encontrar también cuanto tiempo tomará a la
piedra llegar al suelo.

Solución : La dirección positiva se toma hacia arriba. Sea


• t : el tiempo, en segundos, que ha transcurrido desde que se lanzó la piedra.
• s : la distancia, en pies, de la piedra al suelo a los t seg de tiempo.
• υ : la velocidad, en pies por segundos, de la piedra a los t seg de tiempo.
• |υ| : el número de pies por segundo en la rapidez, en pies por segundos, de la piedra a los t seg de tiempo.
La piedra estará en su punto más alto cuando la velocidad sea cero. Sea s el valor particular de s cuando
υ = 0. Cuando la piedra toca el suelo, s = 0. Sean t y υ los valores particulares de t, υ cuando s = 0 y
t 6= 0.

La dirección positiva de la piedra desde el punto de partida se toma hacia arriba. Como la única aceleración
se debe a la gravedad que actúa en dirección hacia abajo, la aceleración tiene un valor constante de −32 p/seg2.

Es conocido que la aceleración a es la primera derivada de υ con respecto a t y la segunda derivada de


s con respecto a t, es decir
dυ d2 s
a= = 2 = −32
dt dt
integramos respecto a t
Z Z 2 Z
dυ d s ds
dt = 2
dt = −32 dt =⇒ υ (t) = = −32t + C1
dt dt dt
como υ = 128 cuando t = 0, tenemos

128 = υ (0) = −32 (0) + C1 =⇒ C1 = 128,

por lo tanto,
ds
= −32t + 128
dt
integramos, nuevamente, respecto a t
Z Z
ds
dt = (−32t + 128) dt =⇒ s (t) = −16t2 + 128t + C2 ,
dt
como s = 0 cuando t = 0, tenemos

0 = s (0) = −16 (0)2 + 128 (0) + C2 =⇒ C2 = 0

y nos queda
s (t) = −16t2 + 128t.
La piedra estará en su punto más alto cuando la velocidad sea cero, así,
128
0 = υ (t) = −32t + 128 =⇒ t= = 4,
32
es decir, la piedra tarda 4 seg para llegar a su punto más alto y la distancia es
2
s (4) = −16 (4) + 128 (4) =⇒ s (4) = 256,

por lo tanto, la mayor altura que la piedra alcanzará es de 256 pies. Para conocer con que velocidad llegará la
piedra al suelo igualamos la función distancia a cero, de allí, obtenemos

0 = s (t) = −16t2 + 128t =⇒ 0 = −16t (t − 8) =⇒ t=0 y t = 8,

Última actualización: Julio 2016 Farith J. Briceño N. farith.math@gmail.com


Matemática II - Guía 1. Antiderivada. 11

pero el valor t = 0 corresponde al momento en que es lanzada la piedra, por lo tanto, la piedra llega al piso en
8 seg, luego la velocidad con la que llega es

υ (8) = −32 (8) + 128 = −128 =⇒ |υ| = 128,

es decir, la piedra llega al suelo con una rapidez de 128 p/seg. ⋆

Ejercicios

1
1. Demuestre que si f (x) = arcsen x, entonces f ′ (x) = √ , con −1 < x < 1.
1 − x2
1
2. Demuestre que si f (x) = arctan x, entonces f ′ (x) = .
1 + x2
1
3. Demuestre que si f (x) = arccos x, entonces f ′ (x) = − √ , con −1 < x < 1.
1 − x2
1
4. Demuestre que si f (x) = sec−1 x, entonces f ′ (x) = √ , con |x| > 1.
|x| x2 − 1
5. Suponga que
d √  d
f (x) = 1− x y g (x) = (x + 2)
dx dx
Encuentre
Z Z Z Z
1. f (x) dx 2. g (x) dx 3. (−f (x)) dx 4. (−g (x)) dx

Z Z Z Z
f (x)
5. dx 6. 2g (x) dx 7. (f (x) + g (x)) dx 8. (f (x) − g (x)) dx
4

6. Suponga que
d √  d
f (x) = 5− x−3 y g (x) = (sen x − π)
dx dx
Encuentre
Z Z Z Z
f (x)
1. g (x) dx 2. dx 3. (f (x) − g (x)) dx 4. (−5g (x)) dx
4
Z Z Z Z
g (x)
5. −7f (x) dx 6. dx 7. (f (x) + g (x)) dx 8. (f (x) − 2g (x)) dx
e4

7. Hallar una función f , tal que se cumpla la siguiente igualdad


Z Z Z
x2
1. f (x) dx = x + C 2. f (x) dx = mx + C 3. f (x) dx = +C
2
Z Z Z
x3 x4 √
4. f (x) dx =
+C 5. f (x) dx = +C 6. f (x) dx = 2 x + C
3 4
Z √ Z √3 Z √
3
2 x3 3 x4 3 x2
7. f (x) dx = +C 8. f (x) dx = +C 9. f (x) dx = +C
3 4 2
Z Z √ Z
2 3
x xn+1
10. f (x) dx = √ + C 11. f (x) dx = +C 12. f (x) dx = +C
x 3 n+1

Última actualización: Julio 2016 Farith J. Briceño N. farith.math@gmail.com


Matemática II - Guía 1. Antiderivada. 12

Z Z Z
13. f (x) dx = sen x + C 14. f (x) dx = − cos x + C 15. f (x) dx = tan x + C
Z Z Z
16. f (x) dx = sec x + C 17. f (x) dx = − csc x + C 18. f (x) dx = − cot x + C
Z Z
19. f (x) dx = sen (2x) + C 20. f (x) dx = − cos (4x) + C
Z Z
21. f (x) dx = tan (πx) + C 22. f (x) dx = sen (3x) + C
Z Z
23. f (x) dx = sec (5x) + C 24. f (x) dx = − csc (2x) + C
Z Z √
25. f (x) dx = − cot (3x) + C 26. f (x) dx = 3x + C
Z Z
27. f (x) dx = sec (ax) + C 28. f (x) dx = arctan x + C

Z Z Z
tan2 x sec2 t sec3 x
29. f (x) dx = +C 30. f (t) dt = +C 31. f (x) dx = +C
2 2 3
Z Z

32. f (t) dt = arctan t2 + C 33. f (x) dx = arcsen x + C

Z  Z
sec2 3t3 tan2 (5t)
34. f (t) dt = +C 35. f (t) dt = +C
2 4
Z Z
sec−3 x csc3 x
36. f (x) dx = +C 37. f (x) dx = +C
3 3
Z Z
cot2 t √ 
38. f (t) dt = +C 39. f (x) dx = arctan x +C
2
Z Z

3

40. f (x) dx = arcsen (2x) + C 41. f (x) dx = arctan x +C
Z Z
 
42. f (x) dx = arcsen x2 + C 43. f (t) dt = arctan t−2 + C

Z Z √
sec−1 (3x) sec−1 x
44. f (x) dx = +C 45. f (x) dx = +C
π x+1
8. Hallar las primitivas de las siguientes funciones
Z Z Z Z Z Z
1
1. dx 2. m dx 3. x dx 4. x2 dx 5. x3 dx 6. √ dx
x
Z Z Z Z √ Z
√ √ 1 3 1
7. x dx 8. 3
x dx 9. √ dx 10. x5 dx 11. √ dx
3
x 5
x4
Z Z Z Z Z
12. xn dx 13. cos x dx 14. sen x dx 15. sec2 x dx 16. csc2 x dx

Z Z Z Z
dx dx
17. sec x tan x dx 18. csc x cot x dx 19. 20. √
1 + x2 1 − x2

Última actualización: Julio 2016 Farith J. Briceño N. farith.math@gmail.com


Matemática II - Guía 1. Antiderivada. 13

9. Con los resultados obtenidos en el ejercicio 8 completar la siguiente tabla

Tabla de integrales básicas


Z Z
k dx = , k = constante xn dx = , para n 6= −1
Z Z
sen x dx = cos x dx =
Z Z
sec2 x dx = csc2 x dx =
Z Z
sec x tan x dx = csc x cot x dx =
Z Z
1 1
dx = √ dx =
1 + x2 1 − x2

10. Calcular las siguientes integrales usando la tabla de integrales de funciones básicas (ver ejercicio 8) y las
propiedades de linealidad de la integral indefinida
Z Z Z Z Z
π
1. 6 dx 2. −7 dx 3. π 3 dx 4. dx 5. −2−5 dx
3
Z Z Z Z Z
√ √
6. 3−2 dx 7. x−2 dx 8. x3/7 dx 9. x dx 10. 3
ω dω
Z √ Z Z Z Z
5 dw dx 7 z
11. x3 dx 12. 13. √ 14. dx 15. dz
w2 n
x xn xn
Z Z Z Z Z
2 6 2 6 x b2 3
16. 5a x dx 17. 5a x da 18. dx 19. db 20. dt
p2 p3 t4
Z Z √ Z Z Z
2 2 b3 a 4
21. dx 22. dt 23. dp 24. dp 25. √ dx
x6 t3 p3 xp2 3
x2
Z Z Z Z
sen x sec2 x sec x tan x
26. dx 27. dx 28. 2 csc2 x dx 29. dx
2 π csc (4π 2 )
Z Z Z Z  
√  1 1
30. (4x − 7) dx 31. (sen 3 + x) dx 32. x − 5x dx 33. − dt
t2 t4
Z √  Z   Z √ √
5 1 √ 1 4 5

34. x2 − 3 dx 35. 3
r+ √ dr 36. t5 + t4 dt
x 3
r
Z  √ Z √ √ Z Z  

4 5

4 5
 5
37. t + t4 dt 38. t7 + t6 dt 39. (3x − 5) dx 40. 2t + 4 dt
t
Z Z Z  π 
√  x π2
41. 3x − x dx 42. (cos x − 2 sen x) dx 43. +√ dx
tan 3 1 − x2
Z √  Z  Z   Z −3
2 π π

2 π3 x dx
44. t − √ dt 45. − 2 dx 46. x + 2 dx 47.
t xr x arcsen t

Última actualización: Julio 2016 Farith J. Briceño N. farith.math@gmail.com


Matemática II - Guía 1. Antiderivada. 14

Z √  Z Z
5 1
48. x2 − √ 5
dx 49. (πx − csc x cot x) dx 50. (4 cos x + 5 sen x) dx
x3
Z Z Z
  
51. y 9 − 2y 5 + 3y dy 52. 2 − 2x − 2x2 dx 53. 3t2 − 2t + 5 dt
Z Z Z  
54. (2x + sec x tan x) dx 55. (3 sen t − 2 cos t) dt 56. x4/3 − 2x1/3 dx
Z Z Z
2
 2
 
57. 5t − 4t + 3 dt 58. 1 − 2x − 3x dx 59. 5y 4 − 5y 2 + 14 dy

Z Z Z  
 5 3
60. (2x + sen x) dx 61. 3t2 − 2 sen t dt 62. sen x + − √ dx
1 + x2 1 − x2
Z Z Z  
  7
63. 2 − 2x − 2x2 dx 64. 3t2 − 2t + 5 dt 65. π π − 8 + 3 cos x dx
2x
Z Z   Z
4 2
 2 1
66. 5t − 6t + 14 dt 67. x + 1 + 2 dx 68. (cos θ + 2 sen θ) dθ
x
Z   Z ! Z
sen (3x) 2 arctan 3 arcsen 2−1
69. − 3 sec t dt 70. √ − dx 71. sen x cos t dx
t4 1 − x2 1 + x2

Z   Z √ ! Z
tan 2 csc t cot t t π √ √
72. π sen x − 5 dx 73. − dt 74. ( y + 3 y − 2) dy
x csc 7 arcsen x 1 + x2

Z Z √ !
2
π t− 2
75. cos t sec x dx 76. sen + dt
x sen (5x)

11. Encuentre, √
en el plano xy, la curva y = f (x) que pasa por el punto (9, 4) y cuya pendiente en cada
punto es 3 x.
12. El punto (3, 2) está en una curva y = f (x) y en cualquier punto (x, y) de la curva, la recta tangente
tiene una pendiente igual a 2x − 3. Encontrar una ecuación de la curva.
13. Encuentre, en el plano xy, la curva y = f (x) que pasa por el punto (1, 2) y cuya pendiente en cada
punto es 4x2 .
14. La pendiente de la recta tangente en cualquier punto (x, y) de una curva y = f (x) es 10 − 4x y el
punto (1, −1) está en la curva. Encontrar una ecuación de la curva.
15. Encuentre, en el plano xy, la curva y = f (x) que pasa por el punto (1, 5) y cuya pendiente en cada
4
punto es − 2 .
x
16. Los puntos (−1, 3) y (0, 2) están en una curva y en cualquier punto (x, y) de la curva, se tiene que
d2 y
= 2 − 4x. Encontrar una ecuación de la curva.
dx2
17. Una ecuación de la recta tangente a una curva en el punto (1, 3) es y = x + 2. Si en cualquier punto
d2 y
(x, y) de la curva se tiene que = 6x, encontrar una ecuación de la curva.
dx2
d2 y
18. En cualquier punto (x, y) de una curva se tiene que = 1 − x2 y una ecuación de la recta tangente
dx2
a la curva en el punto (1, 1) es y = 2 − x. Encontrar una ecuación de la curva.

Última actualización: Julio 2016 Farith J. Briceño N. farith.math@gmail.com


Matemática II - Guía 1. Antiderivada. 15

19. Encuentre una función y = f (x), tal que f cumpla con

d2 y 1 1
= 3/2 − 2 ,
dx2 x x
tenga un punto estacionario en x = 4 y pase por el punto (1, −1).
20. Encuentre una función y = f (x), tal que f cumpla con

d2 y 2 + 3x
= ,
dx2 4x3/2
r !
2 4 2
tenga un mínimo relativo ,− .
3 3 3

d3 y
21. En cualquier punto (x, y) de una curva se tiene = 2 y (1, 3) es un punto de inflexión en el que la
dx3
pendiente de la tangente es −2. Encontrar una ecuación de la curva.
d2 y
22. En cualquier punto (x, y) de una curva, = 4 − x2 y una ecuación de la recta tangente a la curva en
dx2
el punto (1, −1) es 2x − 3y = 3. Encontrar una ecuación de la curva.
d3 y
23. En cualquier punto (x, y) de una curva se tiene = 4x y (−1, 3) es un punto de inflexión en el que
dx3
la pendiente de la tangente es 2. Encontrar una ecuación de la curva.
24. Una partícula se mueve en línea recta, s es la distancia dirigida de la partícula desde el origen en t seg
de tiempo, υ es la velocidad en p/seg de la partícula en t seg y a es la aceleración en p/seg2 de la
partícula en t seg. Si a = 2t − 1 y υ = 3 y s = 4 cuando t = 1, expresar υ y s como funciones de
t.
25. Se lanza una piedra hacia arriba verticalmente desde el suelo con una velocidad inicial de 128 p/seg. Si
la única fuerza considerada es la que se le atribuye a la aceleración de la gravedad, encontrar qué tan alto
llegará la piedra y la velocidad con la que llegará al suelo. Encontrar también cuanto tiempo tomará a la
piedra llegar al suelo.
26. Si el conductor de un automóvil desea aumentar su rapidez de 20 mi/h a 50 mi/h mientras recorre una
distancia de 528 pies. ¿Cuál es la aceleración constante que debe mantener?
27. En los siguientes ejercicios la única fuerza considerada es la debida a la aceleración de la gravedad que
tomamos como 32 p/seg2 en la dirección hacia abajo.
(a) Una piedra se lanza verticalmente hacia arriba desde el suelo, con una velocidad inicial de 20 p/seg.
i. ¿Cuánto tiempo le tomará llegar al suelo y con qué velocidad llegará?
ii. ¿Durante cuánto tiempo estará subiendo la piedra y que tan alto llegará?
(b) Un hombre en un globo suelta sus binoculares cuando se encuentra a 150 pies de altura y está
subiendo a razón de 10 p/seg. ¿Cuánto tiempo tardarán los binoculares en llegar a suelo y cuál es su
velocidad de impacto?
28. Si se aplican los frenos de un carro viajando a 50 mi/h y si los frenos pueden dar al carro una aceleración
negativa constante de 20 p/seg2 . ¿Cuánto tardará el coche en detenerse? ¿Qué distancia recorrerá antes
de parar?
29. Si los frenos de un carro pueden darle una aceleración negativa constante de 20 p/seg2 . ¿Cuál es la
velocidad máxima a que puede ir si es necesario parar el carro dentro de 80 pies después de aplicados los
frenos?

Última actualización: Julio 2016 Farith J. Briceño N. farith.math@gmail.com


Matemática II - Guía 1. Antiderivada. 16

Respuestas: Ejercicios

√ √ √ √
5.1. 1 − x; 5.2. x + 2; 5.3. x − 1; 5.4. − (x + 2) ; 5.5. 1−4 x ; 5.6. 2x + 4; 5.7. x − x + 3;
√ √ √
5.8. − x − x − 1; 6.1. sen x − π; 6.2. 45 − 14 x − 3; 6.3. 5 − x − 3 − sen x + π; 6.4. 5π − 5 sen x;
√ √ √
6.5. 7 x − 3 − 35; 6.6. senex−π
4 ; 6.7. 5 − x − 3 + sen x − π; 6.8. 5 − x − 3 − 2 sen x + 2π; 7.1. f (x) = 1;
2 3 √
7.2. f (x) = m; 7.3. f (x) = x; 7.4. f (x) = x ; 7.5. f (x) = x ; 7.6. f (x) = √1x ; 7.7. f (x) = x;

7.8. f (x) = 3
x; 7.9. f (x) = 1
3x;
√ 7.10. f (x) = − √1 ; 7.11. f (x) = √
3
1
; 7.12. f (x) = xn ;
x3 9 x2

7.13. f (x) = cos x; 7.14. f (x) = sen x; 7.15. f (x) = sec2 x; 7.16. f (x) = sec x tan x; 7.17. f (x) = csc x cot x;
2
7.18. f (x) = csc x; 7.19. f (x) = 2 cos (2x) ; 7.20. f (x) = 4 sen (4x) ; 7.21. f (x) = π sec2 (πx) ; 7.22. y = 3 cos (3x) ;
2 √3
7.23. f (x) = 5 sec (5x) tan (5x) ; 7.24. f (x) = 2 csc (2x) cot (2x) ; 7.25. f (x) = 3 csc (3x) ; 7.26. f (x) = 2 3x
;

7.27. f (x) = a sec (ax) tan (ax) ; 7.28. f (x) = 1


x2 +1
; 7.29. f (t) = sec2 t tan t; 7.30. f (t) = sec2 t tan t;

7.31. f (x) = sec3 x tan x; 2t 1


7.34. f (t) = 9t2 sec2 3t3 tan 3t3 ;
 
7.32. f (t) = t4 +1
; 7.33. f (x) = √ ;
1−x2

7.35. f (t) = t
2 tan (5t) sec2 (5t) ; 7.36. f (x) = − tan x
sec x ; 7.37. f (x) = − csc3 x cot x; 7.38. f (t) = − csc2 t cot t;
1 √ 2 1
7.39. f (x) = ; 7.40. f (x) = √ ; 7.41. f (x) = ; 7.42. f (x) = √ 2x ;
2(1+x) x 1−4x2 3x2/3 x2/3 +1 1−x4

−2t sec−1 x
7.43. f (t) = ; 7.44. f (x) = √1 ; 7.45. f (t) = √ 1
− ; 8.1. x + C; 8.2. mx + C;
t4 +1 π|x| 9x2 +1 2x (x+1)3 (x+1)2

1 2 1 3 1 4 √ 2 3/2 3 4/3
8.3. 2 x + C; 8.4. 3 x + C; 8.5. 4 x + C; 8.6. 2 x + C; 8.7. 3 x + C; 8.8. 4 x + C;
3 2/3 3 8/3 1/5 xn+1
8.9. 2x + C; 8.10. 8x + C; 8.11. 5x + C; 8.12. n+1 + C; 8.13. sen x + C; 8.14. − cos x + C;

8.15. tan x + C; 8.16. sec x + C; 8.17. − csc x + C; 8.18. − cot x + C; 8.19. arctan x + C;

8.20. arcsen x + C; 10.1. 6x + C; 10.2. − 7x + C; 10.3. π 3 x + C; 10.4. 1


3 πx + C; 10.5. − 2−5 x + C;
−2 1 7 10/7 2 3/2 3 4/3 5 8/5
10.6. 3 x + C; 10.7. − x + C; 10.8. 10 x + C; 10.9. 3x + C; 10.10. 4ω + C; 10.11. 8x + C;
1 n (n−1)/n x z2 5 2 7
10.12. − w + C; 10.13. n−1 x + C; 10.14. − 7 (n−1)x n + C; 10.15. 2xn + C; 10.16. 7a x + C;

5 3 6 x2 b3 1 2 2
10.17. 3a x + C; 10.18. 2p2
+ C; 10.19. 3p3
+ C; 10.20. − t3
+ C; 10.21. − 5x5
+ C; 10.22. − 2t2
+ C;

b3 a √
3 1 1
10.23. − 2p2
+ C; 10.24. − px + C; 10.25. 12 x + C; 10.26. − 2 cos x + C; 10.27. π tan x + C;
sec x 2 1 2
10.28. − 2 cot x + C; 10.29. csc(4π 2 )
+ C; 10.30. 2x − 7x + C; 10.31. x sen 3 + 2x + C;

2 3/2 5 2 1 1 1 5 7/5 3 2/3 3 4/3


10.32. 3x − 2x + C; 10.33. 3t3
− t + C; 10.34. 2x2
+ 7x + C; 10.35. 2r + 4r + C;
4 9/4 5 9/5 4 5/4 5 9/5 4 11/4 5 11/5 3 2
10.36. 9t + 9t + C; 10.37. 5t + 9t + C; 10.38. 11 t + 11 t + C; 10.39. 2x − 5x + C;
2 5 3 2 2 3/2 2 xπ+1
10.40. t − 3t3
+ C; 10.41. 2x − 3x + C; 10.42. 2 cos x + sen x + C; 10.43. π arcsen x − (π+1) tan 3
+ C;

2 3/2
√ π 1−r π 1 3 π3 x−2
10.44. 3t − 4 t + C; 10.45. 1−r x − x2 + C; 10.46. 3x − x + C; 10.47. − 2 arcsen t + C;
5 7/5 5 2/5 π 2 3 2 1 6 1 10
10.48. 7x − 2x + C; 10.49. 2x + csc x + C; 10.50. 4 sen x − 5 cos x + C; 10.51. 2y − 3y + 10 y + C;

10.52. 2x − x2 − 2 3
3x + C; 10.53. 5t − t2 + t3 + C; 10.54. x2 + sec x + C; 10.55. − 3 cos t − 2 sen t + C;
3 7/3 3 4/3
10.56. 7x − 2x + C; 10.57. 3t − 2t2 + 5 3
3t + C; 10.58. x − x2 − x3 + C; 10.59. 14y − 5 3
3y + y 5 + C;

10.60. x2 − cos x + C; 10.61. 2 cos t + t3 + C; 10.62. 5 arctan x − cos x − 3 arcsen x + C; 10.63. 2x − x2 − 2 3


3x + C;
2 3 π 1 3 5 1 1 3
10.64. 5t − t + t + C; 10.65. xπ + 2x7
− 3 sen x + C; 10.66. 14t − 2t + t + C; 10.67. x − x + 3x + C;
sen(3x)
arctan 3 arcsen x − arcsen 2−1 arctan x + C;

10.68. sen θ − 2 cos θ + C; 10.69. − 3t3
− 3 tan t + C; 10.70.

tan 2 1 csc t t π+1
10.71. − cos x cos t + C; 10.72. − π cos x + csc 7 4x4 + C; 10.73. − − √ + C;
arcsen x (1+x2 )( π+a)

2 3/2 3 4/3 π 1− 2 √
√t

10.74. 3y − 2y + 4y + C; 10.75. cos t tan x + C; 10.76. sen x + + C; 11. f (x) = 2 x3 − 50;
(− 2) sen(5x)

4x3 +2
12. f (x) = x2 − 3x + 2; 13. f (x) = 3 ; 14. f (x) = 10x − 2x2 − 9; 15. f (x) = 4
x + 1;

16. f (x) = 2
3x + x2 − 2 3
3x + 2; 17. y = x3 − 2x + 4; 18. f (x) = 1 2
2x − 5
3x − 1 4
12 x + 9
4; 19. y = ln x − x;
√ x3
20. y = x3/2 − 2 x; 21. y = 3 − x2 − x + 14
3 ; 22. y = x2 − 4x + 6; 23. y = 1 4
6x − x2 + 2
3x + 9
2;

t3 t2
24. υ = t2 − t + 3 y s= 3 − 2 + 3t + 7
6; 25. Máxima altura : 256 p; Tiempo de vuelo : 8 seg,
77 2
Velocidad de impacto : 128 p/seg; 26. 18 p/seg ;

Última actualización: Julio 2016 Farith J. Briceño N. farith.math@gmail.com


Matemática II - Guía 1. Antiderivada. 17

3 5 25
27.a.i. Tiempo de vuelo : 4 seg, Velocidad de impacto : 20 p/seg; 27.a.ii. Tiempo subiendo : 8 seg, Máxima altura : 4 p;
11 1210
27.b. Tiempo de vuelo : 3.4 seg, Velocidad de impacto : 99 p/seg; 28. Tiempo : 3 seg, Distancia : 9 p;

300 2
29. 11 min/h;

Bibliografía

1. Purcell, E. - Varberg, D. - Rigdon, S.: “Cálculo”. Novena Edición. PEARSON Prentice Hall.
2. Stewart, J.: “Cálculo”. Grupo Editorial Iberoamericano.
3. Thomas, George: “Cálculo de una variable”. 12ma edición. Pearson.
4. Larson - Hostetler - Edwards, “Cálculo”. Vol. 1. Mc Graw Hill.
5. Leithold, Louis, “El cálculo con geometría analítica”. Harla S.A.

Este material ha sido revisado recientemente, pero esto no garantiza que esté libre de errores, por esa razón se
agradece reportar cualquier error que usted encuentre en este material enviando un mensaje al correo electrónico
farith.math@gmail.com
indicando donde se encuentra(n) dicho(s) error(es). MUCHAS GRACIAS.

Última actualización: Julio 2016 Farith J. Briceño N. farith.math@gmail.com


Matemática II - Guía 1. Antiderivada. 18

Última actualización: Julio 2016 Farith J. Briceño N. farith.math@gmail.com


Matemática II - Guía 2
Método de integración: Manipulación algebraica.

Objetivos a cubrir Código : MAT-CI.2


• Método de integración: Manipulación algebraica. Ejercicios resueltos
Z
1/n
Ejemplo 15 : Integre (at) dt.

Solución : Es conocido, por propiedades de potencias que


n
(ab) = an bn ,

entonces

Linealidad de la integral
Sale de la integral por ser constante
respecto a la variable de integración


Z Z z}|{ Z 1 1
t n +1 t n +1 na1/n 1 +1
(at)1/n dt = a1/n t1/n dt = a1/n t1/n dt = a1/n + C = a1/n +C = t n + C.
1 1+n n+1
| {z } + 1
n n

tk+1 1
Z
tk dt = +C con k=
k+1 n

Luego, Z
1/n na1/n 1 +1
(at) dt = t n + C.
n+1

Z
√ √ 
Ejemplo 16 : Integre u u + 3 u du.

Solución : Al aplicar la ley distributiva,

a (b + c) = ab + ac,

se obtiene √ √  √ √
u u + 3 u = u u + u 3 u,
lo cual se escribe como

Propiedades de potencias
an am = an+m

√ √ ↓ 1 1
u u + u 3 u = uu1/2 + uu1/3 = u1+ 2 + u1+ 3 = u3/2 + u4/3 ,
por lo que √ √ 
u u + 3 u = u3/2 + u4/3 .
Al integrar Z Z  Z Z
√ √  
u u + 3 u du = u3/2 + u4/3 du = u3/2 du + u4/3 du

Linealidad de la integral
Z Z Z
(f (x) + g (x)) dx = f (x) dx + g (x) dx

Última actualización: Julio 2016 Farith J. Briceño N. farith.math@gmail.com


Matemática II - Guía 2. Método de integración: Manipulación algebraica. 20

donde Z 3
3/2 u 2 +1 u5/2 2
u du = + C1 = + C1 = u5/2 + C1 ,
3 5 5
| {z } +1
↑ 2 2
un+1 3
Z
un du = +C con n=
n+1 2

mientras que,
Z 4
4/3 u 3 +1 u7/3 3
u du = + C2 = + C2 = u7/3 + C2 ,
4 7 7
| {z } +1
↑ 3 3
un+1 4
Z
un du = +C con n=−
n+1 3

entonces Z
√ √  2 3 2 3
u u + 3 u du = u5/2 + C1 + u7/3 + C2 = u5/2 + u7/3 + (C1 + C2 )
5 7 5 7 | {z }

Constante C

Luego, Z
√ √  2 3
u u + 3 u du = u5/2 + u7/3 + C.
5 7

Z
Ejemplo 17 : Integre tan2 x dx.

Solución : Por identidades trigonométricas, es conocido que

tan2 x + 1 = sec2 x,

por lo que, al despejar tan2 x queda


tan2 x = sec2 x − 1,
así, al integrar Z Z Z Z

tan2 x dx = sec2 x − 1 dx = sec2 x dx − 1 dx

Linealidad de la integral
Z Z Z
(f (x) + g (x)) dx = f (x) dx + g (x) dx

donde Z Z
2 x0+1
sec x dx = tan x + C1 y 1 dx = + C2 = x + C2 ,
0+1
| {z } | {z }
↑ ↑
xn+1
Z
(tan x)′ = sec2 x n
x dx = +C con n=0
n+1

entonces
Z
tan2 x dx = tan x + C1 − (x + C2 ) = tan x + C1 − x − C2 = tan x − x + (C1 − C2 )
| {z }

Constante C

Última actualización: Julio 2016 Farith J. Briceño N. farith.math@gmail.com


Matemática II - Guía 2. Método de integración: Manipulación algebraica. 21

Luego, Z
tan2 x dx = tan x − x + C.


Z
Ejemplo 18 : Integre (ax + b)2 dx.

Solución : Es conocido que


2
(x + y) = x2 + 2xy + y 2 ,
por lo que
(ax + b)2 = (ax)2 + 2 (ax) b + b2 ,
es decir,
(ax + b)2 = a2 x2 + 2abx + b2 .
Al integrar

Linealidad de la integral
Z Z Z
(f (x) + g (x)) dx = f (x) dx + g (x) dx

Z Z ↓ Z Z Z
2 2 2 2
 2 2
(ax + b) dx = a x + 2abx + b 2ab x dx + b2 dx
dx = a x dx + |{z}
↑ ↑ ↑
Linealidad de la integral
Sale de la integral por ser constante
respecto a la variable de integración

Z Z Z
= a2 x2 dx + 2ab x dx + b2 dx,

donde,
Z Z
x2+1 x3 x1+1 x2
x2 dx = + C1 = + C1 , mientras que x dx = + C2 = + C2 ,
2+1 3 1+1 2
| {z } | {z }
↑ ↑
xn+1 xn+1
Z Z
xn dx = +C con n=2 xn dx = +C con n=1
n+1 n+1

y por último Z
x0+1 x
dx = + C3 = + C3 = x + C3 .
0+1 1
| {z }

xn+1
Z
n
x dx = +C con n=0
n+1

entonces Z    
x3 x2
(ax + b)2 dx = a2 + C1 + 2ab + C2 + b2 (x + C3 )
3 2

x3
= a2 + a2 C1 + ab x2 + 2ab C2 + b2 x + b2 C3
3
x3 
= a2 + ab x2 + b2 x + a2 C1 + 2ab C2 + b2 C3
3 | {z }

Constante C

Última actualización: Julio 2016 Farith J. Briceño N. farith.math@gmail.com


Matemática II - Guía 2. Método de integración: Manipulación algebraica. 22

Luego, Z
2 x3
(ax + b) dx = a2 + ab x2 + b2 x + C.
3

Z √
x−x
Ejemplo 19 : Integre √ dx.
3
x
Solución : Es conocido que
a+b a b
= + , con c 6= 0,
c c c
por lo que, √ √
x−x x x
√ = √ − √ ,
3
x 3
x 3
x
puesto que, √ √
x = x1/2 y 3
x = x1/3 ,
se tiene √
x x x1/2 x
√ − √ = 1/3 − 1/3 ,
3
x 3
x x x
por propiedades de potencias, se escribe la expresión anterior como

x1/2 x 1 1 1

1/3
− 1/3 = x 2 − 3 − x1− 3 = x1/6 − x2/3 ,
x x ↑
Propiedades de potencias
an /am = an−m

es decir, √ √
x−x x x
√ = √ − √ = x1/6 − x2/3 .
3
x 3
x 3
x
Al integrar Z √ Z  Z Z
x−x 
1/6 2/3 1/6
√ dx = x −x dx = x dx − x2/3 dx,
3
x

Linealidad de la integral
Z Z Z
(f (x) + g (x)) dx = f (x) dx + g (x) dx

donde,
Z 1
1/6 x 6 +1 x7/6 6
x dx = + C1 = + C1 = x7/6 + C1 ,
1 7 7
| {z } +1
6 6

xn+1 1
Z
n
x dx = +C con n=
n+1 6

mientras que,
Z 2
x 3 +1 x5/3 3
x2/3 dx = + C2 = + C2 = x5/3 + C2 ,
2 5 5
| {z } +1
3 3

xn+1 5
Z
n
x dx = +C con n=−
n+1 3

entonces, Z √  
x−x 6 7/6 3 5/3 6 3
√ dx = x + C1 − x + C2 = x7/6 − x5/3 + C.
3
x 7 5 7 5

Última actualización: Julio 2016 Farith J. Briceño N. farith.math@gmail.com


Matemática II - Guía 2. Método de integración: Manipulación algebraica. 23

Luego, Z √
x−x 6 3
√ dx = x7/6 − x5/3 + C.
3
x 7 5

Z p
4
Ejemplo 20 : Integre 2p3 x dx.

Solución : Por propiedades de radicales

Propiedades de radicales

n √ √ n
ab = n a b

p ↓ p √
4
2p3 x = 4 2p3 4 x,

entonces,
Z p Z p Z Z  
4 3 4 3

4
p
4 3

4
p
4 3 1/4
p
4 3
4 5/4
2p x dx = 2p x dx = 2p x dx = 2p x dx = 2p x + C1 .
| {z } 5
| {z }
↑ ↑
Linealidad de la integral
xn+1 1
Z
Sale de la integral por ser constante xn dx = +C con n=
n+1 4
respecto a la variable de integración

Finalmente, Z p
4 p
4
2p3 x dx = 4
2p3 x5/4 + C,
5
p
donde C = 4
2p3 C1 . ⋆

Z p
4
Ejemplo 21 : Integre 2p3 x dp.

Solución : Por propiedades de radicales

Propiedades de radicales

n √ √ n
ab = n a b

p ↓ √ p
4
2p3 x = 4 2x 4 p3 ,

entonces
Z p Z √ Z p Z  
4 4
p
4

4 4

4 3/4

4 4 7/4
3
2p x dp = |{z}
2x 3
p dp = 2x 3
p dp = 2x p dp = 2x p + C1 .
7
| {z }
↑ ↑
Linealidad de la integral
xn+1 1
Z
Sale de la integral por ser constante xn dx = +C con n=
n+1 4
respecto a la variable de integración

Finalmente Z p
4 √
2x p7/4 + C,
4 4
2p3 x dp =
7

donde C = 4
2x C1 . ⋆

Última actualización: Julio 2016 Farith J. Briceño N. farith.math@gmail.com


Matemática II - Guía 2. Método de integración: Manipulación algebraica. 24

Z
Ejemplo 22 : Integre cos (t − x) dx.

Solución : Es conocida la identidad trigonométrica

cos (t − x) = cos t cos x + sen t sen x,

entonces,

Linealidad de la integral
Z Z Z
(f (x) + g (x)) dx = f (x) dx + g (x) dx

Z Z ↓ Z Z
cos (t − x) dx = (cos t cos x + sen t sen x) dx = cos t
|{z} cos x dx + sen
|{z}t sen x dx
↑ ↑
Linealidad de la integral
Sale de la integral por ser constante
respecto a la variable de integración

Z Z
= cos t cos x dx + sen t sen x dx = − cos t sen x + sen t cos x + C

Finalmente Z
cos (t − x) dx = cos t sen x − sen t cos x + C.


Z
x2 + 2x − 3
Ejemplo 23 : Integre dx.
x−1

x2 + 2x − 3
Solución : Se tiene dos opciones para obtener la familia de primitivas de la función f (x) = , una
x−1
opción es dividir los polinomios y la otra opción es factorizar el polinomio del numerador, por ser un polinomio
de segundo grado usamos la resolvente para a = 1, b = 2 y c = −3, para obtener las raíces del mismo.

q −2 + 4
2 √ √


 x= =1
− (2) ± (2) − 4 (1) (−3) −2 ± 4 + 12 −2 ± 16  2
x= = = =⇒
2 (1) 2 2 
 x = −2 − 4 = −3,


2
luego, la factorización de p es
p (x) = x2 + 2x − 3 = (x − 1) (x + 3) ,
entonces, al integrar

Linealidad de la integral
Z Z Z
(f (x) + g (x)) dx = f (x) dx + g (x) dx

↓|
Z Z Z Z Z Z Z
x2 + 2x − 3 (x − 1) (x + 3)
dx = dx = (x + 3) dx = x dx + 3 dx = x dx + 3 dx,
x−1 x−1
| {z }

Linealidad de la integral
Sale de la integral por ser constante
respecto a la variable de integración

Última actualización: Julio 2016 Farith J. Briceño N. farith.math@gmail.com


Matemática II - Guía 2. Método de integración: Manipulación algebraica. 25

donde
Z Z
x1+1 x2 x0+1 x
x dx = + C1 = + C1 y dx = + C2 = + C2 = x + C2 ,
1+1 2 0+1 1
| {z } | {z }
↑ ↑
xn+1 xn+1
Z Z
xn dx = +C con n=1 xn dx = +C con n=0
n+1 n+1

entonces, Z
x2 + 2x − 3 x2 x2 x2
dx = + C1 + 3 (x + C2 ) = + 3x + C1 + 3C2 = + 3x + C,
x−1 2 2 2
donde C = C1 + 3C2 .

Finalmente Z
x2 + 2x − 3 x2
dx = + 3x + C.
x−1 2

Z
3x2
Ejemplo 24 : Integre dx.
x2 + 1

3x2
Solución : Se tiene dos opciones para obtener la familia de primitivas de la función f (x) = , una
x2 +1
opción es dividir los polinomios y la otra opción es escribir la función f de la siguiente manera

Sumar y restar 3

3x2 ↓ 3x2 + 3 − 3
= ,
x2 + 1 x2 + 1
con lo que,
Factor común 3


z }| {  
3x2 3x2 + 3 −3 3 x2 + 1 − 3 3 x2 + 1 3 3
= = = − 2 =3− 2 ,
x2 + 1 x2 + 1 x2 + 1 ↑ x2+1 x + 1 x +1
Propiedades de los racionales
a+b a b
= +
c c c

es decir,
3x2 3
2
=3− 2 .
x +1 x +1
Al integrar

Linealidad de la integral
Sale de la integral por ser constante
respecto a la variable de integración

Z Z   Z ↓ Z ↓ Z Z
3x2 3 3 1
dx = 3− 2 dx = 3 dx − dx = 3 dx − 3 dx,
x2 + 1 x +1 ↑ x2 +1 x2+1

Linealidad de la integral
Z Z Z
(f (x) + g (x)) dx = f (x) dx + g (x) dx

Última actualización: Julio 2016 Farith J. Briceño N. farith.math@gmail.com


Matemática II - Guía 2. Método de integración: Manipulación algebraica. 26

donde
Z Z
x0+1 x dx
dx = + C1 = + C1 = x + C1 y = arctan x + C2 ,
0+1 1 x2 +1
| {z } | {z }
↑ ↑
xn+1 1
Z
xn dx = +C con n=0 (arctan x)′ =
n+1 x2 + 1

entonces, Z
3x2
dx = 3 (x + C1 ) − 3 (arctan x + C2 ) = 3x − 3 arctan x + C.
x2 +1
Luego, Z
3x2
dx = 3x − 3 arctan x + C.
x2 + 1

Z
sen3 x + 2 sen2 x − sen x − 2
Ejemplo 25 : Integre dx.
cos2 x
Solución : Es conocido que
cos2 x = 1 − sen2 x
así, la integral se puede escribir como
Z Z
sen3 x + 2 sen2 x − sen x − 2 sen3 x + 2 sen2 x − sen x − 2
2
dx = dx
cos x 1 − sen2 x
Observemos que la expresión del numerador se puede factorizar como

Factor común sen x Factor común −1

↓ ↓
z }| { z }| { 
sen x + 2 sen x + − sen x − 2 = sen2 x (sen x + 2) − (sen x + 2) = (sen x + 2) sen2 x − 1 ,
3 2
| {z }

Factor común sen x + 2

mientras que, en el término del denominador podemos sacar −1 como factor común y nos queda

1 − sen2 x = − sen2 x − 1
la integral se escribe
Z Z  Z
sen3 x + 2 sen2 x − sen x − 2 (sen x + 2) sen2 x − 1
dx = dx = − (sen x + 2) dx
1 − sen2 x − (sen2 x − 1)
Z Z
= − sen x dx − 2 dx = cos x − 2x + C.

Linealidad de la integral
Z Z Z
(f (x) + g (x)) dx = f (x) dx + g (x) dx

Finalmente Z
sen3 x + 2 sen2 x − sen x − 2
dx = cos x − 2x + C.
cos2 x

Última actualización: Julio 2016 Farith J. Briceño N. farith.math@gmail.com


Matemática II - Guía 2. Método de integración: Manipulación algebraica. 27

Z 2
(x − 1) dx
Ejemplo 26 : Integre √
4 √ .
x3 ( x − 1)

√ : Aplicamos la conjugada de la expresión
Solución x − 1, es decir, multiplicamos y dividimos por el
término x + 1
Z 2 Z 2 √ Z 2 √
(x − 1) dx (x − 1) ( x + 1) (x − 1) ( x + 1)
√ √ = √ √ √ dx = √  √ 2  dx
x3 ( x − 1) ( x + 1)
4 4
x3 ( x − 1) x3 ( x) − (1)2
4

Z 2 √ Z √
(x − 1) ( x + 1) (x − 1) ( x + 1)
= √
4
dx = √4
dx,
x3 (x − 1) x3
desarrollamos el término del numerador
√  √ √
(x − 1) x + 1 = x x + x − x − 1 = x3/2 + x − x1/2 − 1,

entonces, Z √ Z 3/2
(x − 1) ( x + 1) x + x − x1/2 − 1

4
dx = √4
dx
x3 x3
Z  3/2 
x x x1/2 1
= + − − dx
x3/4 x3/4 x3/4 x3/4
Z  
= x3/2−3/4 + x1−3/4 − x1/2−3/4 − x−3/4 dx
Z Z Z Z
= x3/4 dx + x1/4 dx − x−1/4 dx − x−3/4 dx

4 7/4 4 5/4 4 3/4


= x + x − x − 4x1/4 + C
7 5 3
Finalmente Z 2
(x − 1) dx 4 4 4
√ √ = x7/4 + x5/4 − x3/4 − 4x1/4 + C.
4 3
x ( x − 1) 7 5 3

Z
x2 − 16
Ejemplo 27 : Integre √ dx.
2− x

Solución √: Aplicamos la conjugada de la expresión 2 − x, es decir, multiplicamos y dividimos por el
término 2 + x
Z 2 Z  √ Z  √ Z  √
x − 16 x2 − 16 (2 + x) x2 − 16 (2 + x) x2 − 16 (2 + x)
√ dx = √ √ dx =  √ 2  dx = dx
2− x (2 − x) (2 + x) 2
(2) − ( x) 4−x

Observemos que el polinomio del numerador se puede factorizar como

x2 − 16 = (x − 4) (x + 4) = − (4 − x) (x + 4) ,

así,  √ √
Z Z Z
x2 − 16 (2 + x) − (4 − x) (x + 4) (2 + x) √ 
dx = dx = − (x + 4) 2 + x dx,
4−x 4−x
desarrollando esta expresión
√  √ √
(x + 4) 2 + x = 2x + x x + 8 + 4 x = 2x + x3/2 + 8 + 4x1/2

Última actualización: Julio 2016 Farith J. Briceño N. farith.math@gmail.com


Matemática II - Guía 2. Método de integración: Manipulación algebraica. 28

la integral nos queda


Z Z 
√  
(x + 4) 2 + x dx = 2x + x3/2 + 8 + 4x1/2 dx
Z Z Z Z
2 5/2 8
= 2x dx + x3/2 dx + 8 dx + 4x1/2 dx = x2 + x + 8x + x3/2 + C
5 3

Finalmente, Z  
x2 − 16 2 8
√ dx = − x2 + x5/2 + 8x + x3/2 + C.
2− x 5 3

Z
sen2 x dx
Ejemplo 28 : Integre .
cos2 (x/2)
Solución : Es conocido que
sen 2 (·) = 2 sen (·) cos (·) , (1)
por otro lado, x
sen x = sen 2
2
por la ecuación (1) se tiene
x x  x  x 2 x x
sen x = 2 sen cos =⇒ sen2 x = 2 sen cos = 4 sen2 cos2
2 2 2 2 2 2
así,    
Z 2 Z 4 sen2 x cos2 x Z  
sen x dx 2 2 dx = 4 sen2 x dx
2
= 2
cos (x/2) cos (x/2) 2
como
1 − cos 2 (·)
sen2 (·) = ,
2
entonces, x
x 1 − cos 2 1 − cos x
sen2 = 2 = ,
2 2 2
esto implica
Z x Z Z Z Z 
1 − cos x
4 sen2 dx = 4 dx = 2 (1 − cos x) dx = 2 dx − cos x dx = 2x − 2 sen x + C
2 2

Finalmente Z
sen2 x dx
= 2x − 2 sen x + C.
cos2 (x/2)

Z
cos2 (arcsen x)
Ejemplo 29 : Integre dx.
x5
Solución : Es conocido que

cos2 (·) = 1 − sen2 (·) y sen (arcsen x) = x,

entonces
2
cos2 (arcsen x) = 1 − sen2 (arcsen x) = 1 − (sen (arcsen x)) = 1 − x2 ,

Última actualización: Julio 2016 Farith J. Briceño N. farith.math@gmail.com


Matemática II - Guía 2. Método de integración: Manipulación algebraica. 29

por lo tanto,
Z Z Z  
cos2 (arcsen x) 1 − x2 1 x2
dx = dx = − dx
x5 x5 x5 x5
Z Z Z Z
1 x2 −5 1 1
= dx − dx = x dx − x−3 dx = − + 2 + C,
x5 x5 4x 4 2x
es decir, Z
cos2 (arcsen x) 1 1
dx = − 4 + 2 + C.
x5 4x 2x

Z √ 3 √
x −x−5 x+2
Ejemplo 30 : Integre √ dx.
x−3 x+1
√ √ 3 √ 2
Solución : Es conocido que x3 = ( x) y que x = ( x) , así,
√ √ √ 3 √ 2 √
x3 − x − 5 x + 2 ( x) − ( x) − 5 x + 2
√ = √ 2 √
x−3 x+1 ( x) − 3 x + 1
Se observa que la expresión del numerador se factoriza como
√ 3 √ 2 √ √   √ 2 √ 
x − x −5 x+2= x+2 x −3 x+1 ,

así, el integrando queda


√ √ √ √
2 √ 
3
x −x−5 x+2 ( x + 2) ( x) − 3 x + 1 √
√ = √ 2 √ = x+2
x−3 x+1 ( x) − 3 x + 1
de aquí,
Z √ 3 √ Z
x −x−5 x+2 √  2
√ dx = x + 2 dx = x3/2 + 2x + C.
x−3 x+1 3
Luego,
Z √ 3 √
x −x−5 x+2 2
√ dx = x3/2 + 2x + C.
x−3 x+1 3

Z
cos (2x) dx
Ejemplo 31 : Integre .
cos x − sen x
Solución : Es conocida la identidad trigonométrica

cos (x + y) = cos x cos y − sen x sen y,

por lo tanto,
cos (2x) = cos (x + x) = cos x cos x − sen x sen x = cos2 x − sen2 x.
Por otra parte, también es conocido que

a2 − b2 = (a − b) (a + b) ,

así,
cos2 x − sen2 x = (cos x − sen x) (cos x + sen x) ,
entonces,
cos (2x) = (cos x − sen x) (cos x + sen x) .

Última actualización: Julio 2016 Farith J. Briceño N. farith.math@gmail.com


Matemática II - Guía 2. Método de integración: Manipulación algebraica. 30

Al integrar
Z Z Z
cos (2x) dx (cos x − sen x) (cos x + sen x)
= dx = (cos x + sen x) dx
cos x − sen x cos x − sen x
Z Z
= cos x dx + sen x dx = sen x − cos x + C.

Luego, Z
cos (2x) dx
= sen x − cos x + C.
cos x − sen x

Z
1 − x2
Ejemplo 32 : Integre dx.
1 − x−1
Solución : Por propiedad de potencias, se tiene que
1 1 x−1
x−1 = , de aquí, 1 − x−1 = 1 − = ,
x x x
entonces,

1 − x2 1 − x2 x 1 − x2 x (1 − x) (1 + x) −x (x − 1) (1 + x)
= = = = = − x (1 + x) ,
1 − x−1 x−1 x−1 x−1 x−1
x
es decir,
1 − x2
= − x (1 + x) .
1 − x−1
Al integrar
Z Z Z  
1 − x2 2
 x2 x3
dx = − x (1 + x) dx = − x+x dx = − + + C.
1 − x−1 2 3

Luego, Z
1 − x2 x2 x3
dx = − − + C.
1 − x−1 2 3

Z √
1 − x2 dx
Ejemplo 33 : Integre .
1 − x2
Solución : Al racionalizar se obtiene
Z √ Z √ √ Z Z
1 − x2 dx 1 − x2 1 − x2 1 − x2 1
= √ dx = √ dx = √ dx = arcsen x + C.
1 − x2 1 − x2 1 − x2 (1 − x2 ) 1 − x2 1 − x2
Luego,
Z √
1 − x2 dx
= arcsen x + C.
1 − x2

Z
sen2 (x/2) − sen4 (x/2)
Ejemplo 34 : Integre dx.
1 + cos x
Solución : Se tiene que
x x x   x  x x  x  x 2
sen2 − sen4 = sen2 1 − sen2 = sen2 cos2 = sen cos
2 2 2 2 2 2 2 2

Última actualización: Julio 2016 Farith J. Briceño N. farith.math@gmail.com


Matemática II - Guía 2. Método de integración: Manipulación algebraica. 31

Es conocido que

sen 2 (·)
sen 2 (·) = 2 sen (·) cos (·) , es decir, = sen (·) cos (·) ,
2
por lo tanto,
x x 1 x 1
sen cos = sen 2 = sen x
2 2 2 2 2
así,
x x  2 x x
1 1
sen2 − sen4 = sen x =⇒ sen2 − sen4 = sen2 x,
2 2 2 2 2 4
por lo que, la integral se transforma en
1
Z
sen2 (x/2) − sen4 (x/2)
Z sen2 x 1
Z
sen2 x
dx = 4 dx = dx
1 + cos x 1 + cos x 4 1 + cos x

sen2 x
Para la familia de primitivas de la función f (x) = , por la identidad trigonométrica básica
1 + cos x

sen2 (·) + cos2 (·) = 1,

se obtiene que
sen2 x = 1 − cos2 x
lo cual se escribe como
sen2 x = (1 − cos x) (1 + cos x) ,
por lo tanto,
(1 − cos x) (1 + cos x)
f (x) = = 1 − cos x,
1 + cos x
y la integral queda
Z Z Z Z
sen2 x
dx = (1 − cos x) dx = dx − cos x dx = x − sen x + C1 ,
1 + cos x
entonces
Z Z
sen2 (x/2) − sen4 (x/2) 1 sen2 x 1 x sen x
dx = dx = (x − sen x + C1 ) = − + C.
1 + cos x 4 1 + cos x 4 4 4
Luego, Z
sen2 (x/2) − sen4 (x/2) x sen x
dx = − + C.
1 + cos x 4 4

Z  x π 
Ejemplo 35 : Integre cos2 − sen − x dx.
2 4
Solución : Se tiene que
Z    π  Z   Z π 
2 x 2 x
cos − sen − x dx = cos dx − sen − x dx,
2 4 2 4
Z x
Para resolver cos2 dx, se procede de la siguiente manera, es conocida la identidad trigonométrica
2

1 + cos 2 (·)
cos2 (·) = ,
2

Última actualización: Julio 2016 Farith J. Briceño N. farith.math@gmail.com


Matemática II - Guía 2. Método de integración: Manipulación algebraica. 32

por lo que, x


x 1 + cos 2 1 + cos x
cos2 = 2 = ,
2 2 2
así,
x 1 + cos x
cos2 = .
2 2
Al integrar
Z x Z Z
2 1 + cos x 1 1
cos dx = dx = (1 + cos x) dx = (x + sen x) + C1 .
2 2 2 2
Luego, Z x 1
cos2 dx = (x + sen x) + C1 .
2 2
Z π 
Por otra parte, para resolver sen − x dx, se procede de la siguiente manera, es conocida la identidad
4
trigonométrica
sen (x − y) = sen x cos y − cos x sen y,
así,  π 
π π
− x = sen
sen cos x − cos sen x,
4 4 4
π π √
2
puesto que, sen = cos = , se concluye que
4 4 2
π  √2 √
2 π  √2
sen −x = cos x − sen x =⇒ sen −x = (cos x − sen x) .
4 2 2 4 2
Al integrar
Z π  Z √ √ Z √
2 2 2
sen − x dx = (cos x − sen x) dx = (cos x − sen x) dx = (sen x + cos x) + C2 .
4 2 2 2
Luego, √
Z 
π 2
sen − x dx = (sen x + cos x) + C2 .
4 2
Entonces Z  √
x  π
1 2
cos2 − sen
− x dx = (x + sen x) − (sen x + cos x) + C
2 4 2 2
√ √
x 1− 2 2
= + sen x − cos x + C.
2 2 2
Finalmente Z  √ √
  π 
2 x x 1− 2 2
cos − sen − x dx = + sen x − cos x + C.
2 4 2 2 2

Ejercicios

Calcular las siguientes integrales por manipulación algebraica


Z Z Z 6 Z Z
√ √ √
3
 t − t2 2 2
1. x x dx 2. u u + u du 3. dt 4. (x + 4) dx 5. (3 − 2t) dt
t4
Z Z Z Z Z √
3
2 2 2 2
2 t
6. (ax + b) dx 7. (a + bt) dt 8. (a − bt) da 9. x −1 dx 10. dt
t

Última actualización: Julio 2016 Farith J. Briceño N. farith.math@gmail.com


Matemática II - Guía 2. Método de integración: Manipulación algebraica. 33

Z Z Z   Z
2  √ 1 2
11. x3 − x dx 12. y 2 y 2 − 3 dy 13. x x2 − dx 14. y 2 + 4y dy
x
Z Z Z Z  2

3 2

3 2 3 3
15. a + bt dt 16. a + bt da 17. (x + 1) dx 18. x− dx
x
Z Z Z Z
3 3 3 4
19. (x + 4) dx 20. (3 − 2x) dx 21. (a + bt) dt 22. (a − bt) dt
Z Z Z Z  3
2 2 2 1
23. y 2 + 4y −2 dy 24. a + bt3 dt 25. a + bt3 da 26. x+ dx
x
Z Z √ Z √ Z 4
x3 − 1 x−x 3
x − x2 − π s −8
27. √ dx √ 28.dx 29. 2
dx 30. ds
x 3
x x s2
Z Z   Z
t2 + 1 t2 − 2 √ √
31. (x + 5) (2x − 3) dx 32. √
3 2
dt 33. y ( y + 3 y − 2) dy
t
Z Z Z
√  √ 
34. (x − 1) (3x + 2) dx 35. x + 1 x − x + 1 dx 36. x (x + a) (x + b) dx

Z Z Z Z
1/n 1−n x4 − 1 
37. (at) dt 38. (nx) n
dx 39. dx 40. sen2 t + cos2 t dt
x2 + 1
Z Z Z Z
41. cos (t − x) dx 42. cos (t − x) dt 43. sen (t − x) dt 44. sen (t − x) dx
Z Z Z √3 Z
x2 + 1 x−1 x + x4 4x6 + 3x5 − 8
45. √ dx 46. √ dx 47. √ dx 48. dx
x 3
x2 x x5
Z p Z p Z Z 3
4 4 x4 − 2x2 + 1 x − 3x2 + 1
49. 2p3 x dx 50. 2p3 x dp 51. dx 52. √ dx
x2 x
Z Z Z  2 Z 
2 2 1 √ 2
53. x2 + 1 dx 54. x3 − 1 x+dx dx55. 56. 2 − t dt
x
Z  3 Z Z 2 Z
√ 1 2
3 x −4 1−x
57. t− √ dt 58. x x + 1 dx 59. dx 60. dx
5 2
t x−2 1 − x−1
Z Z Z 2 Z 2
1 − x2 1 − x−2 (xm + xn ) (xm − xn )
61. dx 62. dx 63. dx 64. √ dx
1 − x−1 1 − x−4 x2 x
Z Z 4 Z 3 Z
x4 − 2x2 + 1 x − 2x2 + 1 t −1
65. dx 66. dx 67. dt 68. tan2 t dt
x+1 x−1 t−1
Z Z Z π  Z π 

69. cot2 x dx 70. cot2 x + 1 dx 71. cos + x dx 72. sen − x dx
4 3
Z Z Z Z
x3 − 8 x3 + 27 x4 + x2 + 3 x3 − x2 + x − 3
73. dx 74. dx 75. 2
dx 76. dx
x−2 x+3 x +1 x2 + 1
Z Z Z
x4 + x3 − 2x2 + 2x − 8 sec2 x − tan2 x 1 − cos2 x − sen x
77. 2
dx 78. √ dx 79. dx
x +2 1−x 2 1 − sen x
Z Z Z√ 3 Z √4
x6 − 8 cot x ( 3 x − 1) x3 − 5
80. dx 81. dx 82. √ dx 83. √ √ dx
x2 − 2 tan x x 4
x− 35
Z Z Z   Z
sen (2t) cos2 x 2 t cos (2x) dx
84. dt 85. dx 86. cos dt 87. √
sen t 1 + sen x 2 1 − 2 cos x

Última actualización: Julio 2016 Farith J. Briceño N. farith.math@gmail.com


Matemática II - Guía 2. Método de integración: Manipulación algebraica. 34

Z  Z Z
x π  cos (2t) 5x + 8x2 − 3x3 − 6
88. cos2 − sen − x dx 89. 2
dt 90. dx
2 4 sen t x5 − 3x4
Z Z √ 3 √ Z √ 5
2 sen2 x + 5 sen x − 3 5 x2 − 3x 3 x + 2 x3 (x − 1)
91. dx 92. √ √ dx 93. √ dx
sen x + 3 2− 3x x−1
Z 2 Z Z
(x − 1) dx cos2 (arcsen x) cos2 (arcsen x) − 7
94. √ √ 95. dx 96. dx
4
x3 ( x − 1) x5 x5
Z Z Z
cos2 (arcsen x) + 2x4 cos4 (arcsen x) + π 2 sec4 (arctan x)
97. dx 98. dx 99. dx
x6 2x6 sen2 (arctan x2 )
Z √5 Z Z √ 4
x3 (x − 1) x2 − 2x + 1 x − 3x3 + 3x2 − x
100. √ dx 101. √ √ dx 102. √ dx
3
x−1 4
x3 ( 3 x − 1) x−1
Z Z Z Z
cos (2t) cos (2x) dx cos (2t) dt sen2 x
103. 2
dt 104. 105. 106. dx
cos t cos x − sen x cos t + sen t cos2 (x/2)
Z 2 Z Z
x − 16 dx 2 sec4 x + 3 sec2 x − 2
107. √ dx 108. 4 4 2
109. dx
2− x sen x − cos x − sen x sec2 x + 2
Z Z Z   Z
3 cos2 x + 5 cos x − 2 3 dx 2 t tan x
110. dx 111. 112. sen dt 113. dx
cos x + 2 sec2 (x/2) 2 cot x
Z    cos2 x  Z Z
4 x 2 tan4 x + 7 sec2 x − 22 cos (2x) csc2 x
114. sen − dx 115. dx 116. dx
2 4 sec2 x + 4 sen4 x − cos4 x
Z Z Z
sen3 x + 2 sen2 x − sen x − 2 sen (4x) x−8
117. 2
dx 118. dx 119. √ dx
cos x cos (2x) cos x 3
x−2
Z Z Z √ 3 √
x3 tan x dx sen2 (x/2) − sen4 (x/2) x −x−5 x+2
120. √ 121. dx 122. √ dx
x sec2 x − x cos2 (x/2) x−3 x+1
Z Z √ √ ! Z √ √ 2
csc x tan x x sec2 x − x x4 − 2x2 + 1 ( a − x)
123. dx 124. − dx 125. √ dx
sec x x3 tan x dx 1 − x4 ax
Z Z Z
x2 dx sen2 (x/2) − sen4 (x/2) 2 csc2 x − 3 csc x − csc3 x + 6
126. 127. dx 128. dx
x2 + 1 1 + cos x csc x − 2
Z Z Z Z
tan a sen x dx π dx
129. √ dx 130. 2
dx 131. 2 2
132.
1−x 2 cos x sen x cos x csc2 (x/2)
Z √ Z √ Z Z
x−1 x−1 3x2 dx 2 − x2
133. √ dx 134. √ dx 135. 136. dx
4
x−1 x2 ( 4 x − 1) x2 + 1 x2 + 1
Z Z Z Z
5 + x2 1 − 3x2 2 + 5x2 sen3 x − cos3 x
137. 2
dx 138. 2
dx 139. 2
dx 140. dx
x +1 x +1 x +1 2 + sen (2x)
Z Z Z 2 √ √
cos (2x) cos (2x) sen x x + 4x + 3 − 2 x3 − 2 x
141. dx 142. dx 143. √ √ dx
cos4 x − sen4 x sen4 x − cos4 x x3 − 2x + 3 x
Z √ Z
1 − x2 sen (arcsen x) + x2
144. dx 145. dx
1 − x2 arctan (tan (x4 ))
Respuestas: Ejercicios

5
2 5/2 3 7/3 1 3 1 3
1. 2 2
5x + C; 2. 5u + 7u + C; 3. 3t + 1
t + C; 4. 3x + 4x2 + 16x + C; 5. 9t − 6t2 + 4 3
3t + C;
1 2 3 2 2 2 2 1 2 3 1 3 2 2 2 1 5 2 3
6. 3a x + abx + b x + C; 7. a t + abt + 3b t + C; 8. 3a − a bt + ab t + C; 9. 5x − 3x + x + C;

3 1 7 2 5 1 3 1 5 √
10. 3 t + C; 11. 7x − 5x + 3x + C; 12. 5y − y 3 + C; 13. 72 x7/2 − 2 x + C; 14. 1 5
5y + 2y 4 + 16 3
3 y + C;

Última actualización: Julio 2016 Farith J. Briceño N. farith.math@gmail.com


Matemática II - Guía 2. Método de integración: Manipulación algebraica. 35

15. a2 t + 1
2 abt
4
+ 1 2 7
7b t + C; 16. 1 3
3a + a2 bt3 + ab2 t6 + C; 17. 1 4
4x + x3 + 3 2
2x + x + C; 18. 1 3
3x − 6x − 9
x + C;
1 4
19. 4x + 4x3 + 24x2 + 64x + C; 20. 27x − 27x2 + 12x3 − 2x4 + C; 21. a3 t + 1 3 4
4b t + 3 2 2
2 a bt + ab2 t3 + C;

22. a4 t + 1 4 5
5b t − 2a3 bt2 − ab3 t4 + 2a2 b2 t3 + C; 23. 1 5
5y + 8y − 16
3y3
+ C; 24. a2 t + 1
2 abt
4
+ 1 2 7
7b t + C;

1 3 2 3 2 6 1 4 3 2 1 2 7/2 √ 6 7/6 3 5/3


25. 3a + a bt + ab t + C; 26. 4x + 2x + 3 ln |x| − 2x2
+ C; 27. 7x − 2 x + C; 28. 7x − 5x + C;
3 π 1 3 8 2 3 7 2 3 13/4 3 7/3

3
29. − −x+ x + C; 30. 3s + s + C; 31. 3x + 2x − 15x + C; 32. 13 t − 7t −6 t + C;
2x2/3
2 5/2 3 7/3
33. 5y + 7y − y 2 + C; 34. x3 − 1 2
2x − 2x + C; 35. 2 5/2
5x + x + C; 36. 1 4
4x + 1
3 ax
3
+ 1
3 bx
3
+ 1
2 abx
2
+ C;
1
37. n
n+1 a1/n t n +1 + C; 38. (nx)1/n + C; 39. 1 3
3x − x + C; 40. t + C; 41. cos t sen x − sen t cos x + C;

42. sen t cos x − cos t sen x + C; 43. − cos t cos x − sen t sen x + C; 44. sen t sen x + cos t cos x + C;
2 5/2 √ √ 2 3/2 11/6
3 3 6
48. 3x + 2x2 + x24 + C;

45. 5 x + 2 x + C; 46. x 4 x − 3 + C; 47. 3 x + 11 x + C;
√ √
49. 45 4 2p3 x5/4 + C; 50. 47 4 2xp7/4 + C; 51. 13 x3 − x 52. 72 x7/2 − 65 x5/2 + 2 x + C;
p 1
− 2x + C;
1 5 2 3 1 7 1 4 1 3 1 1 2 8 3/2
53. 5x + 3x + x + C; 54. 7x − 2x + x + C; 55. 3x + 2x − x + C; 56. 4t + 2t − 3t + C;
2 5/2 15 8/5 30 7/10 5 1 8 1 6 3 4 1 2 1 2 x2
57. 5t − 8 t + 7 t + √
5t + C; 58. 8x + 2x + 4x + 2x + C; 59. 2x + 2x + C; 60. − 2 + C;

x2 x3 1 2m−1 2 m+n−1 1 2n−1


61. − 2 − 3 + C; 62. x − arctan x + C; 63. 2m−1 x + m+n−1 x + 2n−1 x + C;

2 2m+1/2 4 m+n+1/2 2 2n+1/2 x4 x3 x2 x4 x3 x2


64. 4m+1 x − 2m+2n+1 x + 4n+1 x + C; 65. 4 − 3 − 2 + x + C; 66. 4 + 3 − 2 − x + C;
√ √
t3 t2 2 2
67. 3 + 2 + t + C; 68. tan t − t + C; 69. − cot x − x + C; 70. − cot x + C; 71. 2 sen x + 2 cos x + C;

1 3
72. 3 1
2 sen x + 2 cos x + C; 73. 3x + x2 + 4x + C; 74. 1 3
3x − 3 2
2x + 9x + C; 75. 1 3
3x + 3 arctan x + C;
x2 x2 1 3 2 3 x5
76. 2 − x − 2 arctan x + C; 77. 2 − 4x + 3x + C; 78. arcsen x + C; 79. cos x + C; 80. 4x + 3x + 5 + C;
18 5/6 18 7/6 √ √
3

3
81. − cot x − x + C; 82. 5 x − 7 x + 32 x3/2 − 2 x + C; 83. 2 3/2
3x + 25 x + 4 5 5 x5/4 + C; 84. 2 sen t + C;
√ √ √
85. x + cos x + C; 86. 1
2t + 1
2 sen t + C; 87. − x − 2 sen x + C; 88. 1
2x + 1−2 2 sen x − 22 cos x + C;
3 1 2 3 2 3 4/3
√ √
9 2 5/3
89. − cot t − 2t + C; 90. x + 2x2
− 3x3
+ C; 91. − 2 cos x − x + C; 92. 2x + 4x + 2x + 5 x + C;
10 21/10 5 8/5 4 7/4 4 5/4 4 3/4 √
4 1 1 1 3
93. 21 x + 8x + C; 94. 7x − 5x − 3x +4 x + C; 95. 2x2
− 4x4
+ C; 96. 2x2
+ 2x4
+ C;

1 1 2 1 1 e2 1 1 2 3 1 5 2 1
97. 2x2 3x3
− x − 5x5
+ C; 98. 3x3
− 10x5
− 2x − 10x5
+ C; 99. 2x + 3x + 5x − x − 3x3
+ C;
15 34/15 15 29/15 5 8/5 4 5/4 √ 12 7/12 12 11/12 12 19/12 12 23/12
100. 34 x + 29 x + 8x + C; 101. 5x −44x− 7 x − 11 x + 19 x + 23 x + C;
2 5/2 2 3/2
102. 5x − 3x + C; 103. 2t − tan t + C; 104. sen x − cos x + C; 105. cos t + sen t + C;
2 5/2 8 3/2
106. 2x − 2 sen x + C; 107. − 5x + 3x − x2 + 8x + C; 108. − tan x + C; 109. 2 tan x − x + C;
3x 3 t 1 x 1
110. 3 sen x − x + C; 111. 2 + 2 sen x + C; 112. 2 − 2 sen t + C; 113. tan x − x + C; 114. 4 − 2 sen x + C;
3 5/3 3 4/3
115. 2 tan x − 5x + C; 116. cot x + C; 117. cos x − 2x + C; 118. − 4 cos x + C; 119. 5x + 2x + 4x + C;
2 7/2 1 1 2 3/2 2
120. 7x + C; 121. 2x − 2 sen x + C; 122. 3x + 2x + C; 123. x + C; 124. − − arctan x + C;
3x3/2

125. 2 ax − 2x + 2

3 a
x3/2 + C; 126. x − arctan x + C; 127. 1
4x − 1
4 sen x + C; 128. 3x − cot x + C;
π π 4 5/4
129. tan a arcsen x + C; 130. sec x + C; 131. tan x − cot x + C; 132. 2x − 2 sen x + C; 133. 5x + x + C;
1 4
134. − x − + C; 135. 3x − 3 arctan x + C; 136. 3 arctan x − x + C; 137. x + 4 arctan x + C;
3x3/4
1 1
138. 4 arctan x − 3x + C; 139. 5x − 3 arctan x + C; 140. − 2 cos x − 2 sen x + C; 141. x + C; 142. cos x + C;

143. 2 x + 23 x3/2 + C; 144. arcsen x + C; 145. − 1
x − 1
2x2
+ C;
Bibliografía

1. Purcell, E. - Varberg, D. - Rigdon, S.: “Cálculo”. Novena Edición. PEARSON Prentice Hall.
2. Stewart, J.: “Cálculo”. Grupo Editorial Iberoamericano.

Este material ha sido revisado recientemente, pero esto no garantiza que esté libre de errores, por esa razón se
agradece reportar cualquier error que usted encuentre en este material enviando un mensaje al correo electrónico
farith.math@gmail.com
indicando donde se encuentra(n) dicho(s) error(es). MUCHAS GRACIAS.

Última actualización: Julio 2016 Farith J. Briceño N. farith.math@gmail.com


Matemática II - Guía 2. Método de integración: Manipulación algebraica. 36

Última actualización: Julio 2016 Farith J. Briceño N. farith.math@gmail.com


Matemática II - Guía 3
Método de integración: u-sustitución.

Objetivos a cubrir Código : MAT-CI.3


• Método de integración: u-sustitución. Ejercicios resueltos
Z
2
Ejemplo 36 : Integre (ax + b) dx.

Solución : En el ejemplo 18 se aplica manipulación algebraica para obtener la familia de primitivas de la


2
función f (x) = (ax + b) , más precisamente, se desarrolla el producto notable y por linealidad de la integral
indefinida se obtiene la familia de primitivas (ver Ejemplo 18). A continuación se resuelve la integral por medio
de un cambio de variable.

Se propone el cambio de variable


Calculo del du
u = ax + b −−−−−−−−−→ du = a dx =⇒ dx = ,
diferencial a

entonces, la integral queda

Diferencial
Cambio Integral de una potencia.
du
u = ax + b dx = Integral de tabla.
a
↓  ↓
Z z }| zZ }| {
{ 2 Z
1 u3
2 du 1 1
ax + b dx = u = u2 du = +C = (ax + b)3 + C.
a a a 3 3a
↑ | {z }
Linealidad de la integral
un+1
Z
Sale de la integral por ser constante un du = +C con n=2
n+1
respecto a la variable de integración

Luego, Z
1
(ax + b)2 dx = (ax + b)3 + C.
3a

Compare el resultado obtenido en este ejemplo, usando el método de u−sustitución con el resultado que se
obtiene en el Ejemplo 18, usando manipulación algebraica. Según su opinión
¿Cuál método le parece más sencillo?
¿Cuál método le parece más natural aplicar?


Z
Ejemplo 37 : Integre (at)1/n dt.

Solución : En el ejemplo 15 se aplica manipulación algebraica para obtener la familia de primitivas de la


1/n
función f (t) = (at) , más precisamente, se aplica propiedades de potencias y por linealidad de la integral
indefinida se obtiene la familia de primitivas (ver Ejemplo 15). A continuación se resuelve la integral por medio
de un cambio de variable.

Se propone el cambio de variable


Calculo del du
u = at −−−−−−−−−→ du = a dt =⇒ dt = ,
diferencial a

Última actualización: Julio 2016 Farith J. Briceño N. farith.math@gmail.com


Matemática II - Guía 3. Método de integración: u-sustitución. 38

entonces, la integral queda

Diferencial
Cambio Integral de una potencia.
du
u = at dt = Integral de tabla.
a
↓ 
Z z}|{ ↓ Z zZ }| {
1/n 1
1/n du 1 1 u n +1 1 u(n+1)/n
at dt = u = u1/n du = +C = +C
a a a 1 a n+1
| {z } +1
↑ n n
Linealidad de la integral
um+1 1
Z
Sale de la integral por ser constante um du = +C con m=
m+1 n
respecto a la variable de integración

1 nu(n+1)/n n
= +C = (at)(n+1)/n + C.
a n+1 a (n + 1)
Luego, Z
1/n n (n+1)/n
(at) dt = (at) + C.
a (n + 1)

Compare el resultado obtenido en este ejemplo, usando el método de u−sustitución con el resultado que se
obtiene en el Ejemplo 15, usando manipulación algebraica. Según su opinión
¿Cuál método le parece más sencillo?
¿Cuál método le parece más natural aplicar?

Z
Ejemplo 38 : Integre cos (t − x) dx.

Solución : En el ejemplo 22 se aplica manipulación algebraica para obtener la familia de primitivas de la


función f (x) = cos (t − x), más precisamente, se aplica la identidad trigonométrica el coseno de la diferencia
de ángulos y por linealidad de la integral indefinida se obtiene la familia de primitivas (ver Ejemplo 22). A
continuación se resuelve la integral por medio de un cambio de variable.

Se propone el cambio de variable


Calculo del
u= t−x −−−−−−−−−→ du = − dt =⇒ dt = −du,
diferencial

entonces, la integral queda

Cambio Diferencial Integral del coseno.


u=t−x dx = −du Integral de tabla.

Z  ↓  ↓ Z z }| {
Z
z }| {
cos t − x dx = cos u (−du) = − cos u du = − sen u + C = − sen (t − x) + C.
↑ | {z }
Linealidad de la integral
Sale de la integral por ser constante [sen u]′ = cos u
respecto a la variable de integración

Luego, Z
cos (t − x) dx = − sen (t − x) + C.

Compare el resultado obtenido en este ejemplo, usando el método de u−sustitución con el resultado que se
obtiene en el Ejemplo 22, usando manipulación algebraica. Según su opinión

Última actualización: Julio 2016 Farith J. Briceño N. farith.math@gmail.com


Matemática II - Guía 3. Método de integración: u-sustitución. 39

¿Cuál método le parece más sencillo?


¿Cuál método le parece más natural aplicar?


Z p
4
Ejemplo 39 : Integre 2p3 x dx.

Solución : Enpel ejemplo 20 se aplica manipulación algebraica para obtener la familia de primitivas de la
función f (x) = 4 2p3 x, más precisamente, se aplica propiedades de radicales y por linealidad de la integral
indefinida se obtiene la familia de primitivas (ver Ejemplo 20). A continuación se resuelve la integral por medio
de un cambio de variable.

Se propone el cambio de variable

Calculo del du
u = 2p3 x −−−−−−−−−→ du = 2p3 dx =⇒ dx = ,
diferencial 2p3

entonces, la integral queda

Diferencial
Integral de una potencia.
du
dx = Integral de tabla.
2p3

Z r ↓ Z Z z }| {
Z
3
√ du 1 √ 1 1 u5/4 1 4u5/4
4 2p x dx =
4
u 3 = 3 4
u du = 3 u1/4 du = 3 +C = 3 + C,
| {z } 2p 2p 2p 2p 5/4 2p 5
| {z }
↑ ↑
Linealidad de la integral
Cambio Z
un+1 1
3
Sale de la integral por ser constante un du = +C con n=
u = 2p x n+1 4
respecto a la variable de integración

como u = 2p3 x, entonces,


Z p
1 4 5/4 2 5/4 4x 1/4
4
2p3 x dx = 3 2p3 x +C = 3 2p3 x +C = 2p3 x + C.
2p 5 5p 5
Finalmente, Z p
4x 1/4
4
2p3 x dx = 2p3 x + C.
5

Compare el resultado obtenido en este ejemplo, usando el método de u−sustitución con el resultado que se
obtiene en el Ejemplo 20, usando manipulación algebraica. Según su opinión

¿Cuál método le parece más sencillo?


¿Cuál método le parece más natural aplicar?


Z
Ejemplo 40 : Integre sen6 x cos x dx.

Solución : Se observa que en el integrando aparece la función seno y su correspondiente derivada, la función
coseno, así, es natural proponer el cambio de variable

Calculo del
u = sen x −−−−−−−−−→ du = cos x dx,
diferencial

con este cambio se espera transformar la integral en una integral sencilla de resolver, es decir, en una integral de
tabla.

Última actualización: Julio 2016 Farith J. Briceño N. farith.math@gmail.com


Matemática II - Guía 3. Método de integración: u-sustitución. 40

Entonces, la integral queda

Integral de una potencia.


Integral de tabla.

Z Z  zZ }| {
6
6 u7 sen7 x
sen x cos x dx = sen
| {z x} cos x dx} = u6 du = +C = + C.
| {z 7 7
↑ ↑ | {z }
Cambio Diferencial Z
n un+1
u du = +C con n=6
u = sen x du = cos x dx n+1

Luego, Z
sen7 x
sen6 x cos x dx = + C.
7

Z √
3
Ejemplo 41 : Integre cos2 t sen t dt.

Solución : Se observa que en el integrando aparece la función coseno y su correspondiente derivada, la


función seno, así, es natural proponer el cambio de variable

Calculo del
u = cos t −−−−−−−−−→ du = − sen t dt =⇒ − du = sen t dt,
diferencial

con este cambio se espera transformar la integral en una integral sencilla de resolver, es decir, en una integral de
tabla.

Entonces, la integral queda

Linealidad de la integral
Integral de una potencia.
Sale de la integral por ser constante
Integral de tabla.
respecto a la variable de integración
zZ }| {
Z √ Z Z  2/3 Z ↓
3 2/3 2/3 2/3
cos2 t sen t dt = cos t sen t dt = cos
|{z}t sen
| {zt dt} = u (−du) = − u du
↑ ↑ | {z }
Cambio Diferencial Z
un+1 2
un du = +C con n=
u = cos t sen t dt = − du n+1 3

u5/3 3u5/3 3
=− +C =− +C =− cos5/3 t + C.
5/3 5 5
Luego, Z √
3 3
cos2 t sen t dt = − cos5/3 t + C.
5

Z
Ejemplo 42 : Integre tan3 (3x) sec2 (3x) dx.

Solución : Se observa que en el integrando aparece la función tangente y su correspondiente derivada, la


función secante, así, es natural proponer el cambio de variable

Calculo del du
u = tan (3x) −−−−−−−−−→ du = sec2 (3x) 3 dx =⇒ = sec2 (3x) dx,
diferencial 3

Última actualización: Julio 2016 Farith J. Briceño N. farith.math@gmail.com


Matemática II - Guía 3. Método de integración: u-sustitución. 41

con este cambio se espera transformar la integral en una integral sencilla de resolver, es decir, en una integral de
tabla.

Entonces, la integral queda

Diferencial
Cambio Integral de una potencia.
du
u = tan (3x) 2
sec (3x) dx = Integral de tabla.
3
↓ ↓ z }| {
Z Z z }| {3 z { Z Z
3 2 2
}|
3 du 1 1 u4
tan (3x) sec (3x) dx = tan (3x) sec (3x) dx = u = u3 du = +C
3 3 3 4
↑ | {z }
Linealidad de la integral
un+1
Z
Sale de la integral por ser constante un du = +C con n=3
n+1
respecto a la variable de integración

u4 tan4 (3x)
= +C = + C.
12 12
Luego, Z
tan4 (3x)
tan3 (3x) sec2 (3x) dx = + C.
12

Z
dx
Ejemplo 43 : Integre √ .
5 − x2
Solución : Es conocido que Z
dx
√ = arcsen x + C.
1 − x2
1 1
Se observa que la función f (x) = √ se diferencia de la función g (x) = √ en la constante 5,
5−x 2 1 − x2
así, si se transforma la función f en alguna función similar a g, entonces la familia de primitivas de f viene
dada por arcosenos.

Al manipular algebraicamente la función f ,

Multiplicar y dividir el Factor Propiedad de radicales Propiedad de potencias


√ √ √
término x2 por 5 común 5 ab = a b an /bn = (a/b)n

↓ ↓ ↓ ↓
1 1 1 1 1 1
f (x) = √ = r = s  = r = s = s 2 ,
5−x2
5x 2
x2
 √ x2 √ x2


x
5− 5 1− 5 1− 5 1 − √ 2 5 1− √
5 5 5 5 5
es decir,
1 1
√ = s
5−x2  2
√ x
5 1− √
5
y la integral se escribe,
Z Z Z
dx dx 1 dx
√ = s 2 = √5 s
5 − x2 √

x

x
2
5 1− √ 1− √
↑ 5 5
Linealidad de la integral
Sale de la integral por ser constante
respecto a la variable de integración

Última actualización: Julio 2016 Farith J. Briceño N. farith.math@gmail.com


Matemática II - Guía 3. Método de integración: u-sustitución. 42

se propone el cambio de variable

x Calculo del 1 √
u= √ −−−−−−−−−→ du = √ dx =⇒ dx = 5 du,
5 diferencial 5

con este cambio se espera transformar la integral en una integral sencilla de resolver, es decir, en una integral de
tabla.

Entonces, la integral queda

Linealidad de la integral
Diferencial Integral de tabla.
√ Sale de la integral por ser constante
dx = 5 du Primitiva : arcoseno.
respecto a la variable de integración

↓ ↓ ւ
Z Z z}|{ Z √ zZ }| {  
dx 1 dx 1 5 du du x
√ = √ v 2 = √5 √ = √ = arcsen u + C = arcsen √ + C.
5−x 2 5 u  1 − u2 1 − u2 5
u
u x
t1 −  √ 
5
|{z}

Cambio
x
u= √
5

Luego, Z  
dx x
√ = arcsen √ + C.
5 − x2 5

Z
dx
Ejemplo 44 : Integre √ .
4x − x2 − 3
Solución : Al completar cuadrado
 2 2
4 (4) 16
2
−x + 4x − 3 = − x + + (−3) − = − (x − 2)2 − 3 + = − (x − 2)2 − 3 + 4
2 (−1) 4 (−1) 4
2 2
= − (x − 2) + 1 = 1 − (x − 2) ,

es decir,
2
−x2 + 4x − 3 = 1 − (x − 2)
y la integral se escribe como Z Z
dx dx
√ = q ,
4x − x2 − 3 1 − (x − 2)
2

se observa que con esta manipulación algebraica la integral es muy similar a la integral de tabla
Z
1
√ du = arcsen u + C,
1 − u2
así, se propone el cambio de variable

Calculo del
u=x−2 −−−−−−−−−→ du = dx,
diferencial

con este cambio se espera transformar la integral en una integral sencilla de resolver, es decir, en una integral de
tabla.

Última actualización: Julio 2016 Farith J. Briceño N. farith.math@gmail.com


Matemática II - Guía 3. Método de integración: u-sustitución. 43

Entonces, la integral queda

Diferencial Integral de tabla.


dx = du Primitiva : arcoseno.

↓ ↓
Z Z z}|{ zZ }| {
dx dx du
√ = s 2 = √ = arcsen u + C = arcsen (x − 2) + C,
4x − x2 − 3  1 − u2
1− x−2
| {z }

Cambio
u=x−2

Luego, Z
dx
√ = arcsen (x − 2) + C.
4x − x2 − 3

Z
dx
Ejemplo 45 : Integre √ .
12x − 4x2 − 5
Solución : Al completar cuadrado
 2 2  2  2
2 12 (12) 3 144 3
−4x + 12x − 5 = −4 x + + (−5) − = −4 x − −5+ = −4 x − −5+9
2 (−4) 4 (−4) 2 16 2
 2
2 3
= − (2) x − + 4 = − (2x − 3)2 + 4 = 4 − (2x − 3)2 ,
2

es decir,
−4x2 + 12x − 5 = 4 − (2x − 3)2 ,
por lo tanto

Multiplicar y dividir el Factor Propiedad de radicales


√ √ √
término (2x − 3)2 por 4 común 4 ab = a b

↓ ↓ ↓
1 1 1 1 1
√ = q = s = v ! = v !
12x − 4x2 − 5 4 − (2x − 3)
2
4 (2x − 3)2
u 2 √
u 2
u (2x − 3) u (2x − 3)
4− t4 1 − 4 t 1−
4 4 4

Propiedad de potencias Operación de racionales


an
 n
a a+b a b
= = +
bn b c c c

↓ ↓
1 1 1 1 1 1 1 1
= s = s = s = s 2 ,
2 2 2 
2x − 3
 2 2 
2x 3
 2 2 
3
(2x − 3)
1− 1− 1− − 1− x−
22 2 2 2 2

entonces
1 1 1
√ = s 2 ,
2
12x − 4x − 5 2 
3
1− x−
2

Última actualización: Julio 2016 Farith J. Briceño N. farith.math@gmail.com


Matemática II - Guía 3. Método de integración: u-sustitución. 44

la integral se escribe como,


Linealidad de la integral
Sale de la integral por ser constante
respecto a la variable de integración


Z Z Z
dx 1 dx 1 dx
√ = s = s 2 ,
12x − 4x2 − 5 2 
3
 2 2 
3
1− x− 1− x−
2 2
se observa que con esta manipulación algebraica la integral es muy similar a la integral de tabla
Z
1
√ du = arcsen u + C,
1 − u2
así, se propone el cambio de variable
3 Calculo del
u=x− −−−−−−−−−→ du = dx,
2 diferencial

con este cambio se espera transformar la integral en una integral sencilla de resolver, es decir, en una integral de
tabla. Entonces, la integral queda
Diferencial Integral de tabla.
dx = du Primitiva : arcoseno.

↓ ↓
Z Z z}|{ zZ }| {  
dx 1 dx 1 du 1 1 3
√ = v !2 = 2 √ = arcsen u + C = arcsen x − + C.
12x − 4x2 − 5 2 u 1 − u2 2 2 2
u
t1 − 3
x−
| {z 2}

Cambio
3
u = x−
2

Luego, Z  
dx 1 3
√ = arcsen x − + C.
12x − 4x2 − 5 2 2

Z
dx
Ejemplo 46 : Integre .
6 + x2
Solución : Es conocido que Z
dx
= arctan x + C,
1 + x2
1 1
Se observa que la función f (x) = se diferencia de la función g (x) = en la constante 6,
6 + x2 1 + x2
así, si se transforma la función f en alguna función similar a g, entonces la familia de primitivas de f viene
dada por arcotangente.

Al manipular algebraicamente la función f ,


Multiplicar y dividir el Factor Propiedad de radicales Propiedad de potencias
√ √ √
término x2 por 6 común 6 ab = a b an /bn = (a/b)n

↓ ↓ ↓ ↓
1 1 1 1 1
f (x) = = =   = ! = 2 ! ,
6 + x2 6x2 x2 x2

x
6+ 6 1+ 6 1 + √ 2 6 1+ √
6 6 6
6

Última actualización: Julio 2016 Farith J. Briceño N. farith.math@gmail.com


Matemática II - Guía 3. Método de integración: u-sustitución. 45

es decir,
1 1
= 2 !
6 + x2 
x
6 1+ √
6
y la integral se escribe,
Z Z Z
dx dx 1 dx
= 2 ! = 6 2 !
6 + x2 
x

x
6 1+ √ 1+ √
6 6

Linealidad de la integral
Sale de la integral por ser constante
respecto a la variable de integración

se propone el cambio de variable

x Calculo del 1 √
u= √ −−−−−−−−−→ du = √ dx =⇒ dx = 6 du,
6 diferencial 6

con este cambio se espera transformar la integral en una integral sencilla de resolver, es decir, en una integral de
tabla.

Entonces, la integral queda

Diferencial Integral de tabla.



dx = 6 du Primitiva : arcotangente.

↓ ↓
Z Z z}|{ Z √ √ Zz }| { √ √  
dx 1 dx 1 6 du 6 du 6 6 x
=  2 = = = arctan u + C = arctan √ + C.
6 + x2 6 6 1 + u2 6 1 + u2 6 6 6
x
1+√ 
6
|{z}

Cambio
x
u= √
6

Luego, √  
Z
dx 6 x
= arctan √ + C.
6 + x2 6 6

Z
x dx
Ejemplo 47 : Integre .
1 + x4
Solución : Escribimos la integral como
Z Z
x dx x dx
= 2,
1 + x4 1 + (x2 )
se propone el cambio de variable
Calculo del du
u = x2 −−−−−−−−−→ du = 2 dx =⇒ dx = ,
diferencial 2

con este cambio se espera transformar la integral en una integral sencilla de resolver, es decir, en una integral de
tabla.

Última actualización: Julio 2016 Farith J. Briceño N. farith.math@gmail.com


Matemática II - Guía 3. Método de integración: u-sustitución. 46

Entonces, la integral queda

Diferencial
du
dx =
2

Z Z z }| { Z Z
x dx x dx du/2 1 du 1 1 
=  2 = = = arctan u + C = arctan x2 + C.
1 + x4 1+u 2 2 1+u 2 2 2
x2
1 + |{z} | {z }
↑ ↑
Cambio Integral de tabla.
u = x2 Primitiva : arcotangente.

Luego, Z
x dx 1 
4
= arctan x2 + C.
1+x 2

Z
dx
Ejemplo 48 : Integre .
x2 + 10x + 26
Solución : Al completar cuadrado
2
x2 + 10x + 26 = (x + 5) + 1,
así, la integral se escribe como Z Z
dx dx
= ,
x2 + 10x + 26 (x + 5)2 + 1
se observa que con esta manipulación algebraica la integral es muy similar a la integral de tabla
Z
1
du = arctan u + C,
1 + u2
se propone el cambio de variable
Calculo del
u=x+5 −−−−−−−−−→ du = dx,
diferencial

con este cambio se espera transformar la integral en una integral sencilla de resolver, es decir, en una integral de
tabla.

Entonces, la integral queda

Diferencial Integral de tabla.


dx = du Primitiva : arcotangente.

↓ ↓
Z Z z}|{ zZ }| {
dx dx du
=  2 = = arctan u + C = arctan (x + 5) + C.
x2 + 10x + 26 u2 + 1
x+5 +1
| {z }

Cambio
u =x+5

Luego, Z
dx
= arctan (x + 5) + C.
x2 + 10x + 26

Última actualización: Julio 2016 Farith J. Briceño N. farith.math@gmail.com


Matemática II - Guía 3. Método de integración: u-sustitución. 47

Z
dx
Ejemplo 49 : Integre .
x2 − 6x + 13
Solución : Al completar cuadrado
2
x2 − 6x + 13 = (x − 3) + 4,
es decir,
1 1
f (x) = =
x2 − 6x + 13 (x − 3)2 + 4

Al manipular algebraicamente la función f ,


Multiplicar y dividir el Factor Propiedad de radicales Propiedad de potencias
√ √ √
término (x − 3)2 por 4 común 4 ab = a b an /bn = (a/b)n

↓ ↓ ↓ ↓
1 1 1 1 1
f (x) = 2 = 2 = 2
!=
2
! =  2 ! ,
(x − 3) + 4 4 (x − 3) (x − 3) (x − 3) x−3
+4 4 1+ 4 1+ 2 4 1+
4 4 (2) 2
es decir,
1 1 1
= 2 = 2 !
x2 − 6x + 13 (x − 3) + 4

x−3
4 1+
2
y la integral se escribe,
Z Z Z
dx dx 1 dx
2
=  2 ! = 4  2
x − 6x + 13 x−3 x−3
4 1+ 1+
2 2

Linealidad de la integral
Sale de la integral por ser constante
respecto a la variable de integración

se observa que con esta manipulación algebraica la integral es muy similar a la integral de tabla
Z
1
du = arctan u + C,
1 + u2
se propone el cambio de variable
x−3 Cálculo del 1
u= −−−−−−−−−→ du = dx =⇒ 2 du = dx,
2 diferencial 2
con este cambio se espera transformar la integral en una integral sencilla de resolver, es decir, en una integral de
tabla.

Entonces, la integral queda


Diferencial Integral de tabla.
dx = 2 du Primitiva : arcotangente.

↓ ↓
Z Z z}|{ zZ }| {  
dx 1 dx 1 2 du 1 1 x−3
= !2 = = arctan u + C = arctan + C.
x2 − 6x + 13 4 x−3 4 u2 + 1 2 2 2
+1
2 }
| {z

Cambio
x−3
u=
2

Última actualización: Julio 2016 Farith J. Briceño N. farith.math@gmail.com


Matemática II - Guía 3. Método de integración: u-sustitución. 48

Luego, Z  
dx 1 x−3
= arctan + C.
x2 − 6x + 13 2 2

Z
x dx
Ejemplo 50 : Integre √ .
1 − x2
1
Solución : No se debe confundir esta integral con la integral de la función f (x) = √ , que tiene
1 − x2
como primitiva a la función arcoseno, ya que, el diferencial de la integral dada está multiplicado por la variable
x, así, se propone el cambio de variable
Cálculo del du
u = 1 − x2 −−−−−−−−−→ du = −2x dx =⇒ − = x dx,
diferencial 2
con este cambio se espera transformar la integral en una integral sencilla de resolver, es decir, en una integral de
tabla. Entonces, la integral queda

Diferencial Linealidad de la integral


Integral de una potencia.
du Sale de la integral por ser constante
x dx = − Integral de tabla.
2 respecto a la variable de integración

↓ ↓ ↓
Z z }| { Z − 1 du Z z}|{ Z Z z
Z }| {
x dx 2 1 du 1 du 1 du 1 −1/2 1 u1/2
q = √ = − √ =− √ =− = − u du = − +C
1 − x2 u 2 u 2 u 2 u1/2 2 2 1
| {z } | {z }
2
↑ ↑
Cambio Z
un+1 1
un du = +C con n=−
u = 1 − x2 n+1 2

√ √
= − u1/2 + C = − u+C =− 1 − x2 + C.
Luego. Z
x dx p
√ = − 1 − x2 + C.
1 − x2

Z

Ejemplo 51 : Integre x x + 3 dx.

Solución : Se propone el cambio de variable


Cálculo del
u=x+3 de aquí x=u−3 −−−−−−−−−→ du = dx,
diferencial

con este cambio se espera transformar la integral en una integral sencilla de resolver, es decir, en una integral de
tabla. Entonces, la integral queda

Linealidad de la integral
Cambio Diferencial Z Z Z
x=u−3 dx = du (f (x) + g (x)) dx = f (x) dx + g (x) dx

Z ↓ ↓ z }| { Z
z}|{ q z}|{ Z √
Z   Z
3/2 1/2
x x + 3 dx = (u − 3) u du = u − 3u du = u3/2 du − 3u1/2 du
| {z }
↑ ↑
Linealidad de la integral
Cambio
Sale de la integral por ser constante
u=x+3
respecto a la variable de integración

Última actualización: Julio 2016 Farith J. Briceño N. farith.math@gmail.com


Matemática II - Guía 3. Método de integración: u-sustitución. 49

Integral de una potencia.


Integral de tabla.

↓ ↓
zZ }| { z
Z }| {
3/2 1/2 u5/2 u3/2 2
= u du − 3 u du = −3 = u5/2 − 2u3/2 + C.
5 3 5
| {z } | {z }
2 2
↑ ↑
un+1 3 un+1 1
Z Z
un du = +C con n= un du = +C con n=
n+1 2 n+1 2

Luego, Z
√ 2 5/2 3/2
x x + 3 dx = (x + 3) − 2 (x + 3) + C.
5

Z

Ejemplo 52 : Integre tan x sen x dx.

sen x
Solución : Puesto que, tan x = , entonces
cos x
Z √ Z r Z r Z
sen x sen2 x sen x
tan x sen x dx = sen x dx = dx = √ dx,
cos x cos x cos x
se observa que la derivada de la función y = cos x está presente en el integrando, salvo una constante, eso
sugiere el cambio de variable
Cálculo del
u = cos x −−−−−−−−−→ du = − sen x dx =⇒ − du = sen x dx,
diferencial

con este cambio se espera transformar la integral en una integral sencilla de resolver, es decir, en una integral de
tabla.

Entonces, la integral queda

Diferencial
Integral de una potencia.
− du = sen x dx
Integral de tabla.

Z Z z }| { Z z
Z }| {
√ sen x dx du −1/2 u1/2 √
tan x sen x dx = q = √ = u du = + C = 2u1/2 + C = 2 cos x + C.
cos u 1
| {zx} | {z }
↑ 2
un+1 1
Z
Cambio un du = +C con n=−
n+1 2
u = cos x

Luego, Z
√ √
tan x sen x dx = 2 cos x + C


Z
Ejemplo 53 : Integre cot7 x csc2 x dx.

Solución : Se observa que la derivada de la función y = cot x está presente en el integrando, salvo una
constante, eso sugiere el cambio de variable
Cálculo del
u = cot x −−−−−−−−−→ du = − csc2 x dx =⇒ − du = csc2 x dx,
diferencial

Última actualización: Julio 2016 Farith J. Briceño N. farith.math@gmail.com


Matemática II - Guía 3. Método de integración: u-sustitución. 50

con este cambio se espera transformar la integral en una integral sencilla de resolver, es decir, en una integral de
tabla.

Entonces, la integral queda

Cambio Diferencial Integral de una potencia.


u = cot x − du = csc2 x dx Integral de tabla.

↓ ↓ zZ }| {
Z Z z }| {7 z }| { Z
7 2 u8 cot8 x
cot x csc x dx = cot x 2 7
csc x dx = u (− du) = − u7 du = − +C =− + C.
↑ 8 8
| {z }
Linealidad de la integral
un+1
Z
Sale de la integral por ser constante un du = +C con n=7
n+1
respecto a la variable de integración

Luego, Z
cot8 x
cot7 x csc2 x dx = − + C.
8

Z
Ejemplo 54 : Integre tan5 x sec2 x dx.

Solución : Se observa que la derivada de la función y = tan x está presente en el integrando, eso sugiere el
cambio de variable
Cálculo del
u = tan x −−−−−−−−−→ du = sec2 x dx,
diferencial

con este cambio se espera transformar la integral en una integral sencilla de resolver, es decir, en una integral de
tabla.

Entonces, la integral queda

Cambio Diferencial Integral de una potencia.


u = tan x du = sec2 x dx Integral de tabla.

↓ ↓ z }| {
Z Z z }| {5 z }| { Z
5 2 u6 tan6 x
tan x sec x dx = 2
tan x sec x dx = u5 du = +C = + C.
6 6
| {z }
un+1
Z
un du = +C con n=5
n+1

Luego, Z
tan6 x
tan5 x sec2 x dx = + C.
6

Z
x2 dx
Ejemplo 55 : Integre √ 4.
(x − 2 x)
Solución : Se propone el cambio de variable

Cálculo del
u2 = x −−−−−−−−−→ 2u du = dx,
diferencial

con este cambio se espera transformar la integral en una integral sencilla de resolver, es decir, en una integral de
tabla.

Última actualización: Julio 2016 Farith J. Briceño N. farith.math@gmail.com


Matemática II - Guía 3. Método de integración: u-sustitución. 51

Entonces, la integral queda


Z Z 2 Z Z Z
x2 dx u2 2u du u4 2u du u4 2u du u du
√ 4 = 4 = 4 = 4 =2 4,
(x − 2 x) (u2 − 2u) (u (u − 2)) u4 (u − 2) (u − 2)

Para resolver la nueva integral se propone el siguiente cambio de variable

Cálculo del
p=u−2 de aquí u=p+2 −−−−−−−−−→ du = dp,
diferencial

con este nuevo cambio se espera transformar la integral en una integral sencilla de resolver, es decir, en una
integral de tabla. Así,
Z Z Z   Z Z Z Z
u du p+2 p 2 p 2
4 = dp = + 4 dp = dp + dp = p dp + 2 p−4 dp
−3
(u − 2) p4 p4 p p4 p4

p−2 p−3 1 2 1 2
= +2 =− 2 − 3
+C =− 2 − 3 + C1 ,
−2 −3 2p 3p 2 (u − 2) 3 (u − 2)
ya que, p = u − 2.

Entonces
Z !
u du 1 2 1 4
2 4 =2 − 2 − 3 + C1 =− 2 − 3 +C
(u − 2) 2 (u − 2) 3 (u − 2) (u − 2) 3 (u − 2)

como u = x, y Z Z
x2 dx u du
√ 4 =2 4,
(x − 2 x) (u − 2)
se tiene Z
x2 dx 1 4
√ 4 =− √ 2 − √ 3 + C.
(x − 2 x) ( x − 2) 3 ( x − 2)

Z
sen x
Ejemplo 56 : Integre dx.
2 − sen2 x
Solución : Por la identidad trigonométrica básica

sen2 x + cos2 x = 1, se tiene que, sen2 x = 1 − cos2 x

y el integrando se escribe como


sen x sen x sen x sen x
f (x) = = = = 2,
2 − sen2 x 2 − (1 − cos2 x) 1 + cos2 x 1 + (cos x)

mientras que, la integral queda Z Z


sen x sen x
dx = 2 dx,
2 − sen2 x 1 + (cos x)
se propone el cambio de variable

Cálculo del
u = cos x −−−−−−−−−→ du = − sen x dx =⇒ − du = sen x dx,
diferencial

con este cambio se espera transformar la integral en una integral sencilla de resolver, es decir, en una integral de
tabla.

Última actualización: Julio 2016 Farith J. Briceño N. farith.math@gmail.com


Matemática II - Guía 3. Método de integración: u-sustitución. 52

Entonces,
Z Z Z
sen x sen x − du
dx = 2 dx = = − arctan u + C = − arctan (cos x) + C.
2 − sen2 x 1 + (cos x) 1 + u2

Luego, Z
sen x
dx = − arctan (cos x) + C.
2 − sen2 x

Z

Ejemplo 57 : Integre 1 − sen x dx.

Solución : Se tiene
Z Z r Z r Z r Z
√ (1 − sen x) (1 + sen x) 1 − sen2 x cos2 x cos x dx
1 − sen x dx = dx = dx = dx = √ .
1 + sen x 1 + sen x 1 + sen x 1 + sen x
Se propone el cambio de variable

Cálculo del
u = 1 + sen x −−−−−−−−−→ du = cos x dx,
diferencial

con este cambio se espera transformar la integral en una integral sencilla de resolver, es decir, en una integral de
tabla.

Entonces, la integral nos queda,


Z Z Z Z
√ cos x dx du u1/2 √ √
1 − sen x dx = √ = √ = u−1/2 du = + C = 2 u + C = 2 1 + sen x + C.
1 + sen x u 1
2
Luego, Z
√ √
1 − sen x dx = 2 1 + sen x + C.


Z
Ejemplo 58 : Integre sen2 x dx.

Solución : Es conocido que


1 − cos (2x)
sen2 x = ,
2
así, Z Z Z Z Z
1 − cos (2x) 1 1 1
sen2 x dx = dx = (1 − cos (2x)) dx = dx − cos (2x) dx,
2 2 2 2
donde, la primera integral del lado derecho de la igualdad es inmediata
Z
dx = x + C1 ,

mientras que, la segunda integral del lado derecho de la igualdad la resolvemos al proponer el cambio de variable

Cálculo del du
u = 2x −−−−−−−−−→ du = 2 dx =⇒ = dx,
diferencial 2

con este cambio se espera transformar la integral en una integral sencilla de resolver, es decir, en una integral de
tabla.

Última actualización: Julio 2016 Farith J. Briceño N. farith.math@gmail.com


Matemática II - Guía 3. Método de integración: u-sustitución. 53

Entonces, la integral queda


Z Z Z
du 1 1 1
cos (2x) dx = cos u = cos u du = sen u + C2 = sen (2x) + C2 .
2 2 2 2
Luego, Z  
2 1 sen (2x) x sen (2x)
sen x dx = x− +C = − + C.
2 2 2 4

Z
sen (2x) + cos x
Ejemplo 59 : Integre 2 dx.
(sen2 x + sen x − 2)

Solución : Es conocido que


sen (2x) = 2 sen x cos x,
así, la integral se expresa como
Z Z
sen (2x) + cos x 2 sen x cos x + cos x
2 dx = dx.
(sen2 x + sen x − 2) (sen2 x + sen x − 2)2

Se propone el cambio de variable

Cálculo del
u = sen2 x + sen x − 2 −−−−−−−−−→ du = (2 sen x cos x + cos x) dx,
diferencial

con este cambio se espera transformar la integral en una integral sencilla de resolver, es decir, en una integral de
tabla.

Entonces, la integral queda


Z Z Z
sen (2x) + cos x 2 sen x cos x + cos x du 1 1
2 2 dx = 2 2 dx = u 2
=− +C =−
u sen2 x + sen x − 2
+ C.
(sen x + sen x − 2) (sen x + sen x − 2)

Luego, Z
sen (2x) + cos x 1
2 dx = − + C.
(sen2 x + sen x − 2) sen2 x + sen x − 2

Z
(x + 1) dx
Ejemplo 60 : Integre √ √ .
3x + x − 2
Solución : Aplicando la conjugada se obtiene
Z Z √ √ Z √ √ 
(x + 1) dx (x + 1) 3x − x − 2 (x + 1) 3x − x − 2
√ √ = √ √ √ √ dx = √ 2 √ 2 dx
3x + x − 2 3x + x − 2 3x − x − 2 3x − x−2

Z √ √  Z √ √ 
(x + 1) 3x − x − 2 (x + 1) 3x − x − 2
= dx = dx
3x − (x − 2) 2x + 2
Z √ √  Z
(x + 1) 3x − x − 2 1 √ √ 
= dx = 3x − x − 2 dx
2 (x + 1) 2
Z Z 
1 √ √
= 3x dx − x − 2 dx ,
2

Última actualización: Julio 2016 Farith J. Briceño N. farith.math@gmail.com


Matemática II - Guía 3. Método de integración: u-sustitución. 54


donde, para obtener la familia de primitivas de la función f (x) = 3x se propone el cambio de variable
Cálculo del du
u = 3x −−−−−−−−−→ du = 3 dx =⇒ = dx,
diferencial 3

con este cambio se espera transformar la integral en una integral sencilla de resolver, es decir, en una integral de
tabla.

Entonces, la integral queda


Z √ Z Z
√ du 1 1 u3/2 1 2 3/2 2 3/2
3x dx = u = u1/2 du = + C1 = u + C1 = (3x) + C1 ,
3 3 3 3 3 3 9
2
por lo tanto, Z √
2
3x dx = (3x)3/2 + C1 .
9

Por otro lado, para obtener la familia de primitivas de la función f (x) = x − 2 se propone el cambio de
variable
Cálculo del
z =x−2 −−−−−−−−−→ dz = dx,
diferencial

con este cambio se espera transformar la integral en una integral sencilla de resolver, es decir, en una integral de
tabla.

Entonces, la integral queda


Z Z Z
√ √ u3/2 2 2 3/2
x − 2 dx = u du = u1/2 du = + C2 = u3/2 + C2 = (x − 2) + C2 ,
3 3 3
2
es decir, Z
√ 2 3/2
x − 2 dx = (x − 2) + C2 .
3
Así,
Z Z Z   
(x + 1) dx 1 √ √ 1 2 3/2 2 3/2
√ √ = 3x dx − x − 2 dx = (3x) + C1 − (x − 2) − C2
3x + x − 2 2 2 9 3

1 3/2 1 3/2
= (3x) − (x − 2) + C,
9 3
C1 − C2
donde, C = .
2
Luego, Z
(x + 1) dx 1 1
√ √ = (3x)3/2 − (x − 2)3/2 + C.
3x + x − 2 9 3

Z
tan3 (1 − 2t)
Ejemplo 61 : Integre dt.
cos (1 − 2t)
Solución : Se propone el cambio de variable
Cálculo del du
u = 1 − 2t −−−−−−−−−→ du = −2 dt =⇒ − = dx,
diferencial 2

con este cambio se espera transformar la integral en una integral sencilla de resolver, es decir, en una integral de
tabla.

Última actualización: Julio 2016 Farith J. Briceño N. farith.math@gmail.com


Matemática II - Guía 3. Método de integración: u-sustitución. 55

Entonces, la integral queda


Z Z   Z
tan3 (1 − 2t) tan3 u du 1 tan3 u
dt = − =− du,
cos (1 − 2t) cos u 2 2 cos u
tan3 u
para obtener la familia de primitiva de la función f (u) = se escribe la integral como
cos u
Z Z Z Z
tan3 u 3 2

du = tan u sec u du = tan u tan u sec u du = sec2 u − 1 tan u sec u du
cos u
y se propone el nuevo cambio de variable
Cálculo del
z = sec u −−−−−−−−−→ dz = tan u sec u dt,
diferencial

con este cambio se espera transformar la integral en una integral sencilla de resolver, es decir, en una integral de
tabla.

Entonces, la integral queda


Z Z Z
tan3 u   z3 1
du = sec2 u − 1 tan u sec u du = z 2 − 1 dz = − z + C1 = sec3 u − sec u + C1 ,
cos u 3 3
por lo tanto, Z
tan3 u 1
du = sec3 u − sec u + C1 ,
cos u 3
1
ahora, se expresa la familia de primitiva F (u) = sec3 u − sec u + C1 , en términos de la variable de integración
3
t, puesto que
u = 1 − 2t,
se tiene que,
Z Z  
tan3 (1 − 2t) 1 tan3 u 1 1 1 1
dt = − du = − sec3 u − sec u + C1 = − sec3 (1 − 2t) + sec (1 − 2t) + C,
cos (1 − 2t) 2 cos u 2 3 6 2
C1
donde C = − .
2
Luego, Z
tan3 (1 − 2t) 1 1
dt = − sec3 (1 − 2t) + sec (1 − 2t) + C.
cos (1 − 2t) 6 2

Ejercicios

Calcular las siguientes integrales utilizando el método de la u-sustitución


Z Z Z Z Z
2 2 2 2 2
1. (x + 4) dx 2. (a + bt) dt 3. (3 − t) dt 4. (3 − 2t) dt 5. (a − bt) da
Z Z Z Z Z
2 3 3 3 3
6. (ax + b) dx 7. (x + 2) dx 8. (x + 4) dx 9. (a + bt) dt 10. (3 − 2t) dt
Z Z Z Z Z
4 6 7 8 n
11. (a − bt) dt 12. (3x + 5) dx 13. (x − 5) dx 14. (9 − 2t) dt 15. (a + bx) dx

Z Z Z Z Z
n m r n m r n 1/n dx
16. (a − bx) dx 17. (a + b x) dx 18. (a − b x) dx 19. (at) dt 20. √
4
5x

Última actualización: Julio 2016 Farith J. Briceño N. farith.math@gmail.com


Matemática II - Guía 3. Método de integración: u-sustitución. 56

Z  Z Z Z Z
n 1/n dx dx dx dx
21. at + dt 22. √ 23. 5 24. 3 25. 10
a 4
5x − 3 (x + 1) (3x + 4) (2 − x)
Z Z Z √ Z p Z p
dx dx 3 4
26. π 27. n 28. 9x dx 29. 2pt dt 30. 2p3 x dx
(a − bx) (a − bx)
Z Z √ Z √ Z √ Z √
dp 3 3
31. p 32. a + bx dx 33. a − bx dx 34. a + bx dx 35. a − bx dx
5
2pt2
Z r Z Z Z q
3
x √
n

n n m
36. a − dx 37. a + bx dx 38. a − bx dx 39. (a + bx) dx
b
Z q Z r Z r Z
x m x m dx
40. n
(a − bt)m dt n
41. a+ dx 42. n
a− dx 43. √
b b 5x − 2
Z Z Z Z Z
dx dx dx dx
44. √7
45. √ 46. √
n
47. √ 48. cos (6t) dt
7x + 1 m
ax + m b − ax b − a2 x
Z Z Z x Z  πx  Z  
t
49. sen (3t) dt 50. cos (−x) dx 51. cos dx 52. sen dx 53. sen dt
2 2 2
Z Z π  Z Z  
2πx
54. sen (π − t) dt 55. cos − x dx 56. sen (1 − 2t) dt 57. sen − φ0 dx
2 T
Z Z Z   Z  
πωt πωt
58. cos (6x) dx 59. cos (at) dt 60. cos + α dt 61. cos + α dα
2 2
Z Z Z Z
62. sen (t − x) dx 63. sen (t − x) dt 64. cos (t − x) dx 65. cos (t − x) dt
Z Z Z Z
66. sec2 (t − x) dx 67. csc2 (t − x) dx 68. sec2 (t − x) dt 69. csc2 (t − x) dt

Z Z Z √ 5 Z √ 5
x dx x dx
70. √ 71. √
5
72. 3 x − π 5 dx 73. 3x − π 5 dx
1 − x2 1 − x2
Z Z Z Z Z
dx dx dx dx
74. 75. 76. 77. 78. sen2 x cos x dx
4 + x2 6 + x2 3 + 4x2 5 + 3x2
Z Z Z
79. sen3 x cos x dx 80. cos−2/3
(bx) sen (bx) dx 81. sen−1/5 (ax) cos (ax) dx

Z Z √ Z Z
3 sen5 (6x)
82. sen6 x cos x dx 83. cos2 t sen t dt 84. dx 85. tan6 x sec2 x dx
sec (6x)
Z Z Z
5/2 2
86. tan x sec x dx 87. tan3 (3x) sec2 (3x) dx 88. tan2/3 (ax) sec2 (ax) dx
Z Z Z q
89. cot x csc2 x dx 90. cot5 x csc2 x dx 91. 4
cot3 (1 − x) csc2 (1 − x) dx

Z Z √ p √ Z Z
3 2 csc2 ( x) cot ( x) dx dx
92. cot (2x) csc (2x) dx 93. √ dx 94. √ 95. √
x 9 − x2 5 − x2
Z Z Z Z Z
dx dx x dx x dx
96. √ 97. √ 98. √ 99. √
3
100. sen (1 − 2t) dt
4 − 3x2 9 − 5x2 x2 + 1 x2 + 1

Última actualización: Julio 2016 Farith J. Briceño N. farith.math@gmail.com


Matemática II - Guía 3. Método de integración: u-sustitución. 57

Z Z Z Z Z
x dx √ x dx
101. sen2 x dx 102. cos2 t dt 103. 104. x 4 − x dx 105. 4
1 + x4 (x + 1)
Z √ Z Z Z
tan t sen z cos z 2
106. dt 107. √ dz 108. sen (3x) dx 109. cos2 (3x) dx
1 − sen2 t cos2 z − sen2 z
Z Z Z Z
x3 dx
110. sec2 (3x) dx 111. tan2 (3x) dx 112. tan2 (3x) sec2 (3x) dx 113. 5
(x4 + 1)
Z Z Z p Z
dx dx dx
114. √ 115. √ 116. 1 + 3 cos2 x sen (2x) dx 117.
4x − x2 6x − x2 − 4 π + x2
Z √ Z Z Z
3
x dx √ cos3 x dx
118. p √ 119. cos3 (2t) dt 120. t2 5t − 2 dt 121. 2
2− 3x (3 sen x − sen3 x + 5)
Z Z Z Z Z
cot3 t tan3 t √ 2 3 dx
122. dt
123. dt 124. x 6 − x dx 125. sen x dx 126.
sen2 t cos2 t 7 + x2
Z Z Z Z
2t2 + t √ dx
127. 5 dt 128. t 2t − 1 dt 129. sen (2 cos x) sen x dx 130. 5
(t + 1) 3 − x1/3
Z Z √ Z Z
4t2 + 3t 3
2x x2 − 3 √
131. 5 dt 132. p √ dx 133. √
3
dx 134. x2 3 − 5x dx
(t + 1) 2 − 3 2x 1−x
Z Z Z Z
x2 dx x3 dx sen (4t − 1) dt
135. √ 136. √ 137. 138. (cos (ax) + sen (ax))2 dx
3
x3 + 1 a2 − x2 1 − sen2 (4t − 1)
Z Z ZZ
x5 dx x dx x7 dx p
139. √ 140. √141. √ 142. sen x cos x 1 + sen2 x dx
x2 − 3 1 − x4 x4 − 1
Z Z √ Z Z
x dx dx sec x tan x
143. √ 144. tan x sen x dx 145. √ 146. √ dx
1 + 2x 2
4x − x − 3 sec x + 1
Z Z √ Z √ Z √
x2 dx sec2 t − 1 x dx 3 − arctan 2x
147. √ 148. √ dt 149. p √ 150. dx
1 − x6 t−1 3
x+3 1 + 4x2
Z √ Z Z Z
sen 1 − t x2 dx x3 − 3x2
151. √ dt 152. √ 4 153. √
3
dx 154. (sec (ax) + tan (ax))2 dx
1−t (x − 2 x) 4−x
Z Z Z Z
sen2 (arctan x) cos x dx 1 + cos (2x) tan3 (1 − 2t)
155. dx 156. 157. dx 158. dt
x2 + 1 1 + sen2 x sen2 (2x) cos (1 − 2t)
Z Z Z Z
x5 dx x2 dx x x
159. √ 160. √ 161. sen4 (at) dt 162. cos sen dx
3
x2 + 1 25 − x6 a a
Z Z p3
√ Z Z
x dx 2−5 x sen (2x) dx (x + 1) dx
163. √ 164. √ dx 165. 2 166. √ √
2x2 − x4 x (1 + sen2 x) 3x + x − 2
Z Z Z Z
sen x dx cot5 (2 − t) x+3 x+1
167. 168. dt 169. 3 dx 170. dx
cos2 x + 1 sen (2 − t) (x + 2) (x − 2)4
Z Z Z
dx sen (4x) dx sen3 (2 + arcsen (1 − t))
171. q 172. p 173. √ dt
(1 − x2 ) (1 − arcsen x)3 cos (4x) + 4 2t − t2

Última actualización: Julio 2016 Farith J. Briceño N. farith.math@gmail.com


Matemática II - Guía 3. Método de integración: u-sustitución. 58

Z √ 
Z Z √ Z
x−1/2 tan3 ( x) tan3 x−1 x x−1 x−1
174. √ dx 175. dx 176. dx 177. √ dx
1 − sen2 ( x) x2 sec (x−1 ) x+3 1+ x
Z Z Z Z
sen (2x) dx cot (ax) dx dx √
178. √ 179. 180. 2
181. t2 3t + 2 dt
4
2 − sen x ln (sen (ax)) 4x − 4x + 17
Z √ Z Z Z
sen ( x) 2 x4
182. √ dx 183. cos (sen x) cos x dx 184. √ dx 185. sen5 t dt
x x3/2 + 2 x
Z √ Z Z p Z 4
sen5 ( x) 5 3 2
x dx
186. √ dx 187. cos (2t) dt 188. 4 + 5 cos x sen (2x) dx 189.
x x2 + 3
Z √ Z Z Z
x dx x6 dx cos3 x dx sen x dx
190. 191. 192. 2 193.
x+3 a2 + x2 (3 sen x − sen3 x + 5) 4 cos2 x − 4 cos x + 17
Z Z Z Z
 dx dx dx
194. x sen 1 − x2 dx 195. 196. √ 197. √
9x2 − 6x + 2 x 2x − 1 4x − 4x2 + 3
Z Z Z
sen (cos (sen x)) sen (sen x) √ √
198. dx 199. tan x sec x dx 200. tan x 3 sec x dx
sec x sec3 (cos (sen x))
Z Z Z Z √
√ √ √ m
csc x
201. tan x n sec x dx 202. cot x csc x dx 203. cot x 5
csc x dx 204. dx
tan x
Z Z Z Z
dx √
3 dx 2x2 + 3
205. √ 206. sen x 3 − cos x dx 207. 2
208. dx
6x − 9x2 x + 6x + 10 x2 + 7
Z Z Z r Z
√ sec2 t dt arcsen t
arcsen t + t
209. 1 − sen x dx 210. √ 211. √ dt dt
212.
1 − tan2 t 1 − t2
1 − t2
Z Z Z Z
p 5x dx

x8 2
x tan2 2x2 + 3 dx
4
213. a + bx3 dx 214. tan (ωt − π) dt 215. 216.
π 2 + x4
Z Z Z Z
  dx dx
217. x sen2 3x2 dx 218. x sen π − x2 dx 219. √ 220. √
4 − 3x2 2π − x2
Z Z Z 2 p
sen x − cos x p x − arctan (2x)
221. dx 222. sen (2x) 4 π − cos (2x) dx 223. dx
1 + sen (2x) 1 + 4x2
Z Z Z Z
dx dx √
224. 5 225. 4 226. x x + 3 dx 227. cos (3x + π) dx
5 − x1/3 3 − x1/3
Z Z Z
a cos x dx √ sen x dx
228. √ 229. t2 4 − t dt 230. √
a − sen2 x cos x − cos2 x
Respuestas: Ejercicios

1. 1
3 (x + 4)3 + C; 2. 1
3b (a + bt)3 + C; 3. − 1
3 (3 − t)3 + C; 4. − 1
6 (3 − 2t)3 + C; 5. 1
3 (a − bt)3 + C;

6. 1
3a (ax + b)3 + C; 7. 1
4 (x + 2)4 + C; 8. 1
4 (x + 4)4 + C; 9. 1
4b (a + bt)4 + C; 10. − 1
8 (3 − 2t)4 + C;
1 5 1 7 1 8 1 9
11. − 5b (a − bt) + C; 12. 21 (3x + 5) + C; 13. 8 (x − 5) + C; 14. − 18 (9 − 2t) + C;
n+1 n+1 m
15. 1
(n+1)b (a + bx) + C; 16. − 1
(n+1)b (a − bx) + C; 17. 1
(n+1)br (a + b x)n+1 + C;
r

18. − 1
(n+1)br
(am − br x)n+1 + C; 19. n
a(n+1)
(at)(n+1)/n + C; 20. 4
15 (5x)3/4 + C;
n (n+1)/n
n 4
(5x − 3)3/4 + C; 1 1

21. a(n+1) at + a + C; 22. 15 23. − 4(x+1)4
+ C; 24. − 6(3x+4)2
+ C;

1 1 1−π 1 1−n 1 4/3


25. 9(2−x)9
+ C; 26. (π−1)b
(a − bx) + C; 27. (n−1)b
(a − bx) + C; 28. 12 (9x) + C;
5/4 4/5
29. 1
3p (2pt)3/2 + C; 30. 2
5p3
2p3 x + C; 31. 5
8t2
2pt2 + C; 32. 2
3b (a + bx)3/2 + C;

Última actualización: Julio 2016 Farith J. Briceño N. farith.math@gmail.com


Matemática II - Guía 3. Método de integración: u-sustitución. 59

4/3
33. − 2
3b (a − bx)3/2 + C; 34. 3
4b (a + bx)4/3 + C; 35. − 3
4b (a − bx)4/3 + C; 36. − 3b
4 a− x
b + C;
n (1+n)/n n (1+n)/n n (m+n)/n
37. (n+1)b
(a + bx) + C; 38. − (n+1)b
(a − bx) + C; 39. (m+n)b
(a + bx) + C;

x (m+n)/n
n (m+n)/n nb
 nb x
(m+n)/n
40. − (m+n)b
(a − bt) + C; 41. m+n a+ b + C; 42. − m+n a− b + C;
2
√ 1 6/7 m (m−1)/m
43. 5 5x − 2 + C; 44. 6 (7x + 1) + C; 45. (m−1)a (ax + m) + C;
n (n−1)/n 2

46. (1−n)a
(b − ax) + C; 47. b − a2 x + C;
− a2
48. 16 sen (6t) + C; 49. − 13 cos (3t) + C;
x 2 πx t
  
50. sen x + C; 51. 2 sen 2 + C; 52. − π cos 2 + C; 53. − 2 cos 2 + C; 54. cos (π − t) + C;
π 1 T 2πx 1
 
55. − sen 2 − x + C; 56. 2 cos (1 − 2t) + C; 57. − 2π cos T − φ0 + C; 58. 6 sen (6x) + C;
1 2 πωt πtω
 
59. a sen (at) + C; 60. πω sen 2 + α + C; 61. sen 2 + α + C; 62. cos (t − x) + C;
63. − cos (t − x) + C; 64. − sen (t − x) + C; 65. sen (t − x) + C; 66. − tan (t − x) + C; 67. cot (t − x) + C;
√ √ √ 6
2 4/5
5 3
3 x − π5

68. tan (t − x) + C; 69. − cot (t − x) + C; 70. − 1 − x2 + C; 71. − 8 1−x + C; 72. 18 + C;
√ 6  √  √ √  √  √ 
3x − π 5 2 1 5 1 x 6 6x 3 2 3

73. 21 3x + 63 π + C; 74. 2 arctan 2 + C; 75. 6 arctan 6 + C; 76. 6 arctan 3 x + C;
√ √ 
77. 15
15 arctan 15
5 x + C; 78. 1
3 sen3 x + C; 79. 1
4 sen4 x + C; 80. − 3
b cos1/3 (bx) + C;

81. 5
4a sen4/5 (ax) + C; 82. 1
7 sen7 x + C; 83. − 3
5 cos5/3 t + C; 84. 1
36 sen6 (6x) + C; 85. 1
7 tan7 x + C;
7/2 4 5/3
86. 2
7 tan x + C; 87. 1
12 tan (3x) + C; 88. 3
5a tan (ax) + C;
89. − cot x + C; 90. − 1
2
1
6 cot6 x + C;
3/2 √
4
cot7/4 (1 − x) + C; 1
cot4 (2x) + C; 4 x
 
91. 7 92. − 8 93. − 3 cot x + C; 94. arcsen 3 + C;
√  √ √  √ √  √
95. arcsen 55 x + C; 96. 3
3
arcsen 23 x + C; 5 5
97. 5 arcsen 3 x + C; 98. x2 + 1 + C;

3 2
2/3 1 x 1 t 1
99. 4 x +1 + C; 100. 2 cos (1 − 2t) + C; 101. 2 − 4 sen (2x) + C; 102. 2 + 4 sen (2t) + C;
2 5/2 3/2 −3
1 2 8 1 1
(x + 1)−2 + C;

103. 2 arctan x + C; 104. (4 − x)
5 − (4 − x) 3 + C; 105. 3 (x + 1) − 2
2
tan3/2 t + C; 1 1 1 1 1
p
106. 3 107. − 2 cos (2z) + C; 108. 2x − 12 sen (6x) + C; 109. 2x + 12 sen (6x) + C;
1 1 1 3 1
110. tan (3x) + C; 111. tan (3x) − x + C; 112. tan (3x) + C; 113. − + C;
3 3 9 16(x4 +1)4
    3/2 √ √ 
x−2 x−3 π π
114. arcsen 2 + C; 115. arcsen √
5
+ C; 116. − 2
9 1 + 3 cos2 x + C; 117. π arctan π x + C;
√ √  √  √ 
118. 48 2 − 3 x − 24 2 − 3 x 3/2 + 2 − 3 x 5/2 − 2 − 3 x 7/2 + C; sen3 (2t) + C;
p
36 6 1 1
5 7 119. 2 sen (2t) − 6
3/2 5/2 7/2
120. 8
375 (5t − 2) + 8
625 (5t − 2) + 2
875 (5t − 2) + C; 121. 1
3(sen3 x−3 sen x−5)
+ C; 122. − 1
4 cot4 t + C;

4
√
123. 1
4 tan t + C; 124. 2
35 5x3 − 6x2 − 48x − 576 6 − x + C; 125. 1
3 cos3 x − cos x + C;
√ √ 
126. 7
7 arctan
7
7 x + C; 127. 1
(t+1)3
− 1
(t+1)2
− 1
4(t+1)4
+ C; 128. 1
6 (2t − 1)3/2 + 1
10 (2t − 1)5/2 + C;
1 6 3 27 5 2 1
129. 2 cos (2 cos x) + C; 130.  3 −  2 −  4 + C; 131. 3(t+1)3
− (t+1)2
− 4(t+1)4
+ C;
3−x1/3 2 3−x1/3 4 3−x1/3

 √ 3/2 p √  √ 5/2  √ 7/2


132. 12 2 − 3 2x − 24 2 − 3 2x − 18
2 − 3 2x 3 3
(1 − x)2/3 153 3 2 9

5 + 7 2− 2x + C; 133. 40 − 8x − 20 x + C;

5/2 3/2
2/3 7/2
3/2 3
134. 12
625 (3 − 5x) − (3 − 5x) −6
125 (3 − 5x) + C; 135. 2
x +1
875 + C; 136. 13 a2 − x2 + a2 x2 + C; 1
2
√ 3/2 5/2
137. 1
4 sec (4t − 1) + C; 1
138. x − 2a cos (2ax) + C; 139. 9 x2 − 3 + 2 x2 − 3 + 15 x2 − 3 + C;
1 2
 1
√ 1 4
 3/2 1 2
3/2
140. arcsen x + C; 4
141. 2 x − 1 + 6 x − 1 + C; 142. 3 1 + sen x + C;
2
1 3/2 1
√ √ √
143. 6 (1 + 2x) − 2 1 + 2x + C; 144. − 2 cos x + C; 145. arcsen (x − 2) + C; 146. 2 sec x + 1 + C;
√ q √ q √ q √
1
arcsen x3 + C; 54
+ 27 3 x + 3 2 − 18 3
x + 3 5 + 14 3 x + 3 8 + C;
   
147. 3 148. 2 tan t − 1 + C; 149. √ 3 √ 5 x+3

150. 3
arctan (2x) − 1
arctan3/2 (2x) + C; 151. 2 cos 1 − t + C; 152. − √
1
− √
4
+ C;
2 3 ( x−2)2 3( x−2)3

153. 72
5 (4 − x)5/3 − 24 (4 − x)2/3 − 27
8 (4 − x)8/3 + 3
11 (4 − x)11/3 + C; 154. 2
a tan (ax) + 2
a sec (ax) − x + C;
1 1 1 1
155. 2 arctan x − 4 sen (2 arctan x) + C; 156. arctan (sen x) + C; 157.
− cot (2x) − csc (2x) + C; 2 2

3
2/3 5/3 8/3
158. − 1
sec (1 − 2t) + sec (1 − 2t) + C;
6
1
2 159. 34 x2 + 1 − x +1 3
+ 16 3
5x2 + 1 2
+ C;
1
arcsen 15 x3 + C; 161. 83 t − 4a
1 1
162. a 2 x
 
160. 3 sen (2at) + 32a sen (4at) + C; 2 sen a + C;
√  √ 
163. arcsen 22 x + C; 164. − 10 3
2 − 5 x 4/3 + C; 165. 1
1+sen2 x
+ C; 166. 1
9 (3x)3/2 − 1
3 (x − 2)3/2 + C;
−2x−5 −x
167. − arctan (cos x) + C; 168. csc (2 − t) − 2
3 csc3 (2 − t) + 1
5 csc5 (2 − t) + C; 169. 2(x+2)2
+ C; 170. 2(x−2)3
+ C;

2
− 12 cos (4x) + 4 + C; 1
cos3 (2 + arcsen (1 − t)) − cos (2 + arcsen (1 − t)) + C;
p
171. √
1−arcsen x
+ C; 172. 173. 3

Última actualización: Julio 2016 Farith J. Briceño N. farith.math@gmail.com


Matemática II - Guía 3. Método de integración: u-sustitución. 60

√   
1
tan4 1 1 2
(x − 1)3 − 6x + 12 arctan x−1
 
174. 2 x + C; 175. − sec x + cos x + C; 176. 3 2 + C;

2 3/2
177. 3x − x + C; 178. 8
7 (2 − sen x)7/4 − 16
3 (2 − sen x)3/4 + C; 179. 1
a ln |ln (sen (ax))| + C;
√ 
1 1 1
(3t + 2)3/2 − 135
8 8
(3t + 2)5/2 + 189
2
(3t + 2)7/2 + C;

180. 8 arctan 2x − 4 + C; 181. 81 182. − 2 cos x + C;
√ √ √ 
183. 1
2 sen x + 1
4 sen (2 sen x) + C; 184. 2 x − 2 2 arctan 2x
2 + C; 185. 23 cos3 t − cos t − 1
5 cos5 t + C;
√  √  √ 
186. − 2 cos x + 4
3 cos3 x − 2
5 cos5 x + C; 187. 1
2 sen (2t) −sen3 (2t) + 10
1
3
1
sen5 (2t) + C;
√ √ 
188. 8
81 (3t + 2)3/2 − 8
135 (3t + 2)5/2 + 2
189 (3t + 2)7/2 + C; 189. 13 x3 − 3x + 3 3 arctan 33 x + C;
√ √ √ 
3x
191. 15 x5 + a4 x − 13 a2 x3 − a7 arctan x 1

190. 2 x − 2 3 arctan 3 + C; a + C; 192. − 9 sen x−3 sen3 x+15
+ C;
 
193. 81 arctan 1−24cos x + C; 194. 12 cos 1 − x2 + C; 195. 13 arctan (3x − 1) + C;


√ √
197. − 12 arcsen 12 − x + C; 1
cos4 (cos (sen x)) + C;

196. 2 arctan 2x − 1 + C; 198. 4 199. 2 sec x + C;
√ √ √ √
200. 3 3 sec x + C; 201. n sec1/n x + C; 202. − 2 csc x + C; 203. − 5 5 csc x + C; 204. − m m csc x + C;
 
205. − 1
3 arcsen (1 − 3x) + C; 206. 3
4 (3 − cos x)4/3 + C; 207. arctan (x + 3) + C; 208. 2x − √11
7
arctan √x7 + C;
√ √
209. 2 1 + sen x + C; 210. arcsen (tan t) + C; 211. 2
3 arcsen3/2 t + C; 212. 1
2 arcsen2 t − 1 − t2 + C;
13/4 9/4 4a2
5/4 1
213. 4
39b3
a + bx3 − 8a
27b3
a + bx3 + 15b3
a + bx3 + C; 214.tan (ωt − π) − t + C;
ω
x2
 
1
tan 2x2 + 3 − 1 2 5
217. 14 x2 − 24
1
sen 6x2 + C;
 
215. 4 2x + C; 216. 2π arctan π + C;
√ √   
1 2 3 3 √x 1

218. − 2 cos x + C; 219. 3 arcsen 2 x + C; 220. arcsen 2π
+ C; 221. sen x+cos x + C;

20x1/3 −6x2/3 −25


222. 2
5 (π − cos (2x))5/4 + C; 223. 1
4x − 1
8 arctan (2x) − 1
3 arctan3/2 (2x) + C; 224.  4 + C;
4 5−x1/3

3x2/3 −9x1/3 +9
225.  3 + C; 226. 2
5 (x + 3)5/2 − 2 (x + 3)3/2 + C; 227. 1
3 sen (3x + π) + C;
3−x1/3
 
228. a arcsen sen
√ x
a
+ C; 229. 16
5 (4 − t)5/2 − 32
3 (4 − t)3/2 − 2
7 (4 − t)7/2 + C; 230. arcsen (1 − 2 cos x) + C;

Bibliografía

1. Purcell, E. - Varberg, D. - Rigdon, S.: “Cálculo”. Novena Edición. PEARSON Prentice Hall.
2. Stewart, J.: “Cálculo”. Grupo Editorial Iberoamericano.
3. Thomas, George: “Cálculo de una variable”. 12ma edición. Pearson.
4. Larson - Hostetler - Edwards, “Cálculo”. Vol. 1. Mc Graw Hill.
5. Leithold, Louis, “El cálculo con geometría analítica”. Harla S.A.

Este material ha sido revisado recientemente, pero esto no garantiza que esté libre de errores, por esa razón se
agradece reportar cualquier error que usted encuentre en este material enviando un mensaje al correo electrónico
farith.math@gmail.com
indicando donde se encuentra(n) dicho(s) error(es). MUCHAS GRACIAS.

Última actualización: Julio 2016 Farith J. Briceño N. farith.math@gmail.com


Matemática II - Guía 4
Notación sigma.

Objetivos a cubrir Código : MAT-CI.4


• Notación sigma. Sumas especiales y telescópicas.
• Principio de Inducción Matemática. Ejercicios resueltos

5
X k
Ejemplo 62 : Desarrolle las siguientes sumas .
k+3
k=1

Solución : Al expandir la suma dada se tiene

k=1 k=2 k=3 k=4 k=5

z }|↓   ↓   ↓   ↓   ↓ 
5
{ z }| { z }| { z }| { z }| {
X k (1)   (2)   (3)   (4)   (5)  1 2 3 4 5
=  + + + + = + + + + .
k+3 (1) + 3 (2) + 3 (3) + 3 (4) + 3 (5) + 3 4 5 6 7 8
k=1

Luego, el desarrollo de la suma es


5
X k 1 2 3 4 5
= + + + + .
k+3 4 5 6 7 8
k=1


1 2 3 4 19
Ejemplo 63 : Exprese en notación sigma la suma dada + + + + ···+ .
2 3 4 5 20
Solución : Observemos que la diferencia entre el numerador y el denominador es igual a 1. Así, podemos
k
escribir el cociente de cada término como , para obtener todos los miembros de la suma dada comenzamos
k+1
la suma desde k = 1 hasta k = 19, luego
19
1 2 3 4 19 X k
+ + + + ··· + = .
2 3 4 5 20 k+1
k=1


n
X
Ejemplo 64 : Hallar la siguiente suma (ai+1 − ai ).
i=1

Solución : Al expandir la suma dada se tiene

i=1 i=2 i=3 i =n−1 i=n

n
X  ↓   ↓   ↓   ↓   ↓ 
z }| { z }| { z }| { z }| { z }| {
(ai+1 − ai ) = a2 − a1 + a3 − a2 + a4 − a3 + · · · + an − an−1 + an+1 − an = an+1 − a1 ,
i=1

por lo tanto,
Término mayor Término menor
evaluado en i = n evaluado en i = 1

n
X ↓ ↓
z }| { z}|{
(ai+1 − ai ) = an+1 − a1 .
i=1
Este tipo de sumas que son diferencias de términos consecutivos se denominan suma telescópica. ⋆

Última actualización: Julio 2016 Farith J. Briceño N. farith.math@gmail.com


Matemática II - Guía 4. Notación sigma. 62

n  
X 1 1
Ejemplo 65 : Hallar la siguiente suma − .
i=7
i+1 i
Solución : Observemos que la suma que queremos calcular cumple con la estructura de las sumas telescópicas,
es decir,
Xn
(ai+1 − ai ) ,
i=1
es la diferencia de dos términos consecutivos, por lo tanto,
Término mayor Término menor
evaluado en i = n evaluado en i = 7

↓ ↓
n   z }| { z}|{
X 1 1 1 1
− = − .
i=7
i + 1 i n + 1 7

n
X 3
Ejemplo 66 : Hallar la siguiente suma .
k (k + 3)
k=1

Solución : Veamos si podemos escribir esta suma como una suma telescópica, para ello, descomponemos la
expresión en sus fracciones simples, es decir,
3 A B
= + ,
k (k + 3) k k+3
donde, A y B son constantes a determinar por medio del método de los coeficientes indeterminados.
3 A (k + 3) + Bk
= =⇒ 3 = A (k + 3) + Bk,
k (k + 3) k (k + 3)
debemos encontrar valores de A y de B para que la igualdad anterior se cumpla. Le damos valores arbitrario
a k para obtener dichas constantes.

• Si k = 0, entonces, 3 = A ((0) + 3) + B (0) =⇒ A = 1.

• Si k = −3, entonces, 3 = A ((−3) + 3) + B (−3) =⇒ B = −1.

Por lo tanto,
n n  
3 1 1 X 3 X 1 1
= − =⇒ = − ,
k (k + 3) k k+3 k (k + 3) k k+3
k=1 k=1

observemos que la nueva forma de escribir la suma nos lleva a la diferencia de dos términos, pero dichos términos
no son consecutivos, por lo tanto, no representa una suma telescópica.
1 1
Si sumamos y restamos los términos y obtenemos
k+1 k+2
n n   X n  
X 3 X 1 1 1 1 1 1 1 1
= − = − + − + −
k (k + 3) k k+3 k k+1 k+1 k+2 k+2 k+3
k=1 k=1 k=1

n   n   X n  
X 1 1 X 1 1 1 1
= − + − + − ,
k k+1 k+1 k+2 k+2 k+3
k=1 k=1 k=1
| {z } | {z } | {z }
↑ ↑ ↑
Diferencia de términos consecutivos, representan sumas telescópicas.

Última actualización: Julio 2016 Farith J. Briceño N. farith.math@gmail.com


Matemática II - Guía 4. Notación sigma. 63

donde,
Término mayor Término menor
evaluado en i = 1 evaluado en i = n

↓ ↓
n   z}|{ z }| {
X 1 1 1 1
− = − ,
k k+1 1 n+1
k=1
similarmente,
n   n  
X 1 1 1 1 X 1 1 1 1
− = − y − = − ,
k+1 k+2 2 n+2 k+2 k+3 3 n+3
k=1 k=1

por lo tanto,
n
X 3 1 1 1 1 1
=1− + − + − ,
k (k + 3) n+1 2 n+2 3 n+3
k=1
es decir,
n
X 3 11 1 1 1
= − − − .
k (k + 3) 6 n+1 n+2 n+3
k=1

n
X
Ejemplo 67 : Hallar la siguiente suma i2 .
i=1

Solución : Es conocido que


3 3
(i + 1) = i3 + 3i2 + 3i + 1 =⇒ (i + 1) − i3 = 3i2 + 3i + 1
sumando, desde i = 1 hasta i = n, la última igualdad se obtiene
Xn   X n
3 
(i + 1) − i3 = 3i2 + 3i + 1 ,
i=1 i=1

donde,
i=1 i=2 i=3 i=4

↓ ↓ ↓ ↓
n 
X  z }| {! z }| {! z }| {! z }| {!
3 3 3 3 3 3 3 3 3 3
(i + 1) − i = (2) − (1) + (3) − (2) + (4) − (3) + (5) − (4)
i=1
| {z }

i=n−1 i=n
Diferencia de términos consecutivos,
representa una suma telescópica. ↓ ↓
z }| {! z }| {!
3 3 3 3 3
+···+ (n − 1) − (n) + (n + 1) − (n) = (n + 1) − 1

mientras que,
Linealidad de la sumatoria Linealidad de la sumatoria
n n n
X X X Sale de la suma por ser constante
(ak + bk ) = ak + bk
k=1 k=1 k=1 respecto al índice de sumación

n n ↓ n ↓ n n n n n
X  ↓ X X X X X X X n (n + 1)
3i2 + 3i + 1 = 3i2 + 3i + 1=3 i2 + 3 i+ 1=3 i2 + 3 + n,
i=1 i=1 i=1 i=1 i=1 i=1 i=1 i=1
2
|{z} | {z }
↑ ↑
Suma especial Suma especial
n n
X n (n + 1) X
i= c = cn
i=1
2 i=1

Última actualización: Julio 2016 Farith J. Briceño N. farith.math@gmail.com


Matemática II - Guía 4. Notación sigma. 64

así,
n   n n
X 3 3
X
2
 3
X n (n + 1)
(i + 1) − i = 3i + 3i + 1 es equivalente a (n + 1) − 1 = 3 i2 + 3 + n,
i=1 i=1 i=1
2

n
X
despejamos i2 y nos queda
i=1

n n  
X
2 n (n + 1)
3
X
2 1 3 3
3 i = (n + 1) − 1 − 3 −n =⇒ i = (n + 1) − n (n + 1) − (n + 1) ,
i=1
2 i=1
3 2

manipulamos el lado derecho de la igualdad,


n    
X
21 3 3 1 2 3n
i = (n + 1) − n (n + 1) − (n + 1) = (n + 1) (n + 1) − −1
i=1
3 2 ↑ 3 2
Factor común n + 1 Factor común n

z }|↓
!   {
2
(n + 1) 2 (n + 1) − 3n − 2 (n + 1) 2n2 + 4n + 2 − 3n − 2 2
(n + 1)  2n + n 
= = =
3 2 3 2 3 2

(n + 1) n (2n + 1) n (n + 1) (2n + 1)
= = .
3 2 6

Finalmente, la suma buscada es


n
X n (n + 1) (2n + 1)
i2 = .
i=1
6

21
X
Ejemplo 68 : Hallar la siguiente suma (2 + i).
i=5

Solución : Es conocido que


n n
X X n (n + 1)
k = kn k = constante y i=
i=1 i=1
2

las cuales son válidas si la suma comienza desde i = 1, observemos que la suma que deseamos calcular comienza
desde i = 5, así, debemos reescribir dicha suma de tal forma que comience desde i = 1, para ello, sumamos y
restamos los cuatro términos que le falta a la suma dada para que comience desde i = 1,
21
X 21
X 4
X 4
X
(2 + i) = (2 + i) + (2 + i) − (2 + i),
i=5 i=5 i=1 i=1
| {z } | {z }
↑ ↑
Suma de los cuatro términos
(2 + (1)) + (2 + (2)) + (2 + (3)) + (2 + (4))

de aquí, !
21
X 4
X 21
X 4
X 21
X 4
X
(2 + i) = (2 + i) + (2 + i) − (2 + i) = (2 + i) − (2 + i) ,
i=5 i=1 i=5 i=1 i=1 i=1

Última actualización: Julio 2016 Farith J. Briceño N. farith.math@gmail.com


Matemática II - Guía 4. Notación sigma. 65

donde,
Linealidad de la sumatoria
n
X Xn n
X
(ak + bk ) = ak + bk
k=1 k=1 k=1

21 ↓ X21 21
X X (21) (22)
(2 + i) = 2+ i = 2 (21) + = 42 + 231 = 273,
i=1 i=1 i=1
2
| {z } |{z}
↑ ↑
Suma especial Suma especial
n n
X X n (n + 1)
c = cn i=
i=1 i=1
2

similarmente,
4 4 4
X X X (4) (5)
(2 + i) = 2+ i = 2 (4) + = 8 + 10 = 18,
i=1 i=1 i=1
2
entonces,
21
X
(2 + i) = 273 − 18 = 255.
i=5


35
X 
Ejemplo 69 : Hallar la siguiente suma 5i − i2 .
i=7

Solución : Es conocido que


n n
X n (n + 1) X n (n + 1) (2n + 1)
i= y i2 = (ver el ejemplo 3)
i=1
2 i=1
6

las cuales son validas si la suma comienza desde i = 1, observemos que la suma que deseamos calcular comienza
desde i = 7, así, debemos reescribir dicha suma de tal forma que comience desde i = 1, para ello, sumamos y
restamos los seis términos que le falta a la suma dada para que comience desde i = 1,
35
X 35 6 6
 X  X  X 
5i − i2 = 5i − i2 + 5i − i2 − 5i − i2 ,
i=7 i=7 i=1 i=1

de aquí,
35 6 35
! 6 35 6
X  X  X  X  X  X 
2 2 2 2 2
5i − i = 5i − i + 5i − i − 5i − i = 5i − i − 5i − i2 ,
i=7 i=1 i=7 i=1 i=1 i=1

donde,

Linealidad de la sumatoria
n
X Xn n
X
(ak + bk ) = ak + bk
k=1 k=1 k=1

35 35 35 35 35
X  ↓ X X X X (35) (36) (35) (36) (71)
5i − i2 = 5i − i2 = 5 i− i2 = 5 − = 3150 − 14910 = −11760,
i=1 i=1 ↑ i=1 i=1 i=1
2 6
|{z} | {z }
Linealidad de la sumatoria ↑ ↑
Sale de la suma por ser constante
Suma especial Suma especial
respecto al índice de sumación n n
X n (n + 1) X 2 n (n + 1) (2n + 1)
i= i =
i=1
2 i=1
6

Última actualización: Julio 2016 Farith J. Briceño N. farith.math@gmail.com


Matemática II - Guía 4. Notación sigma. 66

similarmente,
6 6 6 6 6
X  X X X X (6) (7) (6) (7) (13)
5i − i2 = 5i − i2 = 5 i− i2 = 5 − = 105 − 91 = 14,
i=1 i=1 i=1 i=1 i=1
2 6
entonces,
35
X 
5i − i2 = −11760 − 14 = −11774.
i=7

Ejemplo 70 : Obtenga el siguiente límite, si existe,
n  2
X 1 i
lim .
n→∞
i=1
n n
Solución : En primer lugar, manipulamos algebraicamente la sumatoria
n  2 X n n n
X 1 i 1 i2 X 1 2 1 X 2 1 n (n + 1) (2n + 1) (n + 1) (2n + 1)
= 2
= 3
i = 3
i = 3 = ,
i=1
n n i=1
n n i=1
n n i=1
n 6 6n2
↑ | {z }
Linealidad de la sumatoria ↑
Sale de la suma por ser constante Suma especial
respecto al índice de sumación n
n (n + 1) (2n + 1)
i2 =
X

i=1
6

entonces,
n  2
X 1 i (n + 1) (2n + 1) 2n2 + 3n + 1 L′ H 4n + 3 L′ H 4 1
lim = lim 2
= lim 2
= lim = lim = .
n→∞
i=1
n n n→∞ 6n n→∞
| {z 6n }
n→∞ 12n
| {z }
n→∞ 12 3
↑ ↑
∞ ∞
Indeterminación Indeterminación
∞ ∞

Finalmente,
n  2
X 1 i 1
lim = .
n→∞
i=1
n n 3

n  2

X i 1
Ejemplo 71 : Obtenga el siguiente límite, si existe, lim 4− .
n→∞
i=1
n2 n
Solución : En primer lugar, manipulamos algebraicamente la sumatoria

Suma especial Suma especial


n n
Linealidad de la sumatoria
X X 2 n (n + 1) (2n + 1)
c = cn i =
n
X Xn n
X i=1 i=1
6
(ak + bk ) = ak + bk
k=1 k=1 k=1 ↓
z }| ↓
! { z }| {
n   n   n n n n
X i2 1 1 X i2 ↓ 1 X X i2 1 X 1 X 2
4− = 4− = 4− =  4− 2 i 
i=1
n2 n n i=1
n2 n i=1 i=1
n2 n i=1 n i=1
↑ ↑
Linealidad de la sumatoria Linealidad de la sumatoria
Sale de la suma por ser constante Sale de la suma por ser constante
respecto al índice de sumación respecto al índice de sumación

 
1 1 n (n + 1) (2n + 1) (n + 1) (2n + 1)
= 4 (n) − 2 =4− ,
n n 6 6n2

Última actualización: Julio 2016 Farith J. Briceño N. farith.math@gmail.com


Matemática II - Guía 4. Notación sigma. 67

entonces,
n    
X i2 1 (n + 1) (2n + 1) (n + 1) (2n + 1) 1 11
lim 4− 2 = lim 4 − 2
= lim 4 − lim 2
=4− = .
n→∞
i=1
n n n→∞ 6n n→∞ n→∞
| {z6n } 3 3


Indeterminación

Luego,
n  
X i2 1 11
lim 4− 2 = .
n→∞
i=1
n n 3

n (n + 1)
Ejemplo 72 : Demuestre que .
1 + 2 + 3 + 4 + ···+ n =
2
Demostración : Usando inducción matemática, demostremos, en primer lugar, que la igualdad se cumple
para n = 1, así,
? (1) ((1) + 1) ? (1) (2)
1= =⇒ 1= =⇒ 1 = 1 se cumple
2 2
Hipótesis inductiva : Supongamos que se cumple para n = h, es decir, la siguiente igualdad es cierta
h (h + 1)
1 + 2 + 3 + 4 + ··· + h =
.
2
Tesis inductiva : Demostremos que se cumple la igualdad para n = h + 1, es decir, debemos verificar que
la siguiente igualdad es cierta
Nuevo término en la suma


z }| { ? (h + 1) ((h + 1) + 1)
1 + 2 + 3 + 4 + · · · + h + (h + 1) = .
2
así, por hipótesis inductiva
Hipótesis Inductiva


z }| { h (h + 1) h (h + 1) + 2 (h + 1)
1 + 2 + 3 + 4 + · · · + h + (h + 1) = + (h + 1) =
2 2

Factor común h + 1

↓ (h + 1) (h + 2) (h + 1) ((h + 1) + 1)
= = ,
2 2
por lo tanto,
(h + 1) ((h + 1) + 1)
1 + 2 + 3 + 4 + · · · + h + (h + 1) = se cumple
2
entonces, queda demostrado que
n (n + 1)
1 + 2 + 3 + 4 + ··· + n = .
2

1 1 1 1 n
Ejemplo 73 : Demuestre que + + + ··· + = .
1 · 4 4 · 7 7 · 10 (3n − 2) (3n + 1) 3n + 1
Demostración : Usando inducción matemática, demostremos, en primer lugar, que la igualdad se cumple
para n = 1, así,
1 ? (1) 1 ? 1 1 1
= =⇒ = =⇒ = se cumple
(3 (1) − 2) (3 (1) + 1) 3 (1) + 1 1·4 4 4 4

Última actualización: Julio 2016 Farith J. Briceño N. farith.math@gmail.com


Matemática II - Guía 4. Notación sigma. 68

Hipótesis inductiva : Supongamos que se cumple para n = h, es decir, la siguiente igualdad es cierta
1 1 1 1 h
+ + + ··· + = .
1 · 4 4 · 7 7 · 10 (3h − 2) (3h + 1) 3h + 1
Tesis inductiva : Demostremos que se cumple la igualdad para n = h + 1, es decir, debemos verificar que
la siguiente igualdad es cierta

Nuevo término en la suma

z ↓
}| {
1 1 1 1 1 ? (h + 1)
+ + + ···+ + = .
1 · 4 4 · 7 7 · 10 (3h − 2) (3h + 1) (3 (h + 1) − 2) (3 (h + 1) + 1) 3 (h + 1) + 1
así, por hipótesis inductiva

Hipótesis Inductiva


z }| {
1 1 1 1 1
+ + + ···+ +
1 · 4 4 · 7 7 · 10 (3h − 2) (3h + 1) (3 (h + 1) − 2) (3 (h + 1) + 1)

h 1 h 1
= + = +
3h + 1 (3 (h + 1) − 2) (3 (h + 1) + 1) 3h + 1 (3h + 1) (3 (h + 1) + 1)

h (3 (h + 1) + 1) + 1 h (3h + 4) + 1 3h2 + 4h + 1
= = =
(3h + 1) (3 (h + 1) + 1) (3h + 1) (3 (h + 1) + 1) (3h + 1) (3 (h + 1) + 1)

(h + 1) (3h + 1) (h + 1)
= = ,
(3h + 1) (3 (h + 1) + 1) 3 (h + 1) + 1
por lo tanto,
1 1 1 1 1 (h + 1)
+ + + ···+ + = ,
1 · 4 4 · 7 7 · 10 (3h − 2) (3h + 1) (3 (h + 1) − 2) (3 (h + 1) + 1) 3 (h + 1) + 1
se cumple la igualdad, entonces, queda demostrado que
1 1 1 1 n
+ + + ··· + = .
1 · 4 4 · 7 7 · 10 (3n − 2) (3n + 1) 3n + 1

Ejercicios

1. Desarrolle las siguientes sumas


9 5 6 11 10
X X k X (−1)k+1 X k2 X √
1. (i + 2)2 2. 3. 4. 5. n
i=1
k+3 k 2k − 3 n=1
k=1 k=1 k=4

8 6 10 7 8
X i X
−n
X i X (−1)j X 3m − 1
6. 7. 2 8. 9. 10.
i=2
i−1 n=1 i=4
i2 − 1 j=3
j − j3 m=2
m2 − 2

2. Exprese en notación sigma la suma dada


√ √ √ √ √ 1 2 3 4 19
1. 1 + 2 + 3 + 4 + 5 + · · · + 10 2. 3+ 4+ 5+ 6+ 7 3. + + + + ···+
2 3 4 5 20
1 1 1 1 n 1 2 4 32
4. + + + ···+ 5. 1 − x + x2 − x3 + · · · + (−1) xn 6. + + + ···+
7 9 11 49 3 9 27 729

Última actualización: Julio 2016 Farith J. Briceño N. farith.math@gmail.com


Matemática II - Guía 4. Notación sigma. 69

3 4 5 6 23
7. + + + + ··· + 8. 2 + 4 + 6 + 8 + · · · + 2n 9. 1 + 3 + 5 + 7 + · · · + (2n − 1)
7 8 9 10 27
1 1 1 1 1
10. 1 + + + + + 11. 1 + 2 + 4 + 8 + 16 + 32 12. x + x2 + x3 + x4 + · · · + xn
4 9 16 25 36
3. Hallar las siguientes sumas
n
X n
X n
X n
X
1. i 2. i2 3. i3 4. i4
i=1 i=1 i=1 i=1

4. Hallar las siguientes sumas usando los resultados obtenidos en el ejercicio 3


n n n n   12
X X  X 3
X 3 X
1. (3i − 2) 2. 2
2i + i 3. (2 − i) 4. 2
i − 5. (3 − 2i)2
i=1 i=1 i=1 i=1
2 i=3

n 30 9  2  n
X 2 X 3
X i X 2
6. 4i − i2 7. (i − 3) 8. −i 9. 2 − i2 − 3i
i=7 i=10 i=3
5 i=5

5. Calcular las siguientes sumas


n   n   n n
X 1 1 X 1 1 X 1 X 1
1. − 2. − 3. 4.
i=2
i−1 i i=1
2i − 1 2i + 1 j=1
j (j + 1) i=1
i (i + 3)

n n n n
X 1 X 3 X 1 X 1
5. 2
6. 7. 8.
i=7
i +i k (k + 3) (2k − 1) (2k + 5) (k + 1) (k + 2)
k=1 k=1 k=1

n n n n
X 2 X 1 X 1 X 2
9. 2
10. 2
11. 2
12.
i=2
i −1 i=3
2i − 6i + 4 j=1
4j − 1 i=1
i2 + 4i + 3

n n n
X 1 X 1 X 2
13. 2 − 3m − 2
14. 2 + 7m + 12
15.
m=1
9m m=1
m k (k + 1) (k + 2)
k=1

n n n
X 2i + 1 X 1 − 4i2 − 12i X 6
16. 2 17. 2 2 18.
i=1 i2 (i + 1) i=3 (2i − 1) (2i + 5) k (k + 1) (k + 2) (k + 3)
k=1

n n √
X 6k X √ 
19. 20. k− k−1
k − 5k 2 + 4
4
k=3 k=1

6. Calcular el límite cuando n tiende a infinito en las sumas obtenidas en el ejercicio 5.


10
X 7
X
7. Demuestre que (2k − 5) y (2j + 1) son iguales.
k=3 j=0

8. Obtenga los límites indicados, si existen.


n  2 n   n  3 !
X 1 i X i2 1 X 1 i
1. lim 2. lim 4− 2 3. lim +1
n→∞
i=1
n n n→∞
i=1
n n n→∞
i=1
n n

n  3  ! n  3  !
X 3 3i 3i X 2 2i 2i
4. lim 1+ −2 1+ 5. lim +5
n→∞
i=1
n n n n→∞
i=1
n n n

Última actualización: Julio 2016 Farith J. Briceño N. farith.math@gmail.com


Matemática II - Guía 4. Notación sigma. 70

9. Demostrar que
n
X
(F (i + 1) − F (i − 1)) = F (n + 1) + F (n) − F (1) − F (0) .
i=1

10. Considere el cociente


1 2 + 2 2 + 3 2 + 4 2 + · · · + n2
.
1 + 2 + 3 + ···n
Hallar el valor del cociente para cualquier entero positivo n.
11. Demuestre que
n 
X 
(i + 1)3 − (i − 1)3 = (n + 1)3 + n3 − 1.
i=1

n (n + 1)
12. Demuestre que 1 + 2 + 3 + 4 + · · · + n = .
2
n (n + 1) (2n + 1)
13. Demuestre que la suma de los cuadrados de los n primeros números naturales es igual a .
6
n−1 n−1 n (n + 1)
14. Demuestre que 1 − 22 + 32 − 42 + · · · + (−1) n2 = (−1) .
2
 2
n (n + 1)
15. Demuestre que la suma de los cubos de los n primeros números naturales es igual a .
2
n (n + 1) (n + 2) (n + 3)
16. Demuestre que 1 · 2 · 3 + 2 · 3 · 4 + 3 · 4 · 5 + · · · + n · (n + 1) · (n + 2) = .
4
1 1 1 1 n
17. Demuestre que + + + ··· + = .
1·3 3·5 5·7 (2n − 1) (2n + 1) 2n + 1
12 22 32 n2 n (n + 1)
18. Demuestre que + + + ··· + = .
1·3 3·5 5·7 (2n − 1) (2n + 1) 2 (2n + 1)
1 1 1 1 n
19. Demuestre que + + + ···+ = .
1 · 4 4 · 7 7 · 10 (3n − 2) (3n + 1) 3n + 1
1 1 1 1 n
20. Demuestre que + + + ···+ = .
a (a + 1) (a + 1) (a + 2) (a + 2) (a + 3) (a + n − 1) (a + n) a (a + n)
21. Demuestre que si u0 = 2 y u1 = 3 y si uk+1 = 3uk − 2uk−1 , para todo número natural k, se tiene

un = 2n + 1.

22. Demuestre que si


α2 − β 2 α3 − β 3
u1 = y u2 =
α−β α−β
y si
uk = (α + β) uk−1 − αβuk−2 ,
para todo número natural k > 2, se tiene

αn+1 − β n+1
un = .
α−β

23. Demuestre que la suma


An = 11n+2 + 122n+1
es divisible por 133 cualquiera que sea el número entero n ≥ 0.

Última actualización: Julio 2016 Farith J. Briceño N. farith.math@gmail.com


Matemática II - Guía 4. Notación sigma. 71

24. Demuestre que n2 − n es divisible por 6.



n (n + 1) 6n3 + 9n2 + n − 1
4 4 4 4 4
25. Demuestre que 1 + 2 + 3 + 4 + · · · n = .
30
26. Determine el primer entero N para el cual sea verdadera la proposición para cada n ≥ N y luego
demuestre la proposicón para cada n ≥ N .
1. 3n + 25 < 3n 2. 2n > 2n + 1 3. n2 ≤ 2n

Respuestas: Ejercicios

1 2
1.1. 9 + 16 + 25 + 36 + 49 + 81 + 100 + 121; 1.2. + 36 + 47 + 58 ;
4 + 5 1.3. 1 − 21 + 13 − 14 + 51 − 16 ;
16 25 49 64 27 100 121
√ √ √ √ √ √ √
1.4. 5 + 7 +4+ 11 + 13 + 5 + 17 + 19 ; 1.5. 1 + 2 + 3 + 2 + 5 + 6 + 7 + 8 + 3 + 10;
3 4 5 6 7 8 1 1 1 1 1 1 4 5 6 7 8 9 10
1.6. 2 + 2 + 3 + 4 + 5 + 6 + 7; 1.7. 2 + 4 + 8 + 16 + 32 + 64 ; 1.8. 15 + 24 + 35 + 48 + 63 + 80 + 99 ;
10 9 √
1 1 1 1 1 5 8 11 14 1 20 23
P P
1.9. 24 + 60 + 120 + 210 + 336 ; 1.10. 2 + 7 + 14 + 23 + 2 + 47 + 62 ; 2.1. k; 2.2. k;
k=1 k=3

19 25 n+1 6 27
2n−1 n−4
n 1
(−1)k−1 xk−1 ;
P P P P P
2.3. n+1 ; 2.4. 2n−1 ; 2.5. 2.6. 3n ; 2.7. n ;
n=1 n=4 k=1 n=1 n=7

n n 6 5 n
n(n+1)
1
2n ; xk ;
P P P P P
2.8. 2k; 2.9. (2i − 1) ; 2.10. n2
; 2.11. 2.12. 3.1. 2 ;
k=1 i=1 n=1 n=0 k=1

n(n+1) 6n3 +9n2 +n−1


 
 2
n(n+1)(2n+1) n(n+1) n(3n−1) n(4n+5)(n+1)
3.2. 6 ; 3.3. 2 ; 3.4. 30 ; 4.1. 2 ; 4.2. 6 ;

n(n−3) n2 −3n+4 n 3n−2n2 −6


   

(n − 6) 6n4 − 9n3 − 4n2 + 156n + 1015 ; 1



4.3. − 4 ; 4.4. 6 ; 4.5. 1770; 4.6. 30

4.9. 15 (n − 4) n4 + 14n3 + 81n2 + 314n + 1250 ; 1 1



4.7. 142 443; 4.8. 14; 5.1. 1 − n ; 5.2. 1 − 2n+1 ;
 
1 1 11 1 1 1 1 1 11 1 1 1
5.3. 1 + n+1 ; 5.4. 3 6 − n+1 − n+2 − n+3 ; 5.5. 7 − n+1 ; 5.6. 6 − n+1 − n+2 − n+3 ;
     
5.7. 16 23 1 1 1
15 − 2n+1 − 2n+3 − 2n+5 ; 5.8. 12 − n+2
1
; 5.9. 1 − n+11
; 5.10. 12 1 − n−1
1
; 5.11. 12 1 − 2n+11
;
 
5.12. 65 − n+21 1
− n+3 ; 5.13. 31 1 − 3n+1
1
; 5.14. 14 − n+4
1
; 5.15. 12 − n+1
1 1
+ n+2 ; 1
5.16. 1 − (n+1) 2;
   
1 143 1 1 1 1 7219 1 1 1 1 1 2 1
5.17. − 6 315 − 2n+1 − 2n+3 − 2n+5 − 6 99 225 − (2n+1)2
− (2n+3)2
− (2n+5)2
; 5.18. 3 − n+1 + n+2 − n+3 ;

1 1 1
√ 11 1 11
5.19. 2 − n−1 + n+2 ; 5.20. 1 − n − 1; 6.1. 1; 6.2. 1; 6.3. 1; 6.4. 18 ; 6.5. 7; 6.6. 6 ;
23 1 1 1 5 1 1 1
6.7. 90 ; 6.8. 2; 6.9. 1; 6.10. 2; 6.11. 2; 6.12. 6; 6.13. 3; 6.14. 4; 6.15. 2;
26 132 1 1 1 11 5
6.16. 1; 6.17. − 297 675 ; 6.18. 3; 6.19. 2; 6.20. − ∞; 8.1. 3; 8.2. 3 ; 8.3. 4;
195 2n+1
8.4. 4 ; 8.5. 14; 10. 3 ;

Bibliografía

1. Purcell, E. - Varberg, D. - Rigdon, S.: “Cálculo”. Novena Edición. PEARSON Prentice Hall.
2. Stewart, J.: “Cálculo”. Grupo Editorial Iberoamericano.
3. Thomas, George: “Cálculo de una variable”. 12ma edición. Pearson.
4. Larson - Hostetler - Edwards, “Cálculo”. Vol. 1. Mc Graw Hill.
5. Leithold, Louis, “El cálculo con geometría analítica”. Harla S.A.

Este material ha sido revisado recientemente, pero esto no garantiza que esté libre de errores, por esa razón se
agradece reportar cualquier error que usted encuentre en este material enviando un mensaje al correo electrónico
farith.math@gmail.com
indicando donde se encuentra(n) dicho(s) error(es). MUCHAS GRACIAS.

Última actualización: Julio 2016 Farith J. Briceño N. farith.math@gmail.com


Matemática II - Guía 4. Notación sigma. 72

Última actualización: Julio 2016 Farith J. Briceño N. farith.math@gmail.com


Matemática II - Guía 5
Integral definida.

Objetivos a cubrir Código : MAT-CI.5


• Introducción al calculo de área debajo de una curva.
• Integral definida. Propiedades de la integral definida.
• Primer Teorema Fundamental del Cálculo. Ejercicios resueltos

Ejemplo 74 : Calcule el área aproximada de la región limitada por la función f (x) = 2x − 1, el intervalo
[2, 5] y el eje x, usando una partición regular de 8 subintervalos y polígonos circunscritos.
Solución : La grafica de la región dada es

0 1 2 3 4 5

Región limitada por f (x) = 2x − 1,


las rectas x = 2, x = 5 y el eje x.

Por utilizar particiones regulares se tiene que todos los subintervalos tienen la misma longitud, así,
5−2 3
∆xi = =
8 8
con i = 1, 2, 3, . . . , 8. Tenemos que la partición del intervalo es


 x0 = 2 =⇒ x0 = 2




 3 3
 x =2+ =⇒ x1 = 2 +
 1



 8 8



 3 3 3

 x2 = 2 + + =⇒ x2 = 2 + 2


 8 8 8



 3 3 3 3


 x3 = 2 + + + =⇒ x3 = 2 + 3


 8 8 8 8


 3 3 3 3 3 3
x4 = 2 + + + + =⇒ x4 = 2 + 4 =⇒ xi = 2 + i
 8 8 8 8 8 8



 3 3 3 3 3 3


 x5 = 2 + + + + + =⇒ x5 = 2 + 5


 8 8 8 8 8 8



 3 3 3 3 3 3 3

 x6 = 2 + + + + + + =⇒ x6 = 2 + 6


 8 8 8 8 8 8 8



 3 3 3 3 3 3 3 3


 x7 = 2 + + + + + + + =⇒ x7 = 2 + 7


 8 8 8 8 8 8 8 8



 3 3
 x8 = 2 + + + + + + + + 3 3 3 3 3 3 3
=⇒ x8 = 2 + 8 = 5
8 8 8 8 8 8 8 8 8

Última actualización: Julio 2016 Farith J. Briceño N. farith.math@gmail.com


Matemática II - Guía 5. Integral definida. 74

es decir,
u u u u u u u u u
x0 = 2 x1 x2 x3 x4 x5 x6 x7 x8 = 5

Partición regular del intervalo [2, 5]


de aquí,

Área del i-ésimo


8 polígono
b

4
Altura = f (xi )

0 1 2 5 xi−1 xi

Polígonos circunscritos
Base = ∆xi
entonces,
8 8  
X X 3i
A≈ f (xi ) ∆xi = f 2+ ∆xi ,
i=1 i=1
8
como f ( ) = 2 ( ) − 1, se tiene que
   
3i 3i 6i 3i
f 1+ =2 2+ −1=4+ −1 = 3+
8 8 8 4
así,

Linealidad de la sumatoria Linealidad de la sumatoria


Sale de la suma por ser constante Sale de la suma por ser constante
respecto al índice de sumación respecto al índice de sumación

8   8  ↓ 8   8 8
↓ !
X 3i X 3i 3 3 X 3i 3 X X 3
A≈ f 2+ ∆xi = 3+ = 3+ = 3+ i
i=1
8 i=1
4 8 8 i=1
4 ↑ 8 i=1 i=1
4
Linealidad de la sumatoria
n
X Xn n
X
(ak + bk ) = ak + bk
k=1 k=1 k=1

 
8 8  
3 X 3 X  3 3 8 (8 + 1) 3 153
=  3 + i = 3 (8) + = (24 + 27) =
8 i=1 4 i=1 8 4 2 8 8
| {z } |{z}
↑ ↑
Suma especial Suma especial
n n
X X n (n + 1)
c = cn i=
i=1 i=1
2

luego, el área aproximada es


153
A≈ .
8

Última actualización: Julio 2016 Farith J. Briceño N. farith.math@gmail.com


Matemática II - Guía 5. Integral definida. 75

Ejemplo 75 : Calcule el área aproximada de la región limitada por la función f (x) = x2 + 2, el inter-
valo [1, 7] y el eje x, usando una partición irregular cuyas longitudes de los subintervalos vienen dada por
{0.2, 1, 0.5, 2.2, 1.3, 0.8} y polígonos inscritos.
Solución : La grafica de la región dada es

51

0 1 2 3 4 5 6 7

Región limitada por f (x) = x2 + 2,


las rectas x = 1, x = 7 y el eje x.
La longitud de cada subintervalo es
∆x1 = 0.2; ∆x2 = 1; ∆x3 = 0.5; ∆x4 = 2.2; ∆x5 = 1.3; ∆x6 = 0.8;
Tenemos que, la partición del intervalo es

 x0 = 1 =⇒ x0 = 1






 x1 = 1 + 0.2 =⇒ x1 = 1.2






 x2 = 1 + 0.2 + 1 =⇒ x2 = 2.2


x3 = 1 + 0.2 + 1 + 0.5 =⇒ x3 = 2.7




 x4 = 1 + 0.2 + 1 + 0.5 + 2.2 =⇒ x4 = 4.9








 x5 = 1 + 0.2 + 1 + 0.5 + 2.2 + 1.3 =⇒ x5 = 6.2



x6 = 1 + 0.2 + 1 + 0.5 + 2.2 + 1.3 + 0.8 =⇒ x6 = 7
es decir,
x1 x2 x3 x4 x5
b b b b b b b

x0 = 1 x6 = 7
Partición irregular del intervalo [1, 7]
de aquí,

51
Área del i-ésimo
Polígonos inscritos polígono

2 Altura = f (xi−1 )

0 1 7 xi−1 xi

Base = ∆xi

Última actualización: Julio 2016 Farith J. Briceño N. farith.math@gmail.com


Matemática II - Guía 5. Integral definida. 76

entonces,
6
X
A≈ f (xi−1 ) ∆xi = f (x0 ) ∆x1 + f (x2 ) ∆x2 + f (x2 ) ∆x3 + f (x3 ) ∆x4 + f (x4 ) ∆x5 + f (x5 ) ∆x6 ,
i=1

donde,  2

 f (x0 ) = f (1) = (1) + 2 = 3 =⇒ f (x0 ) = 3



 2


 f (x1 ) = f (1.2) = (1.2) + 2 = 3.44 =⇒ f (x1 ) = 3.44




 f (x2 ) = f (2.2) = (2.2)2 + 2 = 6.84
 =⇒ f (x2 ) = 6.84
 2


 f (x3 ) = f (2.7) = (2.7) + 2 = 9.29 =⇒ f (x3 ) = 9.29



 2


 f (x4 ) = f (4.9) = (4.9) + 2 = 26.01 =⇒ f (x4 ) = 26.01



 2
f (x5 ) = f (6.2) = (6.2) + 2 = 40.44 =⇒ f (x5 ) = 40.44
así,
A ≈ (3) (0.2) + (3.44) (1) + (6.84) (0.5) + (9.29) (2.2) + (26.01) (1.3) + (40.44) (0.8) = 94.063,
luego, el área aproximada es A ≈ 94.063. ⋆

Ejemplo 76 : Hallar el área de la región limitada por la gráfica de la función f (x) = 4 − x2 , la recta vertical
x = 1 y los ejes coordenados.

Solución : La grafica de la región dada es

Utilizamos una partición regular de n subintervalos, por lo


4 tanto, todos los subintervalos tienen la misma longitud
1−0 1
∆xi = = ,
n n
con i = 1, 2, 3, . . . , n.

Tenemos que la partición del intervalo es



 x0 = 0 =⇒ x0 = 0




 1 1
0
x1 = 0 + =⇒ x1 =
1  n n


Región limitada por f (x) = 4 − x2 ,  x =0+ 1 + 1

 1
2 =⇒ x2 = 2
las rectas x = 1 y los ejes coordenados. n n n

1 1 1 1


 x3 = 0 + + + =⇒ x3 = 3


 n n n n



 ..


 .





 1 1 1 1
 xi−1 = 0 +
 + + ···+ =⇒ xi−1 = (i − 1)
n n n n

 1 1 1 1
 xi = 0 + n + n + · · · + n =⇒ xi = i




 n



 ..



 .


 xn = 0 + + + + 1 + 1 + · · · + 1
1 1 1 1


=⇒ xn = n = 1,
n n n n n n n

Última actualización: Julio 2016 Farith J. Briceño N. farith.math@gmail.com


Matemática II - Guía 5. Integral definida. 77

de aquí,
Área del i-ésimo
4 polígono
b

Altura = f (xi )

0 xi−1 xi 1

Base = ∆xi
entonces,
n n   n  
X X i X i 1
A = lim f (xi ) ∆xi = lim f ∆xi = lim f ,
n→∞
i=1
n→∞
i=1
n n→∞
i=1
n n
donde,
   2
i i i2
f =4− = 4 − 2,
n n n
así,
n   n n
! n n
!
X i2 1 1 X X i2 1 X 1 X 2
A = lim 4− 2 = lim 4− = lim 4− 2 i
n→∞
i=1
n n n→∞ n
i=1 i=1
n2 n→∞ n
i=1
n i=1
   
1 1 n (n + 1) (2n + 1) (n + 1) (2n + 1) 1 11
= lim 4 (n) − 2 = lim 4 − 2
=4− =
n→∞ n n 6 n→∞ 6n 3 3
luego, el área es
11
A= .
3

3
Ejemplo 77 : Hallar el área de la región limitada por las gráficas de las funciones f (x) = x , g (x) = −x y
x = 1.
Solución : La grafica de la región dada es

f (x) = x3
1

0 1

−1
g (x) = −x
Región limitada por f (x) = x3 ,
g (x) = −x y x = 1.

Última actualización: Julio 2016 Farith J. Briceño N. farith.math@gmail.com


Matemática II - Guía 5. Integral definida. 78

Utilizamos una partición regular de n subintervalos, por lo tanto, todos los subintervalos tienen la misma
longitud
1−0 1
∆xi = =
n n
con i = 1, 2, 3, . . . , n. Tenemos que la partición del intervalo es


 x0 = 0 =⇒ x0 = 0




 1 1

 x1 = 0 + =⇒ x1 =


 n n




 1 1 1

 x2 = 0 + + =⇒ x2 = 2


 n n n
..
 .



 1 1 1 1


 xi = 0 + + + · · · + =⇒ xi = i


 n n n n

 ..



 .


 xn = 0 + 1 + 1 + 1 + 1 + 1 + · · · + 1 1


=⇒ xn = n = 1,
n n n n n n n
de aquí,

f (x) = x3
1

0 Altura = f (xi ) − g (xi )

−1 Base = ∆xi g (x) = −x

Área del i-ésimo polígono

Observemos que el área del rectángulo representativo que aparece en la grafica anterior viene dada por
    
i i 1
Ai = (Altura) (Base) = (f (xi ) − g (xi )) (∆xi ) = f −g
n n n
 3  !  
i i 1 1 i3 i
= − − = +
n n n n n3 n

con i = 1, 2, 3, . . . , n, entonces,
n n   n   n n
!
X X 1 i3 i 1 X i3 i 1 X i3 X i
A = lim Ai = lim 3
+ = lim 3
+ = lim 3
+
n→∞
i=1
n→∞
i=1
n n n n→∞ n
i=1
n n n→∞ n
i=1
n i=1
n

n n
! !
2
1 1 X 3 1 X 1 n2 (n + 1) 1 n (n + 1)
= lim i + i = lim + 2
n→∞ n n3 i=1 n i=1 n→∞ n4 4 n 2
!
2
(n + 1) (n + 1) 1 1 3
= lim + = + = .
n→∞ 4n2 2n 4 2 4

Última actualización: Julio 2016 Farith J. Briceño N. farith.math@gmail.com


Matemática II - Guía 5. Integral definida. 79

Luego, el área de la región dada es


3
A= .
4

Z 1/2 
Ejemplo 78 : Evalúe 1 − x2 dx, usando la definición de la integral definida.
−1

Solución : Es conocido que


Z b n
X
f (x) dx = lim f (x∗i ) ∆xi ,
a n→∞
i=1

para una partición regular de n subintervalos, x∗i


es un punto cualquiera del subintervalo [xi−1 , xi ] y ∆xi es
la longitud de cada subintervalo, con i = 1, 2, 3, . . . , n, como vamos a considerar una partición regular, tenemos
que
b−a 1/2 − (−1) 3
∆xi = =⇒ ∆xi = = ,
n n 2n
con i = 1, 2, 3, . . . , n. Tenemos que la partición del intervalo es

 x0 = −1 =⇒ x0 = −1



 3 3


 x1 = −1 + =⇒ x1 = −1 +


 2n 2n


 3 3 3
 x2 = −1 + 2n + 2n =⇒ x2 = −1 + 2




 2n


 3 3 3 3
 x = −1 + + + =⇒ x3 = −1 + 3
 3

 2n 2n 2n 2n
..


 .



 3 3 3 3

 xi = −1 + + + ···+ =⇒ xi = −1 + i



 2n 2n 2n 2n



 ..


 .



 3 3 3 3 3 3 3 1
 xn = −1 + 2n + 2n + 2n + 2n + 2n + · · · + 2n =⇒ xn = −1 + n =


2n 2

entonces,
Z 1/2 n n   n  
 X X 3i X 3i 3
1 − x2 dx = lim f (xi ) ∆xi = lim f −1 + ∆xi = lim f −1 + ,
−1 n→∞
i=1
n→∞
i=1
2n n→∞
i=1
2n 2n

donde,
   2  
3i 3i 3i 9i2 3i 9i2
f −1 + = 1 − −1 + =1− 1−2 + 2 = − 2,
2n 2n 2n 4n n 4n
así,
Z n   " n n
#
1/2
2
 X 3i 9i2 3 3 X 3i X 9i2
1−x dx = lim − 2 = lim −
−1 n→∞
i=1
n 4n 2n n→∞ 2n i=1 n i=1
4n2

n
" n
#  
3 3X 9 X 2 3 3 n (n + 1) 9 n (n + 1) (2n + 1)
= lim i− 2 i = lim − 2
n→∞ 2n n 4n i=1 n→∞ 2n n 2 4n 6
i=1
   
3 3 (n + 1) 3 (n + 1) (2n + 1) 9 (n + 1) 9 (n + 1) (2n + 1) 9 9 9
= lim − = lim + 2
= − = .
n→∞ 2n 2 8n n→∞ 4n 16n 4 8 8

Última actualización: Julio 2016 Farith J. Briceño N. farith.math@gmail.com


Matemática II - Guía 5. Integral definida. 80

Luego,
Z 1/2  9
1 − x2 dx = .
−1 8

Z b
b 3 − a3
Ejemplo 79 : Demuestre que x2 dx = .
a 3
Demostración : Consideramos una partición regular de n subintervalos, entonces,
b−a
∆xi = con i = 1, 2, 3, . . . , n.,
n
Tenemos que la partición del intervalo es


 x0 = a =⇒ x0 = a




 b−a b−a

 x1 = a + =⇒ x1 = a +


 n n


 b − a b − a b−a


 x2 = a + + =⇒ x2 = a + 2


 n n n


 b−a b−a b−a b−a
 x =a+ + + =⇒ x3 = a + 3
 3

 n n n n
 .
..





 b−a b−a b−a b−a b−a b−a


 xi = a + + + + + ···+ =⇒ xi = a + i


 n n n n n n



 ..


 .



 b−a b−a b−a b−a b−a b−a b−a

 xn = a + n + n + n + n + n + · · · + n =⇒ xn = a + n =b


n

entonces, puesto que f (x) = x2 , se tiene


Z b n n   n  2
2
X X b−a b−a X b−a b−a
x dx = lim f (xi ) ∆xi = lim f a+i = lim a+i
a n→∞
i=1
n→∞
i=1
n n n→∞
i=1
n n

n  2 n
!
2
b−a X b−a b − a X 2 2a (b − a) (b − a) 2
= lim a+i = lim a + i+ i
n→∞ n i=1 n n→∞ n i=1 n n2

n n n
!
2
b−a X
2
X 2a (b − a) X (b − a) 2
= lim a + i+ i
n→∞ n i=1 i=1
n i=1
n2

n n n
!
2
b−a X 2a (b − a) X
2 (b − a) X 2
= lim a + i+ i
n→∞ n i=1
n i=1
n2 i=1
!
2
b−a 2a (b − a) n (n + 1) (b − a)
2 n (n + 1) (2n + 1)
= lim na + +
n→∞ n n 2 n2 6
!
2
b−a (b − a) (n + 1) (2n + 1)
= lim na2 + a (b − a) (n + 1) +
n→∞ n n 6
!
2
a (b − a) (n + 1) (b − a) (n + 1) (2n + 1)
2
= lim (b − a) a + +
n→∞ n 6n2

Última actualización: Julio 2016 Farith J. Briceño N. farith.math@gmail.com


Matemática II - Guía 5. Integral definida. 81

! !
2 (b − a)2 2 2(b − a)2
= (b − a) a + a (b − a) + = (b − a) a + ab − a +
3 3
   
b2 − 2ab + a2 3ab + b2 − 2ab + a2
= (b − a) ab + = (b − a)
3 3
 2 
b + ab + a2 b 3 − a3
= (b − a) = .
3 3

Luego,
Z b
b 3 − a3
x2 dx = .
a 3

Z x
1
Ejemplo 80 : Considere la expresión f (x) = dt.
1 t
1. Obtenga el intervalo de definición para f (Dominio)
2. Hallar f (1).
3. Hallar los intervalos de crecimiento y decrecimiento de f .
4. Hallar los valores extremos de f .
5. Estudiar la concavidad de f .
6. Esbozar una gráfica para f .

Solución :
1. Observemos que el integrando no está definido en cero, por lo tanto esta integral no existe para ningún
intervalo que incluya al cero, luego, el intervalo de definición es (0, ∞).

Z 1
1
2. f (1) = dt = 0
1 t

3. Derivamos Z x 
′ d 1 1
f (x) = dt =
dx 1 t x
1
y observamos que, para todo x ∈ (0, ∞), se tiene que f ′ (x) = > 0, por lo tanto, la función f es
x
siempre creciente.

4. Por ser una función monótona creciente, no tiene valores extremos.

5. Hallamos la segunda derivada de f y estudiamos su signo


 
′′ d 1 1
f (x) = = − 2,
dx x x
1
y para todo x ∈ (0, ∞), se tiene que f ′′ (x) = 2 < 0, por lo tanto, la función f siempre es concava
x
hacia abajo.

6. Esbozo de la gráfica

Última actualización: Julio 2016 Farith J. Briceño N. farith.math@gmail.com


Matemática II - Guía 5. Integral definida. 82

2
b b
b

Z x
1 1
f (x) = dt
1 t
0 1 2 3 4 5

−1

−2 b

Z xp
Ejemplo 81 : Calcular la derivada de la siguiente función f (x) = 1 + t4 dt.
1

Solución : Tenemos que Z ′


xp √

f (x) = 1+ t4 dt = 1 + x4 .
1 ↑
Primer Teorema
Fundamental del Cálculo

Luego, p
f ′ (x) = 1 + x4 .

Z x
Ejemplo 82 : Calcular la derivada de la siguiente función f (x) = sen4 u tan u du.
0

Solución : Tenemos que


Z x ′
f ′ (x) = sen4 u tan u du = sen4 x tan x.
0 ↑
Primer Teorema
Fundamental del Cálculo

Luego,
f ′ (x) = sen4 x tan x.

Z 4 √ 8
Ejemplo 83 : Calcular la derivada de la siguiente función f (x) = 2+ u du.
x

Solución : Observemos que


Z 4 Z x
√ 8 √ 8
f (x) = 2+ u du = − 2+ u du.
x ↑ 4

Propiedad de la integral
Z a Z b
f (x) dx = − f (x) dx
b a

La propiedad aplicada a la integral es debido a que el Primer Teorema Fundamental del Cálculo exige que el

Última actualización: Julio 2016 Farith J. Briceño N. farith.math@gmail.com


Matemática II - Guía 5. Integral definida. 83

límite variable se encuentre en la cota superior de la integral definida, así,


Z x
√ 8
f (x) = − 2 + u du,
4

derivamos respecto a x,
 Z x ′ Z x ′

√ 8 √ 8 √ 8
f (x) = − 2 + u du =− 2 + u du = − (2 + x) .
4 4 ↑
Primer Teorema
Fundamental del Cálculo

Luego,
√ 8
f ′ (x) = − 2 + x .

Z 2

3
t dt
Ejemplo 84 : Calcular la derivada de la siguiente función f (x) = .
x t2 + 5
Solución : Observemos que √
Z 2 3 Z x √
3
t dt t dt
f (x) = 2
= − 2
.
x t +5 ↑ 2 t +5

Propiedad de la integral
Z a Z b
f (x) dx = − f (x) dx
b a

La propiedad aplicada a la integral es debido a que el Primer Teorema Fundamental del Cálculo exige que el
límite variable se encuentre en la cota superior de la integral definida, así,
Z x √3
t dt
f (x) = − 2+5
,
2 t

derivamos respecto a x,
 Z x

3
′ Z x
√3
′ √
′ t dt t dt 3
x
f (x) = − 2
=− 2
= − 2
.
2 t +5 2 t +5 ↑ x +5
Primer Teorema
Fundamental del Cálculo

Luego, √
x 3

f ′ (x) = − .
x2 + 5

Z √
x
s2
Ejemplo 85 : Calcular la derivada de la siguiente función f (x) = ds.
1 s2 + 1

Solución : Observemos que la función f es la composición de las funciones


Z x
s2 √
g (x) = 2+1
ds y h (x) = x,
1 s
ya que, √
Z x
√  s2
(g ◦ h) (x) = g (h (x)) = g x = ds = f (x) ,
1 s2 + 1

Última actualización: Julio 2016 Farith J. Briceño N. farith.math@gmail.com


Matemática II - Guía 5. Integral definida. 84

por lo tanto, para obtener la derivada de f aplicamos la regla de la cadena

f ′ (x) = [g (h (x))]′ = g ′ (h (x)) h′ (x),


| {z } | {z }
↑ ↑
Derivada de la función externa Derivada de la
evaluada en la función interna función interna

entonces,

Derivada de una
función compuesta


z }| !{
Z √
x 2
√ 2   √
d s ( x) √ ′ x 1 x x
f ′ (x) = ds = √ 2 x = √ = √ = ,
dx 1 s2 + 1 ↑ ( x) + 1 | {z } x + 1 2 x 2 x (x + 1) 2 (x + 1)

Primer Teorema
Fundamental del Cálculo Derivada de la
función interna

Luego, √
′ x
f (x) = .
2 (x + 1)

Z 8
arcsen t
Ejemplo 86 : Calcular la derivada de la siguiente función f (x) = dt.
sen2 x t+5
Solución : Observemos que la función f es la composición de las funciones
Z 8
arcsen t
g (x) = dt y h (x) = sen2 x,
x t+5
ya que, Z 8
 arcsen t
(g ◦ h) (x) = g (h (x)) = g sen2 x = dt = f (x) ,
sen2 x t+5
por lo tanto, para obtener la derivada de f aplicamos la regla de la cadena

f ′ (x) = [g (h (x))] = g ′ (h (x)) h′ (x),
| {z } | {z }
↑ ↑
Derivada de la función externa Derivada de la
evaluada en la función interna función interna

por otra parte, observe que el límite variable está en la cota inferior, así,
Z 8 Z sen2 x
arcsen t arcsen t
f (x) = dt = − dt.
sen2 x t+5 ↑ 8 t+5
Propiedad de la integral
Z a Z b
f (x) dx = − f (x) dx
b a

La propiedad aplicada a la integral es debido a que el Primer Teorema Fundamental del Cálculo exige que el
límite variable se encuentre en la cota superior de la integral definida, así,
Z sen2 x
arcsen t
f (x) = − dt,
8 t+5

Última actualización: Julio 2016 Farith J. Briceño N. farith.math@gmail.com


Matemática II - Guía 5. Integral definida. 85

derivamos respecto a x,

Derivada de una
función compuesta


z }| !{  
Z sen2 x
d arcsen t arcsen sen2 x 2
′ arcsen sen2 x
f ′ (x) = − dt =− sen x = − (2 sen x cos x)
dx 8 t+5 ↑ sen2 x + 5 | {z } sen2 x + 5

Primer Teorema
Fundamental del Cálculo Derivada de la
función interna

Luego, 
′ sen (2x) arcsen sen2 x
f (x) = − .
sen2 x + 5

Z 1 p
Ejemplo 87 : Calcular la derivada de la siguiente función f (x) = x2 u2 + 1 du.
x

Solución : Observemos que


Z 1 p Z 1p Z xp
2 2 2 2 2
f (x) = x u + 1 du = x u + 1 du = − x u2 + 1 du
x ↑ x ↑ 1

Linealidad de la integral Propiedad de la integral


Sale de la integral por ser constante a
Z Z b
f (x) dx = − f (x) dx
respecto a la variable de integración b a

La última propiedad aplicada a la integral es debido a que el Primer Teorema Fundamental del Cálculo exige
que el límite variable se encuentre en la cota superior de la integral definida, así,
Z xp
f (x) = − x2 u2 + 1 du,
1

derivamos respecto a x,
 Z xp ′  Z xp ′
f ′ (x) = − x2 u2 + 1 du = − x2 u2 + 1 du
1 1
| {z }

Derivada de un
producto de funciones

 
Z x p Z x p ′ Z xp p
 2 ′ 2
=−  x 2
u + 1 du + x 2
u + 1 du  = − 2x u2 + 1 du − x2 x2 + 1.
1
| 1 {z } 1


Primer Teorema
Fundamental del Cálculo

Luego, Z x p p
f ′ (x) = − 2x u2 + 1 du − x2 x2 + 1.
1

Última actualización: Julio 2016 Farith J. Briceño N. farith.math@gmail.com


Matemática II - Guía 5. Integral definida. 86

Z x Z t2

1 + u4
Ejemplo 88 : Si F (x) = f (t) dt, donde f (t) = du. Encuentre F ′′ (2).
1 1 u
Solución : Hallemos la primera derivada de F .
Z x  Z x2 √
dF d 1 + u4
= f (t) dt = f (x) = du,
dx dx 1 ↑ 1 u
Primer Teorema
Fundamental del Cálculo

es decir,
Z x2

dF 1 + u4
= du,
dx 1 u
derivamos, de nuevo

Derivada de una
función compuesta


z }|
√ !{ q 4 √ √
2 ′ Z x2 4 1 + (x2 )
d F dF d 1+u 
2 ′ 1 + x8 2 1 + x8
= = du = x = (2x) = ,
dx2 dx dx 1 u ↑ x2 | {z } x2 x

Primer Teorema
Fundamental del Cálculo Derivada de la
función interna

y se tiene que √
′′ 2 1 + x8
F (x) = ,
x
entonces, q
8
2 1 + (2) √
F ′′ (2) = =⇒ F ′′ (2) = 257.
(2)

Ejemplo 89 : Utilice las propiedades de la integral definida para verificar


Z π
2
π2
x cos x dx ≤ .

0
3
Solución : Tenemos, por propiedades de integrales definidas
Z π Z π Z π
2
2

x cos x dx ≤
x cos x dx =
x2 |cos x| dx
0 0 0

como |cos x| ≤ 1, entonces


Z π Z π Z π
2
π3 03 π3
x cos x dx ≤ x2 |cos x| dx ≤ x2 dx = − = ,

0 0 0 ↑ 3 3 3
Ver ejemplo 79
b b3 − a3
Z
x2 dx =
a 3

así, Z
π π2
2
x cos x dx ≤ .

0 3

Última actualización: Julio 2016 Farith J. Briceño N. farith.math@gmail.com


Matemática II - Guía 5. Integral definida. 87

Z x+1
1
Ejemplo 90 : Calcular, si existe, lim √ arcsen u du.
x→0+ x cos x

0
Solución : Observemos que este limite presenta una indeterminación de la forma , así , aplicamos la regla
0
de L’Hospital
Z x+1 ′
Z x+1 arcsen u du
1 cos x
lim √ arcsen u du = lim √ ′
x→0+ x cos x x→0+ [ x]

arcsen (x + 1) + sen x arcsen (cos x)


= lim
x→0+ 1

2 x

= lim 2 x (arcsen (x + 1) + sen x arcsen (cos x))
x→0+
p
= 2 (0) (arcsen ((0) + 1) + sen (0) arcsen (cos (0)))
π π
= 2 (0) (arcsen (1) + (0) arcsen (1)) = 2 (0) + (0) = 0.
2 2
Luego, el límite existe Z x+1
1
lim √ arcsen u du = 0.
x→0+ x cos x

Ejercicios

1. Calcule el área aproximada de la región limitada por la función dada en el intervalo dado y el eje x, usando

(i) Polígonos inscritos (ii) Polígonos circunscritos

(a) f (x) = 2x + 1 en [0, 5], use una partición regular de 8 subintervalos.


(b) f (x) = 16 − x2 en [0, 4], use una partición regular de 8 subintervalos.
(c) f (x) = x3 + 2 en [−1, 2], use una partición regular de 6 subintervalos.
(d) f (x) = 3x − 2 en [1, 5], use una partición irregular cuyas longitudes de los subintervalos vienen
dada por {0.2, 0.8, 0.5, 0.5, 1.2, 0.7, 0.1}.
(e) f (x) = 8 − 2x en [−2, 4], use una partición irregular cuyas longitudes de los subintervalos vienen
dada por {1.3, 0.5, 1, 0.8, 0.2, 2.2}.

2. Calcule el área aproximada de la región limitada por f (x) = x2 − 4x + 5, las rectas verticales x = 0,
x = 3 y el eje x, usando la partición irregular cuyas longitudes de los subintervalos vienen dada por
{0.3, 0.6, 0.5, 0.6, 0.7, 0.3}, tomando como x∗i el punto donde la función alcanza su mínimo en cada
subintervalo.

3. Hallar el área de la región limitada por la gráfica de la función f (x) = 2x + 3, las rectas x = 1 y x = 2
y el eje x.
2
4. Hallar el área de la región limitada por la gráfica de la función y = (x − 1) + 2, las rectas x = 2 y
x = 4 que se encuentra en el primer cuadrante.
5. Hallar el área de la región limitada por las gráficas de la función f (x) = x2 , y la recta y = x + 2.
6. Hallar el área de la región limitada por las gráficas de las funciones y = x2 y y + x2 = 2.

Última actualización: Julio 2016 Farith J. Briceño N. farith.math@gmail.com


Matemática II - Guía 5. Integral definida. 88


7. Hallar el área de la región limitada por la gráfica de la función y = x el eje y y y = 2.
8. Hallar el área de la región limitada por las gráficas de las funciones f (x) = 1 − x2 − 2x, f (x) = x2 + 2
el eje de las ordenadas y la recta x = 3.
9. Hallar el área de la región limitada por la gráfica de la función x + 4 − y = 0, definida en el intervalo
[−1, 1] y el eje x.
10. Hallar el área de la región limitada por la gráfica de la función f (x) = x3 , definida en [0, 1] y la abscisa.
11. Hallar el área de la región limitada por la gráfica de la función f (x) = 1 − x, las rectas verticales x = 2
y x = 5 y el eje x.
12. Hallar el área de la región limitada por la gráfica de la función f (x) = x2 − 9, los ejes coordenados y
x = 2.

13. Hallar el área de la región limitada por la gráfica de la función f (x) = 9 − x, la recta horizontal y = 2
y el eje de la ordenada.
14. Hallar el área de la región limitada por las gráficas de las funciones f (x) = x3 , g (x) = −x y x = 1.

15. Hallar el área de la región limitada por las gráficas de las funciones y = ± x − 2, y + x + 2 = 0 y las
rectas y = −1 y = 2.
16. Hallar el área de la región limitada por la gráfica de la función f (x) = 4 − x2 , la recta vertical x = 1 y
los ejes coordenados.
17. Calcule la suma de Riemann de la función f (x) = 2 + (x − 2)2 en el intervalo [0, 2], usando una partición
con n = 4 y eligiendo x∗i como el extremo izquierdo del i−ésimo subintervalo. Trace la gráfica de f y
los rectángulos de aproximación.
18. Evalúe usando la definición de la integral definida
Z 2 Z 3 Z 2   Z 2
x2 
1. (2x + 3) dx 2. (3x − 2) dx 3. +1 dx 4. x2 + 2 dx
−1 1 −1 2 0

Z 4 Z 1 Z 2 Z 2  
 x2
5. x2 dx 6. 2x2 + 1 dx 7. (x + 1) dx 8. + 1 dx
1 −1 0 0 2
Z 4 Z 1 Z 2 Z 1
  
9. (3x + 1) dx 10. x3 + x dx 11. x2 + x + 1 dx 12. 2x2 − 3 dx
1 0 −1 −2

19. Exprese los siguientes límites como una integral definida en el intervalo indicado
Xn   X n
2 p
1. lim 2 (x∗i ) − 5x∗i △xi , [0, 1] 2. lim x∗i △xi , [1, 4]
n→∞ n→∞
i=1 i=1
n n
X X tan x∗
3. lim cos (x∗i ) △xi , [0, π] 4. lim i
△xi , [2, 4]
n→∞
i=1
n→∞
i=1
x∗i

20. Exprese los siguientes límites como una integral definida


n n   n  
X i3 X π πi X 2 2i
1. lim 4
2. lim sen 3. lim 1+
n→∞
i=1
n n→∞
i=1
n n n→∞
i=1
n n
n n n
"  5 #
X i4 1X 1 X 2i 2
4. lim 5. lim 5 6. lim 3 1+ −6
n→∞
i=1
n5 n→∞ n
i=1 1 + (i/n)
n→∞
i=1
n n
n n n
"  2 #
X 27i2 X 16i3 X 2i 2i 2
7. lim 8. lim 9. lim 1+ +
n→∞
i=1
n3 n→∞
i=1
n4 n→∞
i=1
n n n

Última actualización: Julio 2016 Farith J. Briceño N. farith.math@gmail.com


Matemática II - Guía 5. Integral definida. 89

21. Demuestre que


Z b Z b Z b
b 2 − a2 b 3 − a3
1. dx = b − a 2. x dx = 3. x2 dx =
a a 2 a 3

22. Se conoce que


Z b Z b Z b Z b
b 2 − a2 b 3 − a3
dx = b − a; x dx = ; x2 dx = ; cos x dx = sen b
a a 2 a 3 0

Utilice las propiedades de la integral para calcular las siguientes integrales


Z Z Z √ Z
6 6 − 2 √  4 
1. 3 dx 2. (4 − 7x) dx 3. 2 − 1 dx 4. 2x2 − 3x + 1 dx
2 3 −1 1

Z −1 √ Z 2 Z 2 Z 3
5. 3 dx 6. |x + 1| dx 7. (5x + 3) dx 8. (x − 2) (x + 3) dx
−4 −2 0 1

Z 4 Z 2 Z π/3 Z 1
9. π dx 10. |2x − 3| dx 11. (1 − 2 cos x) dx 12. (5 cos x + 4x) dx
−1 0 0 0
(
Z 5 Z 1 −2x si −1 ≤ x < 0
13. [|x|] dx 14. f (x) dx donde f (x) =
−2 −1 3x2 si 0 ≤ x ≤ 1
(
Z 2 Z 2 1−x si 0 ≤ x ≤ 1
15. [|2x|] dx 16. g (x) dx donde g (x) =
0 0 x−1 si 1 < x ≤ 3
Z 4 Z 10 Z 4 Z 3 Z 1
2 2
17. x dx + x dx 18. (2x + 1) dx + (2x + 1) dx + (2x + 1) dx
0 4 3 1 4

Z b
23. Escriba la suma o diferencia dada como una sola integral de la forma f (x) dx
a
Z 8 Z 5 Z 3 Z 6 Z 12
1. f (x) dx + f (x) dx 2. f (x) dx + f (x) dx + f (x) dx
5 0 1 3 6
Z 10 Z 7 Z 5 Z 0 Z 6
3. f (x) dx − f (x) dx 4. f (x) dx − f (x) dx + f (x) dx
2 2 −3 −3 5

24. Demostrar que


Z a Z a Z b
1. f (x) dx = 0. 2. f (x) dx = − f (x) dx con a < b.
a b a

Z b
25. Demostrar que si f (x) ≥ 0 para a ≤ x ≤ b, entonces f (x) dx ≥ 0.
a
Z b Z b
26. Demostrar que si f (x) ≥ g (x) para a ≤ x ≤ b, entonces f (x) dx ≥ g (x) dx.
a a

27. Demostrar que m ≤ f (x) ≤ M para a ≤ x ≤ b entonces


Z b
m (b − a) ≤ f (x) dx ≤ M (b − a)
a

Última actualización: Julio 2016 Farith J. Briceño N. farith.math@gmail.com


Matemática II - Guía 5. Integral definida. 90

28. Demostrar que Z Z


b b

f (x) dx ≤ |f (x)| dx
a a

29. Utilice las propiedades de la integral para verificar la desigualdad dada en cada uno de los siguientes
ejercicios sin calcular las integrales
Z 1 Z 1 Z 6 Z 8
 
1. x dx ≥ x2 dx ; 2. x2 − 1 dx ≥ 0 ; 3. x2 − 3x + 4 dx ≥ 0 ;
0 0 2 −2

Z π/4 Z π/4 Z 4 Z 6 Z 6
3 2 2 dx dx
4. sen x dx ≤ sen x dx ; 5. 8 ≤ x dx ≤ 32 ; 6. ≤ ;
0 0 2 4 x 4 8−x
Z 2 Z 2 Z 5 Z 5
√ √  
7. 5 − x dx ≥ x + 1 dx ; 8. 4x4 − 3 dx ≥ 3x4 − 4 dx ;
1 1 0 0

Z 2 Z 2 Z π/2 Z π/2
π π π2
9. x dx ≤ x2 dx ; 10. ≤ sen x dx ≤ ; 11. x sen x dx ≤ ;
1 1 6 π/6 3 0 8
Z 1 p √ Z 4 
12. 2 ≤ 1 + x2 dx ≤ 2 2 ; 13. 3 ≤ x2 − 4x + 5 dx ≤ 15 ;
−1 1

Z 5 Z 1 Z π
p p 6 2
π2
14. x2 − 1 dx ≤ 10.5 ; 15. 1 ≤ 1 + x4 dx ≤ ; 16. x cos x dx ≤ ;
2 0 5 0 3

30. (a) Demuestre que 1 ≤ 1 + x2 ≤ 1 + x2 , para x ≥ 0.
Z 1 p
(b) Demuestre que 1 ≤ 1 + x2 dx ≤ 1.25
0
Z 4
31. Calcular f (x) dx, si
0

( 
 1 si 0 ≤ x < 1
2
x si 0 ≤ x < 2 
1. f (x) = ; 2. f (x) = x si 1 ≤ x < 2 ; 3. f (x) = |cos x| ;
x si 2 ≤ x ≤ 4 

4 − x si 2 ≤ x ≤ 4

Z 5
32. Calcular f (x) dx, si
1 

 −3 si 1 ≤ x < 3

f (x) = x si 3 ≤ x ≤ 4


x − x2 si 4 < x ≤ 5

33. Decida si la proposición dada es verdadera o falsa. Justifique su respuesta.


Z b
(a) Si f es continua y f (x) ≥ 0, para todo x de [a, b], entonces f (x) dx ≥ 0.
a

Z b
(b) Si f (x) dx ≤ 0, entonces f (x) ≤ 0, para todo x en [a, b].
a

Z b
(c) Si f (x) dx = 0, entonces f (x) = 0, para todo x en [a, b].
a

Última actualización: Julio 2016 Farith J. Briceño N. farith.math@gmail.com


Matemática II - Guía 5. Integral definida. 91

Z b
(d) Si f (x) ≤ 0 f (x) dx = 0, entonces f (x) = 0, para todo x en [a, b].
a

Z b Z b Z b
(e) Si f (x) dx > g (x) dx, entonces |f (x) − g (x)| dx > 0.
a a a

(f) Si f y g son integrables en [a, b], entonces


Z b Z b Z b
(f (x) + g (x)) dx = f (x) dx + g (x) dx
a a a

(g) Si f y g son integrables en [a, b], entonces


Z b Z ! Z !
b b
(f (x) g (x)) dx = f (x) dx g (x) dx
a a a

(h) Si f y g son funciones continuas y f (x) ≤ g (x) para a ≤ x ≤ b, entonces


Z b Z b
f (x) dx ≤ g (x) dx
a a

34. Calcular la derivada de las siguientes funciones


Z x Z x Z x

1. f (x) = t2 + t dt 2. f (x) = (2t + 1) dt 3. f (x) = sen4 u tan u du
0 −6 0
Z xp Z π/4 Z 1 p
4. f (x) = 1+ t4 dt 5. f (x) = u tan u du 6. f (x) = x2 u2 + 1 du
1 x x
Z Z 3 Z
cos x x p sen x 
7. f (x) = u2 du 8. f (x) = 1 + s4 ds 9. f (x) = u2 + cos u du
sen x x 0
Z 1/x Z x p Z x 20
10. f (x) = sen4 t dt 11. f (x) = t3 + 1 dt 12. f (x) = t2 − 1 dt
2 −1 1
Z u Z t Z 4
sen u  √ 8
13. f (u) = dt 14. f (t) = sen x2 dx 15. f (x) = 2+ u du
π 1 + t4 0 x
Z Z √ Z
2 x 2 17
 s 
16. f (x) = cos t2 dt 17. f (x) = 2
ds 18. f (x) = sen t4 dt
x 1 s +1 tan x
Z π Z x2 Z sen x
sen t sen x 
19. f (x) = dt 20. f (x) = √ dt 21. f (x) = t cos t3 dt
x2 t tan x 2 + t4 −5
Z 2x Z 5x+1 Z x−1
u−1 x4 √
22. f (x) = du 23. f (x) = du 24. f (x) = x sen t dt
3x u+1 0 u2 − 5 arcsen x
2
Z x +1 Z 0 Z x3 √
cos x u4
25. f (x) = dy 26. f (x) = du 27. f (x) = t sen t dt
x2 1 − y2 5x+1
2
x −5 √
x
Z x−1 √
28. f (x) = x arcsen t dt
sen x

35. Si f es una función continua y g y h son funciones diferenciables, encuentre una fórmula para
 
h(x)
Z
d  
 f (t) dt
dx
g(x)

Última actualización: Julio 2016 Farith J. Briceño N. farith.math@gmail.com


Matemática II - Guía 5. Integral definida. 92

Z x 
36. Sea F (x) = t4 + 1 dt
0

(a) Encuentre F (0).


dy
(b) Sea y = F (x). Resuelva la ecuación = x4 + 1.
dx
(c) Encuentre la solución de la ecuación de la parte 36b si se debe satisfacer que y = F (0) cuando
x = 0.
Z 1
 6
(d) Demuestre que x4 + 1 dx = .
0 5
Z x
37. Sea F (x) = sen t dt
0

(a) Encuentre F (0) y F (2π).


dy
(b) Sea y = F (x). Resuelva la ecuación = sen x.
dx
(c) Encuentre la solución de la ecuación de la parte 37b si se debe satisfacer que y = F (0) cuando
x = 0.
Z π
(d) Demuestre que sen x dx = 2.
0
(e) Encuentre todos los puntos extremos relativos y de inflexión de F en el intervalo [0, 4π].
(f) Haga la gráfica de y = F (x) en el intervalo [0, 4π].
Z x
s
38. Demuestre que la gráfica de f (x) = √ ds, a 6= 0, es cóncava hacia arriba en toda su extensión.
0 a + s2
2
Z x
1+t
39. Encuentre el intervalo en que la gráfica de f (x) = √ dt es cóncava hacia arriba.
0 1 + t2
Z x
1
40. Determine el intervalo en que la curva y = 2
dt es cóncava hacia arriba.
0 1+t+t

41. Calcular los siguientes límites, si existen


Z sen x
3
2 arcsen t dt Z xp
x −1 x 1
1. lim 4 2. lim Z 2x 3. lim 1 + t4 dt
x→1 Z x  x→0 p x→0 sen x 0
sen 1 − t 2
dt t3 + 1 dt
1 0

Z x Z x+1 Z cos x
1 2 1
4. lim 5 (1 − cos t) dt 5. lim √ arcsen u du 6. lim cot x (2t + 1) dt
x→0 x 0 x→0+ x cos x x→0 1
Z 2+h
1 p
7. lim 1 + t3 dt
h→0 h 2

Z 5x
1
42. Demuestre que si f (x) = dt, entonces f es una función constante en (0, ∞).
2x t
Z x
43. Encuentre una función f y un valor de la constante a, tal que: 2 f (t) dt = 2 sen x − 1.
a

Respuestas: Ejercicios

1.a.i. 26.875; 1.a.ii. 33.125; 1.b.i. 38.5; 1.b.ii. 46.5; 1.c.i. 7.6875; 1.c.ii. 12.188; 1.d.i. 23.32;

Última actualización: Julio 2016 Farith J. Briceño N. farith.math@gmail.com


Matemática II - Guía 5. Integral definida. 93

38 9 8
1.d.ii. 32.68; 1.e.i. 27.54; 1.e.ii. 44.46; 2. 4.92; 3. 6; 4. 3 ; 5. 2; 6. 3;
8 1 15 46 8 3 33
7. 3; 8. 30; 9. 8; 10. 4; 11. 2 ; 12. 3 ; 13. 3; 14. 4; 15. 2 ;
11 31 9 20 10
16. 3 ; 17. 4 ; 18.1. 12; 18.2. 8; 18.3. 2; 18.4. 3 ; 18.5. 21; 18.6. 3 ;
R1
10 51 3 15
2x2 − 5x dx;

18.7. 4; 18.8. 3 ; 18.9. 2 ; 18.10. 4; 18.11. 2 ; 18.12. − 3; 19.1. 0
R4√ Rπ R4 tan x
R1 3 Rπ
19.2. 1
x dx; 19.3. 0
cos x dx; 19.4. 2 x dx; 20.1. 0
x dx; 20.2. 0
sen x dx;
R3 R1 R1 R3 R1
x4 dx; 1 5
27x2 dx;

20.3. 1
x dx; 20.4. 0
20.5. 0 1+x5
dx; 20.6. 1
3x − 6 dx; 20.7. 0

R1 R2 √ 2
16x3 dx; 2 165 45

20.8. 0
20.9. 0
1+x+x dx; 22.1. 12; 22.2. − 2 ; 22.3. − 2−1 ; 22.4. 2 ;
√ 2 5 π

22.5. 3 3; 22.6. 5; 22.7. 16; 22.8. 3; 22.9. 5π; 22.10. 2; 22.11. 3 − 3; 22.12. 5 sen 1 + 2;
1000
R8
22.13. 7; 22.14. 2; 22.15. 3; 22.16. 1; 22.17. 3 ; 22.18. 0; 23.1. 0
f (x) dx;
R 12 R 10 R6 26 9
23.2. 1
f (x) dx; 23.3. 7
f (x) dx; 23.4. 0
f (x) dx; 31.1. 3 ; 31.2. 2; 31.3. 2 − sen 4;
55
32. − 3 ; 33.a. Verdadero; 33.b. Falso; 33.c. Falso; 33.d. Verdadero; 33.e. Verdadero;

33.f Verdadero; 33.g. Falso; 33.h. Falso; 34.1. x + x2 ; 34.2. 2x + 1; 34.3. sen4 x tan x;
√ R1√ √
34.4. x4 + 1; 34.5. − x tan x; 34.6. 2x x u2 + 1 du − x2 x2 + 1; 34.7. − sen x cos2 x − cos x sen2 x;
√ √ √
34.8. 3x2 1 + x12 − 1 + x4 ; sen2 x + cos (sen x) cos x; 34.10. − x12 sen4 x
1
 
34.9. ; 34.11. x3 + 1;
20 √
x2 − 1 34.13. cos u πu 1+tdt sen u
34.14. sen t2 ; 34.15. − 2 + x 8 ; 34.16. − cos x2 ;
 R   
34.12. ; 4 + 1+u4 ;

2 sen x2
 

 
x R x2 2x
− sen tan4 x sec2 x; √ dt √ 2x − √ sec

34.17. 2(x+1) ; 34.18. 34.19. − x ; 34.20. cos x tan x
+ sen x;
2+t4 2+x8 2+tan4 x

4x−2 9x−3 3 5x+1 5x4


sen x cos x cos sen3 x ; du
 R
34.21. 34.22. 2x+1 − 3x+1 ; 34.23. 4x 0 u2 −5
+ (5x+1)2 −5
;
 
1
R 1/x 1 1 3/2 R x2 +1 dy
− √x 2x 2x

34.24. √ sen t dt − sen ; 34.25. − sen x 1−y2
+ − cos x;
2 x arcsen x x3/2 x
1−x 2 x2 1−(x2 +1)2 1−x4


R 5x+1 4 sen( x)
2x
u du − x25−5 (5x + 1)4 ; 34.27. 3x7/3 sen x3 −

34.26. √ ;
(x2 −5)2 0 2 4x

x5 x5
R 1/x
1 1 1
− x3/2 cos x;

34.28. √
2 x sen x
arcsen t dt − 3/2 arcsen x 36.a. F (0) = 0; 36.b. y = 5 + x + C; 36.c. y = 5 + x;
x

37.a. F (0) = 0 y F (2π) = 0; 37.b. y = − cos x + C; 37.c. y = − cos x + 1;


π 3π 5π 7π
37.e. Valores extremos : Puntos de inflexión :
   
(0, 0) , (π, 2) , (2π, 0) , (3π, 2) , (4π, 0) 2,1 , 2 ,1 , 2 ,1 , 2 ,1 ;
Z x
2 F (x) = sen t dt
0

−∞, − 21 ; 9

37.f. ; 39. (−∞, 1) ; 40. 41.1. − 16 ;

0 π 2π 3π 4π
1 π
41.2. 0; 41.3. 1; 41.4. 20 ; 41.5 0; 41.6. 0; 41.7. 3; 43. y = cos x, a= 6;

Bibliografía

1. Purcell, E. - Varberg, D. - Rigdon, S.: “Cálculo”. Novena Edición. PEARSON Prentice Hall.
2. Stewart, J.: “Cálculo”. Grupo Editorial Iberoamericano.
3. Thomas, George: “Cálculo de una variable”. 12ma edición. Pearson.
4. Larson - Hostetler - Edwards, “Cálculo”. Vol. 1. Mc Graw Hill.
5. Leithold, Louis, “El cálculo con geometría analítica”. Harla S.A.

Este material ha sido revisado recientemente, pero esto no garantiza que esté libre de errores, por esa razón se
agradece reportar cualquier error que usted encuentre en este material enviando un mensaje al correo electrónico
farith.math@gmail.com
indicando donde se encuentra(n) dicho(s) error(es). MUCHAS GRACIAS.

Última actualización: Julio 2016 Farith J. Briceño N. farith.math@gmail.com


Matemática II - Guía 5. Integral definida. 94

Última actualización: Julio 2016 Farith J. Briceño N. farith.math@gmail.com


Matemática II - Guía 6
Teorema Fundamental del Cálculo.

Objetivos a cubrir Código : MAT-CI.6


• Segundo Teorema Fundamental del Cálculo.
• Teorema del Valor Medio para integrales. Teorema sobre simetría.
• Teorema sobre funciones periódicas. Ejercicios resueltos

Z 1
x3 − 8
Ejemplo 91 : Calcular la siguiente integral dx.
−2 x−2

Solución : Por el 2do Teorema Fundamental del Cálculo se tiene que


Z b
f (x) dx = F (b) − F (a) ,
a

x3 − 8
donde F es una primitiva de f , así, se calcula la familia de primitiva de la función f (x) = , para ello
x−2
manipulamos algebraicamente dicha función, factorizamos el numerador

x3 − 8 (x − 2) x2 + 2x + 4
= = x2 + 2x + 4, con x 6= 2,
x−2 x−2
por lo tanto, Z Z
x3 − 8  x3
dx = x2 + 2x + 4 dx = + x2 + 4x + C.
x−2 3
La integral definida es

Primitiva evaluada en Primitiva evaluada en


el límite superior el límite inferior

z ↓ ↓
1 }| {  z }| {
Z 1 3
 3 3 3
x −8 x  (1) 2   (−2) 2
+ x2 + 4x = 

dx = + (1) + 4 (1) −  (−2) + 4 (−2)
−2 x−2 3 −2 3 3

       
1 8 16 20
= +1+4 − − +4−8 = − − = 12.
3 3 3 3
Luego,
Z 1
x3 − 8
dx = 12.
−2 x−2

Z π/2
sen2 x dx
Ejemplo 92 : Calcular la siguiente integral .
0 cos2 (x/2)

Solución : Por el 2do Teorema Fundamental del Cálculo se tiene que


Z b
f (x) dx = F (b) − F (a) ,
a

sen2 x
donde F es una primitiva de f , así, se calcula la familia de primitiva de la función f (x) = , para
cos2 (x/2)
ello procedemos de la siguiente manera:

Última actualización: Julio 2016 Farith J. Briceño N. farith.math@gmail.com


Matemática II - Guía 6. Teorema Fundamental del Cálculo. 96

Es conocido que
sen 2 (·) = 2 sen (·) cos (·) , (2)
por otro lado, x
sen x = sen 2
2
por la ecuación (2) se tiene
x x  x  x 2 x x
sen x = 2 sen cos =⇒ sen2 x = 2 sen cos = 4 sen2 cos2
2 2 2 2 2 2
así,    
Z 2 Z 4 sen2 x cos2 x Z  
sen x dx 2 2 dx = 4 sen2 x dx
2
= 2
cos (x/2) cos (x/2) 2
como
1 − cos 2 (·)
sen2 (·) = ,
2
entonces, x
x 1 − cos 2 1 − cos x
sen2 = 2 = ,
2 2 2
esto implica
Z x Z Z Z Z 
1 − cos x
4 sen2 dx = 4 dx = 2 (1 − cos x) dx = 2 dx − cos x dx = 2x − 2 sen x + C.
2 2

Finalmente Z
sen2 x dx
= 2x − 2 sen x + C.
cos2 (x/2)
La integral definida queda

Primitiva evaluada en Primitiva evaluada en


el límite superior el límite inferior
↓ !  ↓
Z π/2 2
 π/2 z   }|  π { {
sen x dx π z }|
= 2x − 2 sen x = 2 − 2 sen − 2 (0) − 2 sen (0) = π − 2
0 cos2 (x/2) 0 2 2

Ejemplo 93 : Calcular la siguiente integral


Z 8 2
x − 6x + 8 dx.
0

Solución : Por definición de valor absoluto, se tiene que


(
2 x2 − 6x + 8 si x2 − 6x + 8 ≥ 0
x − 6x + 8 = 
− x2 − 6x + 8 si x2 − 6x + 8 < 0

resolvemos una de las dos desigualdades

x2 − 6x + 8 ≥ 0 ó x2 − 6x + 8 < 0,

para obtener la ubicación de cada expresión en la recta real. Resolvemos x2 − 6x + 8 ≥ 0

x2 − 6x + 8 = (x − 2) (x − 4) ≥ 0,

Última actualización: Julio 2016 Farith J. Briceño N. farith.math@gmail.com


Matemática II - Guía 6. Teorema Fundamental del Cálculo. 97

de aquí,
(−∞, 2) (2, 4) (4, ∞)

x−2 − + +

x−4 − − +

(x − 2) (x − 4) + − +

la definición de valor absoluto nos queda


(
2 x2 − 6x + 8 si x ∈ (−∞, 2) ∪ (4, ∞)
x − 6x + 8 = 
− x2 − 6x + 8 si x ∈ (2, 4)
con lo que,
2 4

x2 − 6x + 8 − x2 − 6x + 8 x2 − 6x + 8

entonces, la integral a resolver la dividimos en tres integrales


Z 8 Z 2 Z 4 Z 8
2   
x − 6x + 8 dx = x2 − 6x + 8 dx + − x2 − 6x + 8 dx + x2 − 6x + 8 dx,
0 0 2 4

donde,

Primitiva evaluada en Primitiva evaluada en


el límite superior el límite inferior

z ↓ ↓
2 }| { z }| {
Z 2  3 3 3
 x  (2) 2   (0) 2
x2 − 6x + 8 dx = − 3x2 + 8x = 

− 3 (2) + 8 (2) −  − 3 (0) + 8 (0)
0 3 0 3 3

8 8 20
= − 12 + 16 = + 4 = ,
3 3 3
mientras que,

Primitiva evaluada en Primitiva evaluada en


el límite superior el límite inferior

z ↓ ↓
4 }| { z }| {
Z 4  3 3 3
 x  (4) 2   (2) 2
− x2 − 6x + 8 dx = − − 3x2 + 8x = − 

− 3 (4) + 8 (4) −  − 3 (2) + 8 (2)
2 3 2 3 3

     
64 8 16 20 4
=− − 48 + 32 − − 12 + 16 =− − = ,
3 3 3 3 3
y por último,

Primitiva evaluada en Primitiva evaluada en


el límite superior el límite inferior

z ↓ ↓
}| {  z }| {
Z 8  3 8 3 3
 x  (8) 2   (4) 2
x2 − 6x + 8 dx = − 3x2 + 8x = 

− 3 (8) + 8 (8) −  − 3 (4) + 8 (4)
4 3 4 3 3

Última actualización: Julio 2016 Farith J. Briceño N. farith.math@gmail.com


Matemática II - Guía 6. Teorema Fundamental del Cálculo. 98

   
512 64 128 16 112
= − 192 + 64 − − 48 + 32 = − = .
3 3 3 3 3

Luego, Z 8
x − 6x + 8 dx = 20 + 4 + 112 = 136 .
2
0 3 3 3 3

Ejemplo 94 : Calcular la siguiente integral


Z √
6/2
dx
√ .
0 3 − 2x2

Solución : Por el 2do Teorema Fundamental del Cálculo se tiene que


Z b
f (x) dx = F (b) − F (a) ,
a

donde, F es una primitiva de f .


1
Buscamos la familia de primitiva de f (x) = √ .
3 − 2x2
Es conocido que, Z
1
√ dx = arcsen x + C,
1 − x2
así, manipulando el integrando, obtenemos
1 1 1 1 1 1 1 1
√ =s  = √ r = √ v = √ v !2 ,
3 − 2x2
2x2
 3 2x 2 3 u
u √ ! 2 3 u
u √
3 1− 1− t1 − √2x t1 − 6 x
3 3
3 3

entonces,
√ √ √ √
Z 6 Z 6 Z 6 Z 6
2 dx 2 1 1 1 2 dx 1 2 dx
√ = √ v dx = √ v =√ v !2 ,
0 3 − 2x2 0 3 u
u √ !2 3 0
u √ !2 3 0
u √
t1 − √2x t1 − √2x 6x
u u
t1 −
3 3 3

se propone el cambio de variable


√ √ √
6x Cálculo del 6 3 6
u= −−−−−−−−−→ du = dx =⇒ √ du = dx =⇒ du = dx,
3 diferencial 3 6 2

con este cambio se espera transformar la integral en una integral sencilla de resolver, es decir, en una integral de
tabla.

Cambiamos el intervalo de integración



6 (0)
Si x = 0, entonces, u= =⇒ u=0
3
√ !
√ 6
√ 6 6
6 2
Si x= , entonces, u= = 2 =⇒ u = 1,
2 3 3

Última actualización: Julio 2016 Farith J. Briceño N. farith.math@gmail.com


Matemática II - Guía 6. Teorema Fundamental del Cálculo. 99

la integral queda
Z √6/2 Z 1 √6 √ Z 1 √  1
dx 1 2 du 1 6 du 2
√ =√ √ = √ √ = arcsen u
0 3 − 2x2 3 0 1−u 2 3 2 0 1−u 2 2 0

√ ! √
 √2 π
2 arcsen (1) − arcsen (0) 2 π
= = −0 = .
2 | {z } | {z } 2 2 4
↑ ↑
Primitiva evaluada en Primitiva evaluada en
el límite superior el límite inferior

Luego,
Z √
6/2

dx 2π
√ = .
0 3 − 2x2 4

Ejemplo 95 : Calcular la siguiente integral
Z 3 2
3x + 6x + 8
dx.
2
−1 x + 2x + 3

Solución : Por definición de valor absoluto, se tiene que



3x2 + 6x + 8 3x2 + 6x + 8
2 

 si ≥0
3x + 6x + 8  x2 + 2x + 3 x2 + 2x + 3
=
x2 + 2x + 3  2
 − 3x + 6x + 8

 3x2 + 6x + 8
si <0
x2 + 2x + 3 x2 + 2x + 3
resolvemos una de las dos desigualdades
3x2 + 6x + 8 3x2 + 6x + 8
≥0 ó < 0,
x2 + 2x + 3 x2 + 2x + 3
3x2 + 6x + 8
para obtener la ubicación de cada expresión en la recta real. Resolvemos ≥0
x2 + 2x + 3
Factorizamos numerador y denominador, en ambos casos, por ser polinomios cuadráticos, aplicamos la resol-
vente
• Numerador : 3x2 + 6x + 8. Aplicamos la resolvente para a = 3, b = 6 y c = 8
q  
2 √ √
− (6) ± (6) − 4 (3) (8) −6 ± 36 − 96 −6 ± −60
x= = =   ← raíz imaginaria
2 (3) 2 (3) 6

• Denominador : x2 + 2x + 3. Aplicamos la resolvente para a = 1, b = 2 y c = 3


q  
2 √ √
− (2) ± (2) − 4 (1) (3) −2 ± 4 − 12 −2 ± −8
x= = =   ← raíz imaginaria
2 (1) 2 (1) 2

de aquí,
(−∞, ∞)

3x2 + 6x + 8 +

x2 + 2x + 3 +

3x2 + 6x + 8
+
x2 + 2x + 3

Última actualización: Julio 2016 Farith J. Briceño N. farith.math@gmail.com


Matemática II - Guía 6. Teorema Fundamental del Cálculo. 100

3x2 + 6x + 8
es decir, la expresión siempre es mayor estricto que cero, luego
x2 + 2x + 3
2 Z 3 2 Z 3
3x + 6x + 8 3x2 + 6x + 8 3x + 6x + 8 3x2 + 6x + 8
=
x2 + 2x + 3 , así, dx = dx.
x2 + 2x + 3 2
−1 x + 2x + 3
2
−1 x + 2x + 3

Manipulando algebraicamente el integrando, observemos que podemos dividir los polinomios o también,
escribir el numerador como
 
3x2 + 6x + 8 = 3x2 + 6x + 9 − 1 = 3x2 + 6x + 9 − 1 = 3 x2 + 2x + 3 − 1,

con lo que, el integrando queda


 
3x2 + 6x + 8 3 x2 + 2x + 3 − 1 3 x2 + 2x + 3 1 1
= = − 2 =3− 2 ,
x2 + 2x + 3 x2 + 2x + 3 x2 + 2x + 3 x + 2x + 3 x + 2x + 3
es decir,  
Z 3 Z 3 Z 3 Z 3
3x2 + 6x + 8 1 1
dx = 3− dx = 3 dx − dx,
−1 x2 + 2x + 3 −1
2
x + 2x + 3 −1 −1 x2 + 2x + 3
donde,
Primitiva evaluada en Primitiva evaluada en
el límite superior el límite inferior

Z 3  3  ↓ ↓ 

3 dx = 3 x =3
(3) − (−1) = 3 (3 + 1) = 12,
−1 −1
mientras que, para resolver la segunda integral de la derecha de la igualdad, completamos cuadrado en el de-
nominador y manipulamos algebraicamente
!  2 !
2
2 2 (x + 1) x+1
x + 2x + 3 = (x + 1) + 2 = 2 1 + =2 1+ √ ,
2 2

entonces, Z Z Z
3 3 3
1 1 1 1
dx = 2 ! dx = 2  2 dx,
−1 x2 + 2x + 3 −1

x+1 −1 x+1
2 1+ √ 1+ √
2 2
se propone el cambio de variable

x+1 Cálculo del 1 √


u= √ −−−−−−−−−→ du = √ dx =⇒ 2 du = dx,
2 diferencial 2

con este cambio se espera transformar la integral en una integral sencilla de resolver, es decir, en una integral de
tabla.

Cambiamos el intervalo de integración


(−1) + 1 0
Si x = −1, entonces, u= √ = √ =⇒ u=0
2 2

(3) + 1 4 4 2 √
Si x = 3, entonces, u= √ =√ = √ √ =⇒ u = 2 2,
2 2 2 2
la integral queda
Z 3 Z √
2 2
√ √ Z 2√2 √  2√2
1 1 1 2 du 2 du 2
 2 dx = = = arctan u
2 −1 x+1 2 0 1 + u2 2 0 1 + u2 2
0
1+ √
2

Última actualización: Julio 2016 Farith J. Briceño N. farith.math@gmail.com


Matemática II - Guía 6. Teorema Fundamental del Cálculo. 101

Primitiva evaluada en Primitiva evaluada en


el límite superior el límite inferior

√ ↓ ↓


2  z }| {
√  z }| {  2 √ 
= arctan 2 2 − arctan (0) = arctan 2 2
2 2

Finalmente, 2 √
Z 3  √ 
3x + 6x + 8
dx = 12 − 2 arctan 2 2 .
x2 + 2x + 3 2
−1

Ejemplo 96 : Encuentre
√ todos los valores de x que satisfacen el Teorema de Valor Medio para integrales de
la función f (x) = 2x + 1 en el intervalo [1, 4].

Solución : Observemos que la función f es continua en el intervalo [1, 4], entonces el Teorema del Valor
Medio para integrales garantiza la existencia de, al menos, un número x = c en [1, 4], tal que se cumple
Z 4 Z 4
1 √ 1 √
f (c) = 2x + 1 dx = 2x + 1 dx,
4−1 1 3 1

calculamos la integral, para ello se propone el cambio de variable

Cálculo del du
u = 2x + 1 −−−−−−−−−→ du = 2 dx =⇒ = dx,
diferencial 2

con este cambio se espera transformar la integral en una integral sencilla de resolver, es decir, en una integral de
tabla.

Cambiamos los límites de integración

Si x=1 entonces, u = 2 (1) + 1 =⇒ u=3


Si x=4 entonces, u = 2 (4) + 1 =⇒ u=9

y la integral nos queda

Primitiva evaluada en Primitiva evaluada en


el límite superior el límite inferior

↓ ↓
Z 4 √
Z 9 √
Z 9  9 z}|{ z}|{! √  √
du 1 1 2 3/2 1 1
2x + 1 dx = u = u1/2 du = u = 93/2 − 33/2 = 27 − 3 3 = 9 − 3
1 3 2 2 3 2 3 3 3 3

entonces, √
1 √  3
f (c) = 9 − 3 =⇒ f (c) = 3 − ,
3 3

como f (c) = 2c + 1, tenemos
√ √ !2
√ 3 3 √ 1
2c + 1 = 3 − =⇒ 2c + 1 = 3− =⇒ 2c + 1 = 9 − 2 3 +
3 3 3

√ 1 25 √ 25 √
=⇒ 2c = 8 − 2 3 + =⇒ 2c = −2 3 =⇒ c= − 3.
3 3 6

Última actualización: Julio 2016 Farith J. Briceño N. farith.math@gmail.com


Matemática II - Guía 6. Teorema Fundamental del Cálculo. 102

Ejemplo 97 : Encuentre los números b tales que el valor promedio de f (x) = 2 + 6x − 3x2 en el intervalo
cerrado [0, b] sea igual a 3.

Solución : Es conocido que el valor promedio de una función f en el intervalo [a, b] se define como
Z b
1
fprom = f (x) dx
b−a a
así,
Z b
1 
3 = fprom = 2 + 6x − 3x2 dx,
b−0 0
observemos que b 6= 0, ¿Por qué?. Calculamos la integral,
Z b  b
2

2 + 6x − 3x dx = 2x + 3x2 − x3 = 2b + 3b2 − b3

0 0

con lo que obtenemos


1 
3= 2b + 3b2 − b3 =⇒ 3 = 2 + 3b − b2 =⇒ b2 − 3b + 1 = 0,
b
las raíces reales del polinomio de segundo grado son los valores de b, las cuales son
√ √
3+ 5 3− 5
b= y b=
2 2

Z a
Ejemplo 98 : Demuestre que si f es una función impar, entonces f (x) dx = 0.
−a

Demostración : Como f es impar, se cumple que f (x) = −f (−x), para todo x ∈ Dom f , así,
Z a Z 0 Z a Z 0 Z a
f (x) dx = f (x) dx + f (x) dx = − f (−x) dx + f (x) dx,
−a −a 0 −a 0
Z 0
Consideremos la primera integral del lado derecho de la última igualdad, f (−x) dx. Se propone el
−a
cambio de variable
Cálculo del
u = −x −−−−−−−−−→ du = − dx =⇒ − du = dx,
diferencial

Cambiamos los límites de integración


Si x = −a entonces, u = − (−a) =⇒ u=a

Si x=0 entonces, u = − (0) =⇒ u = 0,


la integral nos queda, Z Z Z
0 0 a
f (−x) dx = − f (u) du = f (u) du,
−a a 0
luego, Z Z Z
a a a
f (x) dx = − f (u) du + f (x) dx,
−a 0 0
observemos que ambas integrales a la derecha de la igualdad representan la misma cantidad numérica, ya que si
F es una primitiva de f , entonces
Z a  a Z a  a

f (u) du = F (u) = F (a) − F (0) y f (x) dx = F (x) = F (a) − F (0) ,
0 0 0 0

Última actualización: Julio 2016 Farith J. Briceño N. farith.math@gmail.com


Matemática II - Guía 6. Teorema Fundamental del Cálculo. 103

por lo tanto, Z a
f (x) dx = − (F (a) − F (0)) + (F (a) − F (0)) = 0.
−a


Z a Z a
Ejemplo 99 : Demuestre que si f es una función par, entonces f (x) dx = 2 f (x) dx.
−a 0

Demostración : Como f es par, se cumple que f (x) = f (−x), para todo x ∈ Dom f , así,
Z a Z 0 Z a Z 0 Z a
f (x) dx = f (x) dx + f (x) dx = f (−x) dx + f (x) dx,
−a −a 0 −a 0

Z 0
Consideremos la primera integral del lado derecho de la última igualdad, f (−x) dx. Se propone el
−a
cambio de variable
Cálculo del
u = −x −−−−−−−−−→ du = − dx =⇒ − du = dx.
diferencial

Cambiamos los límites de integración

Si x = −a entonces, u = − (−a) =⇒ u=a

Si x=0 entonces, u = − (0) =⇒ u = 0,

la integral nos queda, Z Z Z


0 0 a
f (−x) dx = − f (u) du = f (u) du,
−a a 0

luego, Z Z Z
a a a
f (x) dx = f (u) du + f (x) dx,
−a 0 0

observemos que ambas integrales a la derecha de la igualdad representan la misma cantidad numérica, ya que si
F es una primitiva de f , entonces
Z a  a Z a  a

f (u) du = F (u) = F (a) − F (0) y f (x) dx = F (x) = F (a) − F (0) ,
0 0 0 0

por lo tanto,
Z a Z a
f (x) dx = (F (a) − F (0)) + (F (a) − F (0)) = 2 (F (a) − F (0)) = 2 f (x) dx.
−a 0

Ejemplo 100 : Calcular la integral Z 3


sen π3 (x − 2)
2 dx.
1 (x − 2) + 1
Solución : Se propone el cambio de variable

Cálculo del
u=x−2 −−−−−−−−−→ du = dx,
diferencial

con este cambio se espera transformar la integral en una integral sencilla de resolver, es decir, en una integral de
tabla.

Última actualización: Julio 2016 Farith J. Briceño N. farith.math@gmail.com


Matemática II - Guía 6. Teorema Fundamental del Cálculo. 104

Cambiamos los límites de integración

Si x=1 entonces, u = (1) − 2 =⇒ u = −1

Si x=3 entonces, u = (3) − 2 =⇒ u=1

la integral nos queda 


Z Z
3
sen π3 (x − 2) 1
sen π3 u
2 dx = du,
1 (x − 2) + 1 −1 u2 + 1
en vista que estamos integrando sobre un intervalo simétrico, estudiamos la simetría de la función
 
sen π3 (−u) − sen π3 u sen π3 u
f (−u) = 2 = =− 2 = −f (u) ,
(−u) + 1 u2 + 1 u +1

sen π3 u
es decir, la función es impar, luego, la integral de la función impar f (u) = 2 sobre el intervalo simétrico
u +1
[−1, 1] es igual a cero (ver ejemplo 98) 
Z 1
sen π3 u
2
du = 0.
−1 u + 1

Así, Z 3
sen π3 (x − 2)
2 dx = 0.
1 (x − 2) + 1

Z π Z π
π
Ejemplo 101 : Use la sustitución u = π − x para demostrar que xf (sen x) dx = f (sen x) dx.
0 2 0

Demostración : El cambio de variable propuesto es

Cálculo del
u=π−x =⇒ x=π−u −−−−−−−−−→ du = − dx =⇒ − du = dx,
diferencial

cambiamos los límites de integración,

Si x=0 entonces, u = π − (0) =⇒ u=π

Si x=π entonces, u = π − (π) =⇒ u = 0,

la integral nos queda


Z π Z 0 Z 0
xf (sen x) dx = (π − u) f (sen (π − u)) (−du) = − (π − u) f (sen (π − u)) du
0 π π
Z π
= (π − u) f (sen (π − u)) du.
0

Además, es conocido que

sen (π − u) = sen (π) cos u − sen u cos (π) = (0) cos u − sen u (−1) = sen u,

por lo tanto,
Z π Z π Z π Z π
(π − u) f (sen (π − u)) du = (π − u) f (sen u) du = πf (sen u) du − uf (sen u) du,
0 0 0 0

de aquí, Z Z Z
π π π
xf (sen x) dx = πf (sen u) du − uf (sen u) du,
0 0 0

Última actualización: Julio 2016 Farith J. Briceño N. farith.math@gmail.com


Matemática II - Guía 6. Teorema Fundamental del Cálculo. 105

puesto que, las integrales


Z π Z π
xf (sen x) dx y uf (sen u) du
0 0

son iguales, se tiene Z Z Z


π π π
xf (sen x) dx + xf (sen x) dx = π f (sen u) du,
0 0 0
esto implica que Z Z
π π
2 xf (sen x) dx = π f (sen u) du,
0 0
con lo que concluimos que Z Z
π π
π
xf (sen x) dx = f (sen u) du.
0 2 0

Z 1 Z 1
b a
Ejemplo 102 : Si a y b son números positivos, demuestre que xa (1 − x) dx = xb (1 − x) dx.
0 0

Demostración : Se propone el cambio de variable

Cálculo del
u = 1−x =⇒ x=1−u −−−−−−−−−→ du = − dx =⇒ − du = dx,
diferencial

cambiamos los límites de integración,

Si x=0 entonces, u = 1 − (0) =⇒ u=1

Si x=1 entonces, u = 1 − (1) =⇒ u = 0,

la integral nos queda


Z 1 Z 0 Z 0 Z 1
b a a a
xa (1 − x) dx = (1 − u) ub (−du) = − ub (1 − u) du = ub (1 − u) du,
0 1 1 0
Z 1 Z 1
a a
observemos que la integral ub (1 − u) du tiene el mismo valor numérico que xb (1 − x) dx, por lo
0 0
tanto Z Z
1 1
b a
xa (1 − x) dx = xb (1 − x) dx.
0 0

Ejemplo 103 : Demuestre que si f es una función periódica, con período p, entonces
Z b+p Z b
f (x) dx = f (x) dx.
a+p a

Demostración : Como f es periódica, con período p, se cumple que f (x) = f (x + p) = f (x − p), para
todo x ∈ Dom f , entonces
Z b+p Z b+p
f (x) dx = f (x − p) dx,
a+p a+p

se propone el cambio de variable

Cálculo del
u=x−p −−−−−−−−−→ du = dx,
diferencial

Última actualización: Julio 2016 Farith J. Briceño N. farith.math@gmail.com


Matemática II - Guía 6. Teorema Fundamental del Cálculo. 106

cambiamos los límites de integración,


Si x=a+p entonces, u = (a + p) − p =⇒ u=a

Si x=b+p entonces, u = (b + p) − p =⇒ u = b,
la integral nos queda
Z b+p Z b
f (x − p) dx = f (u) du,
a+p a
Z b Z b
observemos que la integral f (u) du tiene el mismo valor numérico que f (x) dx, ya que, si F es una
a a
primitiva de la función f , entonces
Z b  b Z b  b

f (u) du = F (u) = F (b) − F (a) y f (x) dx = F (x) = F (b) − F (a) ,
a a a a

así,
Z b Z b
f (u) du = F (b) − F (a) = f (x) dx,
a a
por lo tanto,
Z b+p Z b
f (x) dx = f (x) dx.
a+p a

Z 100π
Ejemplo 104 : Calcular |sen x| dx.
0

Solución : Es conocido que la función f (x) = sen x es una función períodica, de período p = 2π.

1 y
y = sen x


− 2

− 2

− 2 − π2 π
2

2

2

2 4π

−4π −3π −2π −π 0 π 2π 3π

−1

mientras que, la función g (x) = |sen x| es una función períodica, de período p = π.

1 y y = |sen x|


− 2

− 2

− 2 − π2 π
2

2

2

2

−4π −3π −2π −π 0 π 2π 3π 4π

−1

Última actualización: Julio 2016 Farith J. Briceño N. farith.math@gmail.com


Matemática II - Guía 6. Teorema Fundamental del Cálculo. 107

Por lo tanto,
Z 100π Z π Z 2π Z 3π Z 4π Z 100π
|sen x| dx = sen x dx + sen x dx + sen x dx + sen x dx + · · · + sen x dx
0 0 π 2π 3π 99π
Z π Z π Z π Z π Z π Z π
= sen x dx + sen x dx + sen x dx + sen x dx + · · · + sen x dx = 100 sen x dx,
0 0 0 0 0 0
| {z }
100 veces

de aquí,

Primitiva evaluada en Primitiva evaluada en


el límite superior el límite inferior

Z 100π Z π  π ↓ ↓ !
z }| { z }| {
|sen x| dx = 100 sen x dx = 100 − cos (x) = −100 cos (π) − cos (0) = 200.
0 0 0

Luego, Z 100π
|sen x| dx = 200.
0

Z 1+π
Ejemplo 105 : Calcular |cos x| dx.
1
Solución : Por el ejercicio 23 se tiene
Z 1+π Z π
|cos x| dx = |cos x| dx,
1 0
de aquí,

1 1 y = |cos x|
y = cos x

π 0 π π π 0 π π
−2 2 −2 2

−1 −1

por lo que

Primitiva evaluada en Primitiva evaluada en


el límite superior el límite inferior

 
Z π Z π/2  π/2 z }|
 π { z }| {
|cos x| dx = 2 cos x dx = 2 sen (x) = 2  sen − sen (0)
 = 2.
0 0 0 2
Luego
Z 1+π
|cos x| dx = 2.
1

Última actualización: Julio 2016 Farith J. Briceño N. farith.math@gmail.com


Matemática II - Guía 6. Teorema Fundamental del Cálculo. 108

Ejercicios

1. Calcule las siguientes integrales


Z 4 Z 2 Z −1 Z 3/2 Z 3 Z 2
1. dx 2. 3 dx 3. −2 dx 4. π dx 5. x dx 6. x3 dx
0 −1 −3 1/2 1 0

Z 2 Z 4 Z 4 Z 2 Z 8
4 1 √ 2 √
3
7. x dx 8. dw 9. t dt 10. dt 11. t dt
−1 1 w2 0 1 t3 1

Z π/2 Z π/2 Z π/2 Z 2 


12. cos t dt 13. 2 sen t dt 14. csc2 t dt 15. 4t3 + 7 dt
0 π/6 π/4 1

Z 4   Z 1   Z π/2
2 1 4/3 1/3
16. t + 3 dt 17. x − 2x dx 18. (2x + sen x) dx
2 t 0 0
Z 1 √ √  Z 8a   Z 2
4 1/3 1/3

3x2 − 2x + 3 dx
5
19. x5 + 4 dx 20. a −x dx 21.
0 a −1

Z π/2 Z 1 Z −1
 1 − t4
22. (4t + 3 + cos t) dt 23. 2t4 − 3t2 + 5 dt 24. dt
0 0 −4 2t2
Z 4 Z 4 Z 1 Z 1
x5 − x s4 − 8 x + x3 + x5 2
25. dx 26. ds 27. dx 28. t2 + 2t dt
1 3x3 1 s2 −1 1 + x2 + x4 0

Z π Z π Z 1 Z π/4
x2 − 1 cos2 x
29. sen x dx 30. cos x dx 31. dx 32. dx
−π −π 0 x+1 0 1 + sen x
Z π/6 Z 1 Z 2 Z 1/2
sen (4x) dx x2 − 16 x4 − 16 x3 − 1
33. 34. √ dx 35. dx 36. dx
0 cos (2x) cos x 0 2− x 0 x2 + 4 −1/2 x−1
Z 2 Z 1/2 Z π/4
2
 2 1 − x2 cos (2x)
37. sen x + cos x dx 38. dx 39. dx
−2 0 x4 − 1 0 cos x + sen x
Z 3 Z 3 Z 1 Z 2
 2   3 
40. [[x]] dx 41. x dx 42. x dx 43. [[2x − 1]] dx
−1 −2 −1 0

Z Z Z √
5/3 2   7  
44. [[2 − 3x]] dx 45. √ t2 − 1 dt 46. 2 − x2 dx
0 − 5 −2

Z √ Z Z
3   2   1  
47. √ x2 + 1 dx 48. x2 − 2x − 1 dx 49. x2 + 2x + 1 dx
− 2 −2 −2

Z 0  2 
50. −x − 4x − 2 dx
−3

2. Calcular las siguientes integrales usando un cambio de variable apropiado


Z 3 Z 2 Z 1 Z 10
dt x dx z dz p
1. 2 2. 2 3. 3 4. y − 1 dy
−1 (t + 2) 0 (x2 + 1) 0 (z 2 + 1) 2

Z Z Z √
6 Z √
1 2 3
dx dx 3 dz dx
5. √ 6. √ 7. 8.
−1 6 − x2 0 3 − 5x2 0 3z 2 + 2 −1 x2 + 3

Última actualización: Julio 2016 Farith J. Briceño N. farith.math@gmail.com


Matemática II - Guía 6. Teorema Fundamental del Cálculo. 109

Z 8 Z 1 Z 7 Z 3
√ 2
10 dt dt
9. 3x + 1 dx 10. 2x x + 1 dx 11. √ 12. √
5 0 1 2t + 2 1 7 − 2t
Z 3 Z π/2 Z π/2
t2 + 1
13. √ dt 14. cos2 x sen x dx 15. sen2 (3x) cos (3x) dx
1 t3 + 3t 0 0

Z 4 Z 2π Z 1/2 Z π/2
dt arcsen t x2 dx
16. √ √ 3 17. sen3 x dx 18. √ dt 19.
1 t t+1 0 0 1 − t2 −π/2 1 + x6

Z π/3 Z 3 Z 3 Z 4
x3 dx p √
20. sen5 θ dθ 21. √ 22. t 7 + t2 dt 23. x 16 − 3x dx
−π/3 0 x2 + 9 −3 0

Z 0 Z 4 Z 2π
p √ √  sen x dx
24. 3x2 x3 + 1 dx 25. x+ 2x + 1 dx 26.
−1 0 −2π cos2 x + sec2 x
Z 5 Z 4 Z π/6 Z 2

3 x dx dx x3 dx
27. 2 − 3x dx 28. √ 29. 30. √
−1 0 1 + 2x π/4 cos x sen2 x
2
0
3
3 − 2x2
3. Calcular las siguientes integrales
Z 1 Z 1 Z 2 Z 4
3
1. |x| dx 2. x dx 3. (x − 2 |x|) dx 4. |x − 2| dx
−2 −2 −1 0
Z 2π Z 3 Z 3 Z 1
2 3 
5. |sen t| dt 6. |2x − 3| dx 7. x − 1 dx 8. t + t3 dt
0 0 −2 −1
Z 2 Z π Z 5
 
9. x2 − |x − 1| dx 10. x5 + |sen x| dx 11. 7x − 2x2 − 3 dx
0 −π 0
Z 4 Z 3 p Z 1 p
2
12. x − 4x + 3 dx 13. 3 − |t| dt 14. |t| − t dt
0 −3 −1

Z 4
4. Si f (1) = 12, f ′ continua y f ′ (x) dx = 17, ¿cuál es el valor de f (4)?
1

5. Si f es continua sobre R, demuestre que


Z b Z −b
f (−x) dx = f (x) dx.
a −a

6. Si f es continua sobre R, demuestre que


Z b Z b+c
f (x + c) dx = f (x) dx.
a a+c

7. Si a y b son números positivos, demuestre que


Z 1 Z 1
b a
xa (1 − x) dx = xb (1 − x) dx.
0 0

Z π Z π
π
8. Use la sustitución u = π − x para demostrar que xf (sen x) dx = f (sen x) dx.
0 2 0

9. Suponga que f ′ es integrable y |f ′ (x)| ≤ M , para todo x. Demuestre que

|f (x)| ≤ |f (a)| + M |b − a| .

Última actualización: Julio 2016 Farith J. Briceño N. farith.math@gmail.com


Matemática II - Guía 6. Teorema Fundamental del Cálculo. 110

Z x
10. Encuentre una función f y un valor de la constante a, tal que: 2 f (t) dt = 2 sen x − 1.
a
Z b
2 2
11. Si f ′ es continua en [a, b], demuestre que: 2 f (x) f ′ (x) dx = [f (b)] − [f (a)] .
a

12. Demuestre que si F ′ (x) = D, para toda x de (a, b), existe una constante C, tal que, F (x) = Dx + C,
para toda x de (a, b).
13. Suponga que F ′ (x) = 5 y F (0) = 4. Encuentre la fórmula de F (x).
14. Encontrar el valor c que satisface el Teorema del Valor Medio para integrales si
Z 2 Z 1 Z 4 Z 1
 
1. x3 dx 2. x (1 − x) dx 3. x2 + 4x + 5 dx 4. x2 + x − 6 dx
1 0 1 0
Z 1 Z 2 Z π Z 2
√ 
5. x4 dx 6. x + 1 dx 7. cos (2x) dx 8. x3 + 1 dx
−2 0 0 −2
Z b
15. Si f es una función continua en [a, b] y f (x) dx = 0. Demuestre que existe, al menos, un número c
a
en [a, b], tal que f (c) = 0.
Z 3
16. Si f es una función continua en [1, 3] y f (x) dx = 8. Demuestre que f toma el valor 4 por lo
1
menos una vez sobre el intervalo [1, 3].
17. Si fprom [a, b] denota el valor promedio de f en el intervalo [a, b] y a < x < b, demuestre que
c−a b−c
fprom [a, b] = fprom [a, c] + fprom [c, b]
b−a b−a
18. Demuestre que si f es una función par, entonces
Z a Z a
f (x) dx = 2 f (x) dx.
−a 0

19. Demuestre que si f es una función impar, entonces


Z a
f (x) dx = 0.
−a

20. Sea f una función impar y g una función par y suponga que
Z 1 Z 1
|f (x)| dx = g (x) dx = 3.
0 0
Utilice un razonamiento geométrico para calcular cada una de las siguientes integrales
Z 1 Z 1 Z 1 Z 1
1. f (x) dx 2. g (x) dx 3. |f (x)| dx 4. (−g (x)) dx
−1 −1 −1 −1

Z 1 Z 1
5. xg (x) dx 6. f 2 (x) g (x) dx
−1 −1

21. Demuestre que !


Z 1
5 9 sen x
x − 6x + 4 dx = 0.
−1 (1 + x4 )
22. Demuestre que si f es una función periódica con período p, entonces
Z b+p Z b
f (x) dx = f (x) dx.
a+p a

Última actualización: Julio 2016 Farith J. Briceño N. farith.math@gmail.com


Matemática II - Guía 6. Teorema Fundamental del Cálculo. 111

23. Demuestre que si f es una función periódica con período p, entonces


Z a+p Z p
f (x) dx = f (x) dx.
a 0

24. Calcular
Z 4π Z 4π Z 4π Z 100π
1. |cos x| dx 2. |sen x| dx 3. |sen (2x)| dx 4. |sen x| dx
0 0 0 0

25. Calcular
Z 1+π Z 2+π/2 Z 1+π
1. |sen x| dx 2. |sen (2x)| dx 3. |cos x| dx
1 2 1

Z 3
sen π3 (x − 2)
26. Calcule la integral dx.
1 (x − 2)2 + 1
Z 4 2 
27. Calcular x − 4x + 3 − |sen (πx)| dx.
0

28. Sea g : R −→ R una función continua. Definimos


Z x
G (x) = g (t) dt
0

para x ∈ R. Demuestre que si G (x) = 0, para todo x ∈ R, entonces g (x) = 0, para todo x ∈ R.

Respuestas: Ejercicios

33 3 16 3
1.1. 4; 1.2. 9; 1.3. − 4; 1.4. π; 1.5. 4; 1.6. 4; 1.7. 5 ; 1.8. 4; 1.9. 3 ; 1.10. 4;
45
√ 1801 15 1 2
1.11. 4 ; 1.12. 1; 1.13. 3; 1.14. 1; 1.15. 22; 1.16. 96 ; 1.17. − 14 ; 1.18. 4π + 1;

5 4 17 4/3 3 1 2 22 81 27
1.19. 4+ 9; 1.20. − 4 a ; 1.21. 15; 1.22. 2π + 2π + 1; 1.23. 5 ; 1.24. − 8 ; 1.25.
4 ;

38 1 π 2
√ √
1.26. 15; 1.27. 0; 1.28. 15 ; 1.29. 0; 1.30. 0; 1.31. − 2; 1.32. 4 +
2 − 1; 1.33. 4 − 2 3;
181 16 13 1
 √
1.34. − 15 ; 1.35. − 3 ; 1.36.
1.37. 4; 12 ;
1.38. − arctan ; 2 1.39. 2 − 1; 1.40. 2; 1.41. 4;
5
√ √ √ √ √ √ √
1.42. 1; 1.43. 1; 1.44. 1.45. 3 5 − 3 3 − 2 2 + 2;
3; 1.46. 3 + 2 + 1; 1.47. 3 3 + 2 − 2;
√ √ √ √
1.48. 4 − 3 − 2 2; 1.49. 5 − 2 − 3; 1.50. 0; 2.1. 45 ; 2.2. 52 ; 2.3. 3
16 ; 2.4. 52
3 ;
√  √  √  √ √
2.5. 2 arcsen 66 ; 2.6. 55 arcsen 2 315 ; 2.7. π246 ; 2.8. 5363 π; 2.9. 122
9 ; 2.10. 2047
11 ; 2.11. 2;
√ 8 1 1 5 1 2 2

π3

2.12. 5 − 1; 2.13. 3; 2.14. 3; 2.15. − 9; 2.16. 36 ; 2.17. 0; 2.18. 72 π ; 2.19. 3 arctan 8 ;
√ 3008 2

5 3

13 3
2.20. 0; 2.21. 18 − 9 2; 2.22. 0; 2.23. 135 ; 2.24. 3; 2.25. 14; 2.26. 0; 2.27. 4 5 − 4 13;
10
√ √
27 3

15 3
2.28. 3 ; 2.29. − 23 3; 2.30. 80 9 − 16 25; 3.1. 5
2; 3.2. 17
4 ; 3.3. − 7
2; 3.4. 4; 3.5. 4;
9 28 1 5 85
√ 4

3.6. 2; 3.7. 3 ; 3.8. 2; 3.9. 3; 3.10. 4; 3.11. 4 ; 3.12. 4; 3.13. 4 3; 3.14. 3 2;
q √ √
3
4. 0; 10. y = cos x, a= π
6; 13. F (x) = 5x + 4; 14.1. c = 3 15
4 ; 14.2. c = 1
2 − 6 , + 63 ;
c= 1
2

√ √
39 1
q q
14.3. c = 21 − 2; 14.4. c = 6 − 2; 14.5. c = − 4 11
3 ; 14.6. c = 115
81 ; 14.7. c = π
4; 14.8. c = 3 − 13
4 ;

20.1. 0; 20.2. 0; 20.3. 6; 20.4. − 6; 20.5. 0; 20.6. 0; 24.1. 8; 24.2. 8; 24.3. 8;


8
24.4. 200; 25.1. 2; 25.2. 1; 25.3. 2; 26. 0; 27. 4 − π;
Bibliografía

1. Purcell, E. - Varberg, D. - Rigdon, S.: “Cálculo”. Novena Edición. PEARSON Prentice Hall.
2. Stewart, J.: “Cálculo”. Grupo Editorial Iberoamericano.

Última actualización: Julio 2016 Farith J. Briceño N. farith.math@gmail.com


Matemática II - Guía 6. Teorema Fundamental del Cálculo. 112

3. Thomas, George: “Cálculo de una variable”. 12ma edición. Pearson.


4. Larson - Hostetler - Edwards, “Cálculo”. Vol. 1. Mc Graw Hill.
5. Leithold, Louis, “El cálculo con geometría analítica”. Harla S.A.

Este material ha sido revisado recientemente, pero esto no garantiza que esté libre de errores, por esa razón se
agradece reportar cualquier error que usted encuentre en este material enviando un mensaje al correo electrónico
farith.math@gmail.com
indicando donde se encuentra(n) dicho(s) error(es). MUCHAS GRACIAS.

Última actualización: Julio 2016 Farith J. Briceño N. farith.math@gmail.com

S-ar putea să vă placă și